You are on page 1of 189

www.cfa-aficionado.cjb.

net
www.marbella.to/cfa-aficionado

479 Questions + Answers


of the

CFA EXAM
Level 1

Study Session :
Global Economics
Introduction by the Author :

Hi there, CFA fellows, here you are . You see , it doesn”t need to be an
expensive prep course to get first class preparation for the CFA exams.

The following questions are original CFA AIMR questions and not just
composed by prep course providers. They all come with a clear answer.

In order to understand why the questions are commented by “answer is


correct / incorrect” it is important to know that all questions have
automatically been responded with the first (and only the first ) answer.

Your CFA-Aficionado

cfa-aficionado@softhome.net
cfa-aficionado@flashmail.com

And now , here we go …

The current exchange rate for French francs is $0.20. For a U.S. bank this is an example of:

* An indirect quote in European terms


* An indirect quote in American terms
* A direct quote in American terms
* A direct quote in European terms

That answer is incorrect.


Correct answer:
A direct quote in American terms

A direct quote provides the home currency price of a specific quantity of foreign currency. An indirect
quote states the foreign currency price of one unit of home currency. A quote in American terms gives the
number of US dollars per units of foreign currency, and in European terms gives the number of foreign
currency units per U.S. dollar.

------------------------------

If a nation is running a "trade deficit," it is

* spending more on public services than it is raising in tax revenues.


* exporting more goods and services than it imports.
* importing more goods and services than it exports.
* encountering a balance of payments disequilibrium.
* worse off as the result of its trade with foreign countries.
That answer is incorrect.
Correct answer:
importing more goods and services than it exports.

A trade deficit implies that a country is importing more value in terms of goods and services than it is
exporting to foreign countries.

------------------------------

Which of the following transactions belong to the Current account?

I. Balance of services.
II. Unrequited transfers.
III. Purchase of a foreign company.
IV. Income from foreign investments.

* I, II, III & IV


* I, II & III
* I only
* I, II & IV
* II only
* I & II
* III only
* IV only

That answer is incorrect.


Correct answer:
I, II & IV

The current account consists of Trade and Services balance, Net income from all foreign investments and
Unrequited transfers, which include aid to foreign countries, losses due to expropriation of property by
foreign governments, etc. Hence, I, II and IV are directly specified as part of the Current account. A
purchase of a foreign firm is a capital investment and as such belongs to the Capital account.

------------------------------

John makes T-shirts and sells them only to American citizens and he purchases one bottle of French wine
with his income. Which of the following will be true?

* John will not be affecting the U.S. trade deficit.


* John will be reducing the U.S. capital account surplus.
* John will be adding to the U.S. trade deficit.
* John will be reducing the U.S. trade deficit.

That answer is incorrect.


Correct answer:
John will be adding to the U.S. trade deficit.

John adds to the U.S. trade deficit because he purchases imports why contributing no exports to the trade
balance.

------------------------------

________ permits the producers of each nation to concentrate on the things they do best while trading for
those things they do least well.

* None of these answers


* A tariff
* A quota
* Trade

That answer is incorrect.


Correct answer:
Trade

Trade leads to mutual gains because it allows each country to specialize more fully in the production of
those things that it does best.

------------------------------

Use the table below, which outlines the production possibilities of Qatar and Botswana in wine and wheat,
to answer the following question.

Qatar Botswana

Wine Wheat Wine Wheat

0 8 0 5
3 6 2 4
6 4 4 3
9 2 8 1
12 0 10 0

The law of comparative advantage suggests that

* both countries would gain if Botswana traded wine for Qatarian wheat.
* neither country would gain from trade, even if the costs for transporting the products were zero.
* Qatar would not gain from trade because it has an absolute advantage in producing both goods.
* both countries would gain if Botswana traded wheat for Qatarian wine.

That answer is correct!

According to the law of comparative advantage, production will be maximized if Qatar produces only
wheat and Botswana produces only wine. This is because total production under this scenario is greater
than any other combination of possibilities. Thus, specialization according to comparative advantage is the
most efficient allocation of production.

------------------------------
What would the bid-ask spread be for pound sterling quoted at $14419-28?

* 6.4%
* 0.64%
* 6.2%
* 0.62%
* 0.63%

That answer is incorrect.


Correct answer:
0.62%

Percent spread = [(1.4428 - 1.4419)/1.4428] x 100 = 0.062%

------------------------------

An appreciation of the U.S. dollar

* is the same thing as an increase in the domestic price level.


* increases the purchasing power of the U.S. dollar in foreign markets for goods and services.
* decreases the purchasing power of the U.S. dollar in foreign markets for goods and services.
* is the same thing as a decrease in the consumer price level.

That answer is incorrect.


Correct answer:
increases the purchasing power of the U.S. dollar in foreign markets for goods and services.

An appreciation of the U.S. dollar implies that it takes less U.S. dollars to purchase one unit of foreign
currency. Thus, a U.S. dollar goes further than it did before. This is an increase in the purchasing power of
the U.S. dollar.

------------------------------

Relative to the French franc, the dollar has over the period 1973 to 1993:

* fallen from a high of 25.5 francs to the dollar in 1973 to a low of 3.4 francs to the dollar in 1985 and risen
again to 12.6 francs to the dollar in 1993.
* none of these answers.
* fallen from a high of 22.5 francs to the dollar in 1973 to a low of 11.1 francs to the dollar in 1985 to 17.6
francs to the dollar in 1993.
* risen from 22.5 francs to the dollar in 1973 to 35.2 francs to the dollar in 1985 to 40.5 francs to the dollar
in 1993.
* has remained relatively stable around 15 francs to the dollar.

That answer is incorrect.


Correct answer:
fallen from a high of 22.5 francs to the dollar in 1973 to a low of 11.1 francs to the dollar in 1985 to 17.6
francs to the dollar in 1993.
The U.S. dollar has moved greatly over this period relative to the French franc. The dollar fell from a high
of 22.5 francs to the dollar in 1973 to a low of 11.1 francs to the dollar in 1985 and then rose again to 17.6
francs to the dollar in 1993.

------------------------------

Which of the following is/are true?

I. All Central bank transactions are excluded from the BOP account.

II. All cash flows associated with foreign investments belong to the Capital account.

III. Repatriation of domestic currency to a foreign country by foreign nationals living in the country are
treated as part of the Current account.

* I & II
* III only
* II only
* III only
* I only

That answer is incorrect.


Correct answer:
III only

All Central Bank transactions are excluded from the Current and Capital accounts and charged to the
Official Reserve account. However, since the Reserve account is part of the BOP account, I is false.
Direct foreign investments are Capital account items. However, income associated with these is treated as
part of the Current account. Hence, II is false.

III is a direct outflow of domestic currency and treated as a debit entry on the Current account.

------------------------------

A ________, which is between a bank and a customer (or another bank), specifies delivery at a fixed
future date, of a fixed amount of one currency against dollar payment.

* foreign exchange contract


* forward contract
* futures contract
* currency arbitrage
* cross rate

That answer is incorrect.


Correct answer:
forward contract

A U.S. firm buys goods from Germany with payment of DM 500,000 due in 90 days. The current price of
the DM is $0.4490, yet may rise against the dollar which increases the dollar cost of the goods. The U.S.
firm can protect itself against this exchange risk by entering into a 90-day forward contract with a bank at a
price of $0.4511. Thus, according to the forward contract, the bank will give the U.S. firm DM 500,000 in
90 days to pay for the goods and the U.S. firm will give the bank $225,550 million (DM 500,000 x 0.4511).
If the spot rate in 90 days is less than $0.4511, the U.S. firm will experience a loss on the forward contract
because it is buying marks for more than the current rate. On the other hand, if the spot rate is higher than
$0.4511, the importer will implicitly profit.

------------------------------

________ is an international banking organization with more than 100 member nations, designed to
oversee the operation of the international monetary system.

* The IMF
* NAFTA
* None of these answers
* The World Bank
* GATT

That answer is correct!

The IMF does not control the world supply of money but does hold currency reserves for member nations
and makes currency loans to national central banks.

------------------------------

The simple or linear annualized short-term interest rate is calculated according to:

* multiply the annual rate by the length of the period in months.


* divide the annual rate by the length of the period in months.
* multiply the annual rate by the length of the period, as a proportion of a year.
* multiply the annual rate by the length of the period as a percentage of six months.
* divide the annual rate by the length of the period as a proportion of the year.

That answer is incorrect.


Correct answer:
multiply the annual rate by the length of the period, as a proportion of a year.

The rate to be paid over the period is equal to the annual rate multiplied by the length of the period, as a
proportion of a year.

------------------------------

________ exchange-rates are determined by the market forces of supply and demand.

* None of these answers


* Fixed
* Mixed
* Flexible

That answer is incorrect.


Correct answer:
Flexible

A system of flexible exchange rates is a system where the exchange rate is determined by the market
forces. Floating exchange rate is synonymous with flexible exchange rate.

------------------------------

Exports of goods (merchandise) and services minus the imports of goods and services equals ________.

* current account
* balance of merchandise trade
* balance on current accounts
* balance on goods and services

That answer is incorrect.


Correct answer:
balance on goods and services

When the balance of service exports and imports are added to the balance of merchandise trade the
balance on goods and services is obtained.

------------------------------

A country cannot maintain currency convertibility if:

* none of these answers.


* it allows the exchange rate value of its currency to fluctuate and follows an independent monetary
policy.
* it fixes the exchange rate value of its currency and has a dependent monetary policy.
* it fixes the exchange rate value of its currency and follows an independent monetary policy.

That answer is incorrect.


Correct answer:
it fixes the exchange rate value of its currency and follows an independent monetary policy.

A country can either follow an independent monetary policy and allow its exchange rate to fluctuate or tie
its monetary policy to the maintenance of the fixed exchange rate.

------------------------------

Under a(n) ________ exchange-rate system, each nation "pegs" the price of its currency to another
currency, such as the dollar, for long periods of time.

* fixed
* none of these answers
* floating
* mixed
That answer is correct!

Fixed exchange rates implies that a county's currency is set at a fixed rate relative to all other currencies;
government policies are used to maintain the fixed rate.

------------------------------

When a domestic resident purchases a foreign stock directly on the foreign stock exchange, the
transaction is entered as a ________ in the BOP account.

* deficit
* surplus
* credit
* debit

That answer is incorrect.


Correct answer:
debit

In BOP accounting, by convention:

1. Any inflow of domestic currency represents a credit and any outflow of domestic currency represents a
debit on the BOP account.

2. Any inflow of foreign currency represents a debit and any outflow of foreign currency represents a credit
on the BOP account.

Thence, direct purchase of a foreign stock, which represent an outflow of domestic currency, is treated as
a debit in the BOP account.

------------------------------

If high-yield investment opportunities attract capital from abroad and lead to a capital account surplus,
then the

* nation will also experience a balance of trade surplus.


* nation must run a current account surplus under a pure flexible exchange rate system.
* nation's currency must depreciate.
* nation must run a current account deficit under a pure flexible exchange rate system.
* nation's currency must appreciate.

That answer is incorrect.


Correct answer:
nation must run a current account deficit under a pure flexible exchange rate system.

If a nation is experiencing a surplus on its capital account balance, it must experience an offsetting deficit
on its current account and vice versa. By definition, the balance of payments must be in balance so that
the capital account offsets the current account.
------------------------------

If a restrictive fiscal policy places downward pressure on real interest rates, we would expect

* all of these answers are correct.


* inflationary pressures to increase.
* an increase in the dollar.
* an outflow of capital.
* an increase in the budget deficit.

That answer is incorrect.


Correct answer:
an outflow of capital.

Investors facing a declining real interest rate would liquidate their assets and move their capital elsewhere
in search of a higher yield. The consequence would be a capital outflow from the country.

------------------------------

The flow of money for the purpose of taking advantage of a covered interest differential is known as
________.

* an outright swap
* covered interest differential
* the swap rate
* covered interest arbitrage
* interest rate parity

That answer is incorrect.


Correct answer:
covered interest arbitrage

The flow of money for the purpose of taking advantage of a covered interest differential is known as
covered interest arbitrage'

------------------------------

Supplementary reserves in the form of accounting entries maintained with the IMF by various
governments are known as:

* Official Reserve Accounts.


* BOP funds.
* Special Depository Receipts.
* Special Drawing Rights.

That answer is incorrect.


Correct answer:
Special Drawing Rights.
Special Drawing Rights are accounting entries established with the International Monetary Fund which can
be used to satisfy debits or credits on the BOP account. Of course, these are simply accounting devices
used to keep track of the assets or debts of the different nations arising from international exchange.
Ultimately, the entries must come to fruition through a supply of goods, services or unrequited transfers.
So if country A uses the SDR to make a short-term payment to country B on its BOP account, it is not
considered to be in default but must ultimately extinguish this debt by supplying goods or services to the
B.

The SDRs are more significant for economies trying to artificially maintain a fixed exchange rate system.
For floating currencies, the BOPs can be at least partially offset through changes in exchange rates.

------------------------------

________ risk is the risk that a bank will deliver currency on one side of a foreign exchange deal while the
counterparty does not send any money in return.

* Trading
* Settlement (Herstatt)
* Foreign Exchange
* Exchange
* Credit

That answer is incorrect.


Correct answer:
Settlement (Herstatt)

In order to minimize credit risk, most banks will transact large amounts only with blue chip customers.

------------------------------

Which of the following would most likely cause a nation's currency to appreciate?

* an increase in real interest rates abroad


* an increase in inflation of the nation's trading partners
* an increase in the nation's domestic inflation rate
* a decrease in domestic real interest rates

That answer is incorrect.


Correct answer:
an increase in inflation of the nation's trading partners

Inflation in a nation's trading partners implies that prices are rising abroad. This would increase the
demand for the nation's output because consumers will prefer imports to domestic output because it will
be relatively less expensive since its prices are not rising. Increased demand for output increases the
demand for the nation's currency and therefore the currency will appreciate.

------------------------------

If restrictive monetary policy results in a deceleration in the domestic inflation rate and higher real interest
rates, other things constant, the

* nation will run a current account surplus.


* nation's currency will appreciate.
* nation will run a capital account deficit.
* nation will run a balance of trade surplus.
* nation's currency will depreciate.

That answer is incorrect.


Correct answer:
nation's currency will appreciate.

Contractionary monetary policy will lead to a deceleration in inflation and higher real interest rates. As a
result, demand for the nation's exports and assets will increase as will the demand for the nation's
currency. This in turn will cause the currency to appreciate.

------------------------------

Which of the following would most likely cause a nation's currency to depreciate?

* an increase in domestic real interest rates


* higher inflation than one's trading partners and a reduction in domestic real interest rates
* a reduction in domestic real interest rates
* higher inflation than one's trading partners
* an increase in exports coupled with a decline in imports

That answer is incorrect.


Correct answer:
higher inflation than one's trading partners and a reduction in domestic real interest rates

Higher relative inflation and a reduction in domestic real interest rates causes the demand for the nation's
exports and assets to decline. This in turn causes the demand for the nation's currency to decline. Once
the demand for the currency falls, the "price" of the currency (or the exchange rate) falls. This is a
depreciation in the currency.

------------------------------

The ______of services from foreigners ________ the supply of dollars to the exchange market.

* import; reduces
* export; expands
* none of these answers
* import; expands

That answer is incorrect.


Correct answer:
import; expands

The import of services from foreigners expands the supply of dollars to the exchange market; service
imports are entered on the balance of payments accounts as debit items.
------------------------------

Which of the following will most likely result from an unanticipated shift to a more expansionary monetary
policy?

* lower real interest rates and a depreciation in the exchange rate of the nation's currency
* higher real interest rates and an appreciation in the exchange rate of the nation's currency
* higher real interest rates and a depreciation in the exchange rate of the nation's currency
* lower real interest rates and an appreciation in the exchange rate of the nation's currency

That answer is correct!

An unanticipated expansionary monetary policy will increase economic growth, accelerate in the inflation
rate and lower real interest rates. These factors lead to an decrease in the demand for the nation's exports
and assets and consequently and decrease in the demand for the nation's currency. This serves to cause
the currency to depreciate.

------------------------------

Both a fixed exchange rate and ________ can be maintained if a country is willing to use its monetary
policy to maintain the fixed exchange rate.

* current account deficit


* currency convertibility
* capital account surplus
* capital account deficit
* current account surplus

That answer is incorrect.


Correct answer:
currency convertibility

A country can either follow an independent monetary policy and allow its exchange rate to fluctuate or tie
its monetary policy to the maintenance of the fixed exchange rate.

------------------------------

If prices in two countries are rising at the same annual rate, then the prices of imports and exports will:

* increase in the country with the more valuable currency and decrease in the country with the less
valuable currency.
* increase relative to domestically produced goods.
* decrease relative to domestically produced goods.
* none of these answers.
* remain unchanged relative to domestically produced goods.

That answer is incorrect.


Correct answer:
remain unchanged relative to domestically produced goods.

If prices in two countries are rising at the same annual rate, then the prices of imports and exports will
remain unchanged relative to domestically produced goods. Equal rates of inflation in each of the
countries will not cause the value of exports to change relative to imports.

------------------------------

The opportunity cost of producing a good:

* generally decreases as the country produces more and more of the good.
* is not dependent on the marginal cost of producing the good.
* generally increases as the country produces more and more of the good.
* is usually uniform across countries.
* none of these answers.

That answer is incorrect.


Correct answer:
generally increases as the country produces more and more of the good.

The opportunity cost of producing a good beyond some level of production will often increase. This is
because of rising marginal costs as the level of output increases. Thus, the degree to which a country will
specialize in the production of a good is limited.

------------------------------

A tariff can be defined simply as a

* tax on imports.
* legal limit on imports.
* tax on exports.
* legal limit on exports.

That answer is correct!

A tariff is a tax levied on goods imported into a country.

------------------------------

________ utilize forward contracts to end (or cover) the chance of loss on export or import orders that are
denominated in foreign currencies.

* Traders
* Hedgers
* All of these answers
* Speculators
* Arbitrageurs
That answer is correct!

Hedgers utilize forward contracts to safeguard the value (in home currency terms) of assets denominated
in a foreign currency on its balance sheet. Speculators choose to be exposed to currency risk (unlike the
three above who attempt to minimize/eliminate risk) by using forward contracts to benefit from exchange
rate fluctuations. Arbitrageurs utilize forward contracts to rid themselves of exchange risk in the currency
markets.

------------------------------

American textile manufacturers and union members have often lobbied successfully for restrictive quotas
limiting the importation of textile products. The major impact of these quotas is

* lower prices for American consumers and an improvement in the quality of textile products available.
* a permanent reduction in unemployment in the U.S.
* higher prices for American consumers, a narrower selection of products and less competition in the U.S.
textile industry.
* long-run profits in the U.S. textile industry that are substantially above market equilibrium.

That answer is incorrect.


Correct answer:
higher prices for American consumers, a narrower selection of products and less competition in the U.S.
textile industry.

Import quotas cause the market price for the imported good to rise domestically. This hurts consumers but
helps domestic producers who in addition to gaining market share also enjoy a higher price.

------------------------------

Which of the following would be a debit in the U.S. balance of payments?

* the purchase of a German car by an American


* the purchase of a German car by an American and a short-term loan extended to a Japanese
manufacturer by a U.S. bank
* the purchase of U.S. grain by a Japanese bakery
* the purchase of air service from a U.S. airline by a Japanese traveler
* a short-term loan extended to a Japanese manufacturer by a U.S. bank

That answer is correct!

Since the American had to trade U.S. dollars for marks in order to purchase the German car this
transaction appears as a debit on the U.S. balance of payments. This is because the transaction
increased the amount of U.S. currency on the foreign exchange market and decreased the amount of
German marks.

------------------------------

Which of the following is/are true about advantages of international trade?


I. It leads to lower prices for imported goods.
II. It leads to lower prices for exported goods.
III. It leads to an expansion of consumption possibilities.
IV. It leads to an expansion of the production possibility frontier.

* I, III & IV
* III & IV
* II only
* I & IV
* I & III
* III only
* I, II, III & IV

That answer is correct!

International trade takes advantage of the comparative advantages that the trading partners have in
producing goods and services. This expands the production possibilities and hence, the consumption
possibilities. Further, in the presence of exports, the supply of an exported good in the domestic market is
lower than it would be otherwise. This causes a higher price for the exported good to prevail in the
domestic market. hence, (II) is incorrect. The reverse occurs for imported goods, so that (I) holds.

------------------------------

In a given year, Gondolpha had total imports of 976 and total exports of 734. It also made direct foreign
investments of 297. There were no other transactions on the BOP account. The Official Reserves account:

* decreases by 1,413
* increases by 539
* decreases by 539
* decreases by 55

That answer is incorrect.


Correct answer:
decreases by 539

Remember the convention:

1. Any inflow of domestic currency represents a credit and any outflow of domestic currency represents a
debit on the BOP account.

2. Any inflow of foreign currency represents a debit and any outflow of foreign currency represents a credit
on the BOP account.

Therefore, the credits on the combined current and capital account equal 734 and the debits equal 976 +
297 = 1,273.

Since debits must equal credits at the end of the accounting period, there must be a credit entry in the
amount of (1,273 - 734) = 539 in the Official Reserves account, since the sum of the changes in the
current, capital and official reserves accounts equals zero over an accounting period. Thus, the Official
Reserves account decreases by 539.

If you do not like the above rule-based treatment of the problem, try the intuition: The total net outflow of
cash from the current and capital accounts of Gondolpha equals 976 + 297 - 734 = 539. This amount has
to come from somewhere. That "somewhere" is represented by the Official Reserves Account, which
loses 539 in order to satisfy the liabilities of the current and capital accounts.

Caution: One of the bizarre quirks in international accounting is that decreases in Official Reserves
Account are represented by a "+."

------------------------------

If the U.S. is viewed by foreigners as a great nation in which to invest, generating a large inflow of foreign
investment, this will cause the U.S. to run a

* deficit on the official reserve account.


* deficit on the capital account.
* deficit on the current account.
* surplus on the current account.

That answer is incorrect.


Correct answer:
deficit on the current account.

If foreigners invest often in the U.S. the U.S. will tend to run a capital account surplus. By definition of the
balance of payments, the U.S. must also tend to run a current account deficit since these two accounts
must counteract each other and sum to zero.

------------------------------

Another name for flexible exchange rates is:

* none of these answers.


* all of these answers.
* floating exchange rates.
* moveable exchange.
* freely determined exchange.

That answer is incorrect.


Correct answer:
floating exchange rates.

Flexible exchange rates, also called floating exchange rates, are determined by market forces.

------------------------------

Suppose the lowest-wage state in the U.S. is West Virginia and the highest-wage state is New York.
Which of the following would be true?

* If New York trades with West Virginia, wages in New York will fall until they equal the wages in West
Virginia.
* New York would be better of if its state government imposed restrictions on the importation of goods
made in West Virginia.
* Both New York and West Virginia will be better of if they are allowed to trade freely.
* If New York trades with West Virginia, consumers in New York will be worse off.

That answer is incorrect.


Correct answer:
Both New York and West Virginia will be better of if they are allowed to trade freely.

Low-wage and high-wage states (like countries) are better off if they are allowed to freely trade with each
other. The comparative advantage of low wage states will be in the production of labor intensive goods
while the advantage for high wage states will be in the production of capital intensive goods. By
specializing in the area of their comparative advantage, total production for each state will increase as will
real income.

------------------------------

The domestic demand Q for a good A at a price P is given by Q = 500 - 5P while the supply function is
given by 300 + 3P. The world price for good X is 19.

If the government imposes a 10% tariff on imports, the revenues of the domestic producers will

* decrease by 233.
* increase by 579.
* increase by 797.
* decrease by 485.

That answer is incorrect.


Correct answer:
increase by 797.

First note that without imports, the price prevailing in the domestic market will satisfy 500 - 5P = 300 + 3P,
giving P = 25. The world price is 19 and with a 5% import tariff, it becomes 19 * 1.1 = 20.9. Since this price
is lower than 25, there will continue to be imports and the price prevailing in the domestic market after the
tariffs will equal 20.9. Before the tariffs, the producers supply a quantity equal to 300 + 3 * 19 = 357 and
have revenues of 357 * 19 = 6,783. With the tariff in place, the producers produce 300 + 3 * 20.9 = 362.7
units and have revenues of 362.7 * 20.9 = 7,580. Thus, the revenues increase by 7,580 - 6,783 = 797.
Note that we have implicitly used the fact that the domestic producers do not have to pay the tariff and
pocket the entire higher price.

------------------------------

Each trading nation can gain by specializing in producing those services for which it is a low-opportunity
cost producer while trading for those things for which it is a high-opportunity cost producer. This statement
best describes the implications of the

* law of comparative advantage.


* infant-industry argument.
* free rider problem.
* equation of exchange.
That answer is correct!

The law of comparative advantage implies that trading partners can be made better off if each specializes
in the production of goods for which it is a low opportunity cost producer and trades for those goods for
which it is a high opportunity cost producer. This will minimize the cost of production and leads to
maximum output.

------------------------------

What gives the home currency price of a certain quantity of the foreign currency quoted?

* bid-ask spread
* spot rate
* cross rate
* direct quotation
* indirect quotation

That answer is incorrect.


Correct answer:
direct quotation

For example, the price of foreign currency is expressed in French francs in France and in Deutsche marks
in Germany. Thus, in France, the Deutsche mark might be quoted at FF 4, whereas in Germany, the franc
would be quoted at DM 0.25.

------------------------------

Given the following data, what will an arbitrageur earn after one year on a covered interest arbitrage if he
begins with $1 million?

In New York today Interest rate = 7%


In London today spot rate = $1.75/L
Interest rate = 12%
In London one year forward rate = $1.68/L

* $7,560
* $5,200
* $4,300
* $3,580
* $6,800

That answer is incorrect.


Correct answer:
$5,200

Begin by borrowing $1,000,000 at 7%, owing $1,070,000 at year end.


Convert the $1,000,000 to pounds at $1.75/L = L571,428.57 and invest this in London at 12% yielding
L640,000 at year end. At the same time, sell the L640,000 forward at a rate of $1.68/L for delivery in one
year yielding $1,075,200 at year end. At the end of the year, collect the L640,000 and deliver to the bank's
foreign exchange dept. in return for $1,075,200. Repay the $1,070,000 from the $1,075,200 leaving a
profit of $5,200.
------------------------------

Which of the following is correct?

* Without tariff protection the number of jobs available to domestic workers would decline.
* Less than 5 percent of world output is sold in a country different from the one in which it is produced.
* Exports represent about 30 percent of U.S. GDP.
* Regarding international trade, if one country gains, another country must lose.
* The volume of international trade has grown rapidly in recent decades.

That answer is incorrect.


Correct answer:
The volume of international trade has grown rapidly in recent decades.

The volume of international trade, enhanced by improved transportation and communication has grown
rapidly in recent years. Approximately 21 percent of the world's total output is now sold in a country other
than that in which it was produced. This is double the figure of three decades ago.

------------------------------

If the one month FF/$ exchange rate is 8.0200-50 this implies:

* the bank is willing to commit itself today to sell dollars in one month for 8.0200 francs or to buy them for
8.0250 francs.
* the bank is willing to commit itself today to buy dollars in one month for 8.0200 francs or to sell them for
8.0250 francs.
* the bank is willing to commit itself today to buy dollars for 8.0200 francs or to sell them for 8.0250
francs.
* the bank is willing to commit itself today to buy dollars in one month for 8.0200 francs or to sell them for
8.0150 francs.
* none of these answers

That answer is incorrect.


Correct answer:
the bank is willing to commit itself today to buy dollars in one month for 8.0200 francs or to sell them for
8.0250 francs.

In a forward, or futures, contract, a commitment is irrevocably made on the transaction date, but delivery
takes place later, here a month later. The bid-ask quotation here implies that the bank is willing to commit
itself today to buy dollars in one month for 8.0200 francs or to sell them for 8.0250 francs.

------------------------------

If the French franc moves from 8.00 to 7.10 francs to the dollar:

* the dollar now trades for 0.141 francs.


* the French franc has depreciated.
* the franc now trades for 7.10 dollars.
* the French franc has appreciated.
* the U.S. dollar has appreciated.

That answer is incorrect.


Correct answer:
the French franc has appreciated.

Since the U.S. dollar now purchases less French francs the U.S. dollar has depreciated; symmetrically,
the French franc has appreciated. Each franc now buys .141 dollars whereas before each franc purchased
.125 dollars.

------------------------------

The exchange rate of a country's currency will ________ if the income of the country and its trading
partners rises.

* neither of these answers


* increase
* This answer depends on which income rises the fastest: for countries that are similar in size and
propensity to import, the country that is growing the fastest will increase its demand for imports relatively
more than its trading partner, resulting in a decrease in the value of the more rapidly growing nation's
currency.
* decrease

That answer is incorrect.


Correct answer:
This answer depends on which income rises the fastest: for countries that are similar in size and
propensity to import, the country that is growing the fastest will increase its demand for imports relatively
more than its trading partner, resulting in a decrease in the value of the more rapidly growing nation's
currency.

Fore countries are similar in size and propensity to import, the country that is growing the fastest will
increase its demand for imports relatively more than its trading partner, resulting in a decrease in the value
of the more rapidly growing nation's currency. Sluggish growth of income relative to one's trading partners
tends to cause the slow-growth nation's currency to appreciate.

------------------------------

The supply curve for British pounds in the foreign exchange market:

* is independent on the purchases of British goods by foreigners.


* none of these answers.
* is dependent on the purchases of foreign goods by the British.
* is dependent on the purchases of British goods by foreigners.

That answer is incorrect.


Correct answer:
is dependent on the purchases of foreign goods by the British.

The supply curve for pounds is dependent on the purchases of foreign goods by British citizens. An
increase in the foreign price of the pound means that a pound will purchase more foreign currency and
more goods priced in terms of foreign currency. The British buy more from abroad and therefore supply
more pounds to the market as the foreign price of the pound rises.

------------------------------

When both exports and imports are considered, the major advantage of international trade is that it allows
us to

* share our technology and efficiency with less-developed countries that would otherwise never have the
opportunity to observe modern goods and services.
* sample foreign products that many of us would otherwise never see.
* consume a larger, more diverse quantity of goods and services at lower prices than would otherwise
prevail.
* maintain jobs for workers who would otherwise have little to do.

That answer is incorrect.


Correct answer:
consume a larger, more diverse quantity of goods and services at lower prices than would otherwise
prevail.

In the absence of trade the consumption of each country is constrained by the country's production
possibilities. Trade expands the consumption possibilities of both countries. Specialization and exchange
permits two countries to expand their joint output and as a result, both countries can increase their
consumption of both all goods.

------------------------------

A tax levied on imported goods is called a(n)

* excise tax.
* foreign profits tax.
* tariff.
* quota.

That answer is incorrect.


Correct answer:
tariff.

A tariff is a tax levied on goods imported into a country.

------------------------------

Under a flexible exchange rate system, a nation that offers more attractive investment opportunities than
its trading partners can expect to run a

* balance of merchandise trade surplus.


* deficit on its capital account transactions.
* surplus on current account transactions.
* deficit on current account transactions.

That answer is incorrect.


Correct answer:
deficit on current account transactions.

Since a high yield on investments will attract capital, the nation will run a capital account surplus which
must be offset by a current account deficit.

------------------------------

What is the working balance that is maintained to facilitate delivery and receipt of currencies?

* nostro account
* fed funds
* position sheet
* contra account
* Clearing House Interbank Payment System

That answer is correct!

The nostro account is a working balance account that is maintained with the correspondent to facilitate
delivery and receipt of currencies.

------------------------------

Which of the following will tend to encourage strong growth in per capita GDP?

* low taxes on international trade


* high tariffs
* strict import quotas
* exchange rate controls

That answer is correct!

The average annual growth of per capita GDP in low restriction countries was 4.7 percent during 1980-91.
Thus, low taxes on international trade (or the absence of trade barriers) appears to be conduce to a strong
growth rate in per capita GDP.

------------------------------

According to the method for calculating short-term interest rates from annualized rates, a three-month
interest rate of 4% means that the interest rate paid after three months will be ________.

* .25%
* 1%
* 2%
* 4%
* none of these answers

That answer is incorrect.


Correct answer:
1%

The rate to be paid over the period is equal to the annual rate multiplied by the length of the period, as a
proportion of a year. Thus: 4%(3/12) = 1%.

------------------------------

If you are about to go to Brazil and would like to obtain Brazilian currency, this acquisition would be
accomplished in the

* internal affairs office.


* Office of the U.S. Treasury.
* Federal Reserve market.
* export/import market.
* foreign exchange market.

That answer is incorrect.


Correct answer:
foreign exchange market.

The foreign exchange market is the market in which the currencies of different countries are bought and
sold. Thus, in order to exchange your dollars for cruzeiros you must access the foreign exchange market.

------------------------------

________ is the sale of goods abroad at a price below their cost.

* Dumping
* Piling
* Transference
* None of these answers

That answer is correct!

Dumping is the sale of a good by a foreign supplier in another country at a price lower than the supplier
sells it in its home market. Dumping may occurs because the exporter wishes to penetrate a foreign
market.

------------------------------

The inevitable consequence of price controls is:

* increased currency convertibility.


* none of these answers.
* the development of black markets.
* trade war.

That answer is incorrect.


Correct answer:
the development of black markets.

Since price controls, of which exchange rate control are a type, lead to shortages, black markets develop.

------------------------------

An unanticipated shift to a more expansionary monetary policy will most likely cause

* depreciation in the exchange rate of the nation's currency, higher real interest rates and an inflow of
capital.
* appreciation in the exchange rate of the nation's currency, higher real interest rates and an inflow of
capital.
* appreciation in the exchange rate of the nation's currency, lower real interest rates and an outflow of
capital.
* depreciation in the exchange rate of the nation's currency, lower real interest rates and an outflow of
capital.

That answer is incorrect.


Correct answer:
depreciation in the exchange rate of the nation's currency, lower real interest rates and an outflow of
capital.

An unanticipated expansionary monetary policy will increase economic growth, accelerate in the inflation
rate and lower real interest rates. These factors lead to an decrease in the demand for the nation's exports
and assets and consequently a decrease in the demand for the nation's currency. This serves to cause
the currency to depreciate. Lower real interest rates cause a capital outflow because investors liquidate
their assets in search of a higher yield abroad.

------------------------------

The dollar would have appreciated if

* the U.S. had a balance of trade surplus under a system of flexible exchange-rates.
* under fixed exchange-rates, the number of dollars demanded in the foreign exchange market increased.
* it had been exchanging for one mark but can now be exchanged for two.
* it had been exchanging for three marks but can now be exchanged for two.

That answer is incorrect.


Correct answer:
it had been exchanging for one mark but can now be exchanged for two.

An increase in the value of a domestic currency relative to foreign currencies defines a currency
appreciation. Thus, the value of the dollar in this example has increased since one dollar now buys two
marks whereas before it only purchased one mark. The U.S. dollar now "buys" more marks.
------------------------------

Under a system of fixed exchange rates, which of the following will most likely increase a balance of
payments deficit?

* an increase in foreign-aid grants


* an increase in foreign-aid grants and expansionary monetary policy, which will drive prices up and
interest rates down
* expansionary monetary policy, which will drive prices up and interest rates down
* a reduction in income from investments abroad
* restrictive monetary policy, which will keep prices down and interest rates up

That answer is incorrect.


Correct answer:
expansionary monetary policy, which will drive prices up and interest rates down

Expansionary monetary policy will create inflation and cause interest rates to fall. Both of these
consequences will cause the demand for U.S. exports and assets to fall which will increase the balance of
payments deficit.

------------------------------

Which of the following items is a credit in the U.S. balance of trade?

* the sale of a life insurance policy to an Italian, by an American company


* the purchase of a U.S. car by a German
* a trip to Japan by a Mexican student
* a short-term loan extended to a South American by a U.S. resident

That answer is correct!

The balance of payments is a summary of all economic transactions between a country and all other
countries for a specific period of time. Transactions that create a demand for the nation's currency (or a
supply of foreign currency) on the foreign exchange market are recorded as a credit or plus item. The only
example of such an occurrence is when and Italian demands U.S. currency in order to purchase a life
insurance policy from a U.S. company. He must pay for the policy in dollars and therefore supplies Italian
lira to the market in exchange for U.S. dollars.

------------------------------

Which of the following is a partially valid economic argument for restricting free trade?

* Infant industries may need temporary protection to develop and gain productive efficiency.
* A nation needs to protect its national defense; hence, it should restrict all foreign competition.
* Restrictions on foreign trade will increase employment and permanently reduce unemployment.
* Removal of restrictions that have existed for years will initially cause inflation.

That answer is correct!


Advocates of the infant-industry argument hold that new domestic industries should be protected from
older, established foreign competitors. As the new industry matures, it will be able to stand on its own feet
and compete effectively with foreign producers, at which time protection can be removed.

------------------------------

A Japanese investor purchasing a U.S. government bond

* causes the yen to appreciate.


* creates a demand for yen and a supply of dollars in the foreign exchange market.
* causes the dollar to depreciate.
* creates a demand for dollars and a supply of yen in the foreign exchange market.

That answer is incorrect.


Correct answer:
creates a demand for dollars and a supply of yen in the foreign exchange market.

A foreigner must pay for a U.S. bond using U.S. dollars. Therefore, the investor must exchange Japanese
yen on the foreign exchange market for U.S. dollars. This transaction increases the supply of Japanese
yen and the demand for U.S. dollars.

------------------------------

When referring to foreign exchange trades, "American terms" refers to:

* a direct quote outside the U.S.


* trades by Americans living abroad
* the number of foreign currency units per U.S. dollar
* an indirect quote in the U.S.
* the number of U.S. dollars per unit of foreign currency

That answer is incorrect.


Correct answer:
the number of U.S. dollars per unit of foreign currency

"European terms" refers to the number of foreign currency units per U.S. dollar.

------------------------------

If a U.S. dollar exchanges for 2.0 German marks, then the dollar price of a mark is

* 50 cents.
* 100 cents.
* 200 cents.
* two cents.

That answer is correct!


The following equality holds: $1=2.0 marks. To determine the value of a mark, divide both sides of the
equation by 2.0 marks. This reveals that one mark buys $.5.

------------------------------

In a purely floating exchange rate economy, a shift toward a more expansionary fiscal policy will move the
capital account toward a ________. The current account will move toward a ________.

* surplus, surplus too


* surplus, deficit
* deficit, surplus
* deficit, deficit too

That answer is incorrect.


Correct answer:
surplus, deficit

An expansionary fiscal policy serves to raise interest rates and increase capital inflow. This moves the
capital account toward a surplus. Since the changes in the capital and current accounts must balance out
in a purely floating exchange rate economy, the current account moves toward a deficit.

------------------------------

Suppose a German-produced car becomes very popular in the U.S. This would tend to

* increase a balance of trade deficit in the U.S.


* reduce any existing balance of trade deficit in the U.S.
* increase a balance of trade surplus in the U.S.
* affect the U.S. balance of payments but not the balance of trade.

That answer is correct!

The balance of payments is a summary of all economic transactions between a country and all other
countries for a specific period of time. Any transaction that supplies the nation's domestic currency in the
foreign exchange market is recorded as a minus item. If the demand for German cars increases individual
U.S. citizens will be supplying U.S. dollars to the foreign exchange market as they exchange them for
German marks (with which they will purchase German cars). As a result, the increased demand for
German cars will cause an increase in the balance of payments deficit in the U.S.

------------------------------

In an open economy, the market demand for domestic products is

* overstated by the horizontal sum of the domestic and foreign demand.


* the horizontal sum of domestic and foreign demand.
* understated by the horizontal sum of the domestic and foreign demand.
* the vertical sum of domestic and foreign demand.
That answer is incorrect.
Correct answer:
the horizontal sum of domestic and foreign demand.

The domestic producer can supply both the domestic consumer and consumers abroad. Thus, market
demand, which is the addition of domestic and foreign demand, can be determined by finding the
domestic demand (at the intersection of the domestic demand curve with the international price) and
adding it to foreign demand (found in the same way with the demand curve for foreigners).

------------------------------

Loans to foreign governments by the Central Bank are treated as

* an Official Reserves account item.


* a Capital account item.
* a Current account item.
* an Off-BOP account item

That answer is correct!

All Central Bank transactions are excluded from the Current and Capital accounts and charged to the
Official Reserve account.

------------------------------

________ utilize forward contracts to safeguard the value (in home currency terms) of assets
denominated in a foreign currency on its balance sheet.

* All of these answers


* Arbitrageurs
* Traders
* Hedgers
* Speculators

That answer is incorrect.


Correct answer:
Hedgers

Speculators choose to be exposed to currency risk (unlike the three above who attempt to
minimize/eliminate risk) by using forward contracts to benefit from exchange rate fluctuations. Arbitrageurs
utilize forward contracts to rid themselves of exchange risk in the currency markets. Traders utilize forward
contracts to end (or cover) the chance of loss on export or import orders that are denominated in foreign
currencies.

------------------------------

When the government puts restrictions on imports, which of the following hold(s)?
I. The domestic consumers of the imported goods are hurt.
II. The domestic producers of the imported goods gain.
III. The domestic consumers of the imported goods gain.
IV. The domestic producers of the imported goods are hurt.

* I & II
* I only
* II only
* III & IV
* IV only
* III only

That answer is correct!

When imports are restricted, the supply of the good in the domestic market is lower than it would be in the
absence of the restrictions. Hence, the price prevailing in the domestic market is higher in the presence of
restrictions. Thus, domestic producers of the imported good gain at the expense of the consumers of that
good.

------------------------------

The supply curve for American dollars in the foreign exchange market:

* none of these answers.


* is dependent on the purchases of American goods by foreigners.
* is independent on the purchases of American goods by foreigners.
* is dependent on the purchases of foreign goods by Americans.

That answer is incorrect.


Correct answer:
is dependent on the purchases of foreign goods by Americans.

The supply curve for dollars is dependent on the purchases of foreign goods by Americans. An increase in
the foreign price of the dollar means that a dollar will purchase more foreign currency and more goods
priced in terms of foreign currency. The Americans buy more from abroad and therefore supply more
dollars to the market as the foreign price of the dollar rises.

------------------------------

Trade restrictions limiting the sale of goods produced in low-wage countries like Mexico will

* reduce real income in the U.S. since the restrictions will keep the U.S. from specializing more fully in
producing those things that it does best.
* increase real income in the U.S., even though the restrictions will retard income levels in low-wage
countries.
* help protect the high wages of workers in the U.S. and at the same time help promote economic
prosperity in the low-wage countries.
* benefit U.S. consumers at the expense of foreign producers.

That answer is correct!


If Mexico is a low wage country and the U.S. is a high wage country then Mexico will have a low
opportunity cost in the production of labor intensive goods. Therefore, if free trade existed, the Mexicans
would export these goods and import capital intensive goods from the U.S. where the opportunity cost for
such goods is low (the opportunity cost for labor intensive goods in the U.S. is high). Under this scenario
each country specializes in the production of goods which it can produce relatively efficiently. If trade is
restricted both countries lose through a reduction in real incomes.

------------------------------

Suppose the government institutes an unexpected expansionary fiscal policy. In the short run,

I. real interest rates will increase.


II. inflation will increase.
III. the domestic currency will depreciate.

* I only
* I & II
* III only
* I & III
* I, II & III
* II only

That answer is incorrect.


Correct answer:
I & II

------------------------------

In a purely floating exchange rate economy, a shift toward a restrictive fiscal policy will move the capital
account toward a ________. The current account will move toward a ________.

* deficit, surplus
* none of these answers
* deficit, deficit also
* surplus, surplus also
* surplus, deficit

That answer is correct!

When the government institutes a restrictive fiscal policy, the aggregate demand decreases and the
inflation rate decreases. Savings increase and the supply of loanable funds rises. Further, since the
government is not as active as a borrower, the demand for loanable funds decreases. This leads to a
reduction in the real rates of interest. Consequently, foreign investment decreases, leading to a decreased
demand for the domestic currency. The domestic currency therefore depreciates.

The capital account moves toward a deficit because of a decreased demand for domestic currency.
Consequently, the current account moves toward a surplus since in a purely floating exchange rate
system, changes in the current and capital account must sum up to zero.
While analyzing questions about current and capital account, always remember that capital account
effects will win out over current account effects due to the high fluidity in the financial markets. Therefore,
always look at the effects on the capital account first.

------------------------------

International trade can benefit domestic firms because:

* the market contracts and allows economies of scale in production.


* consumers purchase more from local firms.
* none of these answers.
* competition is promoted in domestic markets and consumers can purchase a wide diversity of goods at
economic prices.
* opportunity costs for all goods increase.

That answer is incorrect.


Correct answer:
competition is promoted in domestic markets and consumers can purchase a wide diversity of goods at
economic prices.

Competition from abroad helps keep domestic producers on their toes. Domestic producers that otherwise
might have few rivals will have to constantly be seeking ways to improve quality and keep costs low.
Simultaneously, the diversity of goods that is available from abroad provides consumers with a broader
array of choices than would be available in the absence of international trade.

------------------------------

The coupon payments to a domestic investor from a foreign bond is represented as a _______ entry in the
_______ account.

* credit, current
* debit, current
* credit, capital
* debit, capital

That answer is correct!

Income on foreign investments are part of the Current account.

To determine whether it is a debit or a credit entry, remember that by convention,

1. Any inflow of domestic currency represents a credit and any outflow of domestic currency represents a
debit on the BOP account.

2. Any inflow of foreign currency represents a debit and any outflow of foreign currency represents a credit
on the BOP account.

Since the coupon income represents an inflow of domestic currency (from the foreign exchange markets,
where the coupon payments in foreign currency were converted to domestic currency), it is a debit entry
------------------------------

Inflation contributes to the depreciation of a nation's currency only when a country's rate of inflation is
________ that of its trading partners.

* less rapid than


* equal to
* more rapid than
* none of these answers, inflation rate is irrelevant

That answer is incorrect.


Correct answer:
more rapid than

If the rate of inflation in a nation is greater than that of its trading partners prices are rising faster at home
than abroad. As a result, foreign consumers demand less of the nation's exports because they are now
relatively more expensive. Falling demand for exports causes the demand for the nation's currency to
decline and thus the currency depreciates.

------------------------------

Which of the following is true?

* A country with highly attractive (compared with other countries) domestic investment opportunities and a
low saving rate will tend to run a current account deficit.
* A country with relatively poor (compared with other countries) domestic investment opportunities and a
high saving rate will tend to run a current account deficit.
* A nation cannot run a Current account deficit over a long period.
* The U.S. current account deficit is a financial obligation of the federal government.

That answer is correct!

A country which receives heavy foreign investment will tend to run a capital account surplus. According to
the balance of payments, which states that the capital account and current account must sum to zero, the
current account of such a nation must be in deficit.

------------------------------

Since countries continue ________ the current system is ________.

* to engage in official reserve transactions, not a purely flexible rate system


* to use monetary policy to affect the exchange rate, a fixed rate system
* hold special drawing rights at the IMF, a purely flexible rate system
* none of these answers
* to engage in official reserve transactions, a fixed rate system

That answer is correct!


Since countries, to some extent, engage in official reserve transactions, the current system is not a pure
flexible rate system. If it were, there would be no official reserve transactions.

------------------------------

Which one of the following will most likely cause a depreciation in the U.S. dollar on the foreign exchange
market under a system of flexible exchange rates?

* a sudden increase in the demand for California cabernet wines by Europeans


* an increase in real interest rates among European nations
* a switch from French stocks and bonds to U.S. stocks and bonds by U.S. speculators
* rampant inflation in Japan, a major trading partner of the U.S.

That answer is incorrect.


Correct answer:
an increase in real interest rates among European nations

An increase in real interest rates abroad will cause the demand for high yield assets in those countries to
increase. As a result the demand for assets in the U.S. will fall, as will the demand for U.S. currency. The
U.S. dollar will depreciate under falling demand.

------------------------------

State the formula for approximating interest rate parity.

* rh - rf = f1 - e0 * e0
* rh + rf = f1 - e0/e0
* rh + rf = f1 + e0/e0
* rh - rf = f1 + e0/e0
* rh - rf = f1 - e0/e0

That answer is incorrect.


Correct answer:
rh - rf = f1 - e0/e0

Interest rate parity will hold when there are no covered interest arbitrage opportunities.

------------------------------

Capital account transactions include:

* direct investments by Americans in real assets abroad and loans from foreigners.
* direct investments by Americans in real domestic assets and loans to and from foreigners.
* unilateral transfers to and from foreigners and direct investments by Americans in real assets abroad.
* direct investments by Americans in real assets abroad and loans to and from foreigners.
* unilateral transfers to and from foreigners and loans to and from foreigners.

That answer is incorrect.


Correct answer:
direct investments by Americans in real assets abroad and loans to and from foreigners.

Capital account transactions are composed of direct investment by Americans in real assets abroad (or by
foreigners in the U.S.) and loans to and from foreigners.

------------------------------

The domestic demand Q for a good X at a price P is given by Q = 400 - 2P while the supply function is
given by 100 + 4P. The world price for good X is 45 in international markets. All quantities are in
thousands of units. If the government imposes a tariff of 5% on imports, the imports will fall by:

* 9.75
* 13.25
* 16.5
* 19

That answer is incorrect.


Correct answer:
16.5

First note that without imports, the price prevailing in the domestic market will satisfy 400 - 2P = 100 + 4P,
giving P = 50. The world price is 45 and with a 5% import tariff, it becomes 45 * 1.05 = 47.25. Since this
price is lower than 50, there will continue to be imports and the price prevailing in the domestic market
after the tariffs will equal 47.25.

Before the tariffs, the producers supply a quantity equal to 100 + 4 * 45 = 280 and domestic consumers
demand 400 - 2 * 45 = 310 units. Thus, imports without the tariffs equal 300 - 280 = 30 units. With the tariff
in place, the producers produce 100 + 4 * 47.25 = 289 units. The consumers demand 400 - 2 * 47.25 =
305.5 units. Thus, the imports now equal 305.5 - 289 = 16.5 units.

------------------------------

The ________ elasticity of domestic demand for imports and foreign demand for exports is ______.

* short-run; elastic
* long-run; inelastic
* short-run; unit-elastic
* none of these answers
* short-run; inelastic

That answer is incorrect.


Correct answer:
short-run; inelastic

The short run elasticity of domestic demand for imports and foreign demand for exports is inelastic. So a
depreciation initially increases import expenditures and export sales. In the long run, the demand for both
imports and exports is elastic.

------------------------------
I. Free trade with low-wage countries will cause the wages of U.S. workers to fall because high hourly
wages mean higher per-unit labor cost while low hourly wages yield lower per-unit labor costs.

II. Trade restrictions on the importation of goods produced by cheap foreign labor are a major source of
the high standard of living enjoyed by most U.S. workers

* Both statements are false.


* Both statements are true.
* I is false, II is true.
* I is true, II is false.

That answer is correct!

High hourly wages do not necessarily mean high per unit labor cost. Labor productivity must also be
considered: higher productivity coupled with higher labor cost could lead to cheaper per unit costs than
low productivity coupled with low labor cost. Trade restrictions do not increase the standard of living for
U.S. workers; the removal of such restrictions would increase this standard. This is because under free
trade countries could specialize in the area of their comparative advantage and import all other goods. For
the U.S., this would mean producing high technology products and importing labor intensive goods. Free
trade would increase the real income of U.S. workers.

------------------------------

Which transaction comprises the largest portion of a nation's balance of payments account?

* none of these answers


* unilateral transfers
* loans to foreigners
* income from investments
* service exports and imports

That answer is correct!

The export and import of merchandise goods comprise by far the largest portion of a nation's balance-of-
payments account.

------------------------------

During the early 1980s, the U.S. shifted to a more restrictive monetary policy (to fight inflation) and a more
expansionary fiscal policy (tax rates were reduced sharply). How does economic theory indicate that this
policy combination will influence real interest rates and the exchange rate of the dollar?

Real interest rates will ________, the dollar will ________.

* rise; appreciate
* rise; depreciate
* fall; depreciate
* fall; appreciate
That answer is correct!

Restrictive monetary will cause the real interest rate to rise and as a result, the dollar will appreciate.
Restrictive fiscal policy will cause inflation to fall, the exchange rate to appreciate and the real interest rate
to fall. It is assumed then that the exchange rate of the dollar will increase while the real interest rate will
rise as monetary policy has a stronger effect.

------------------------------

Which of the following would be a debit in the U.S. balance of payments?

* a short-term loan extended to a Japanese manufacturer by a U.S. bank


* the purchase of a Japanese car by an American
* the purchase of air service from a U.S. airline by a Japanese traveler
* the purchase of U.S. grain by a Japanese bakery

That answer is incorrect.


Correct answer:
the purchase of a Japanese car by an American

Since the purchase of a car from a Japanese firm by an American increases the supply of U.S. dollars on
the foreign exchange market, this transaction is accounted for as a debit on the U.S. current account.

------------------------------

A(n) ________ in a nation's currency in the foreign exchange market compensates for the nation's high
________ rate.

* depreciation; exchange
* appreciation; exchange
* depreciation; inflation
* appreciation; inflation
* none of these answers

That answer is incorrect.


Correct answer:
depreciation; inflation

Exchange rate adjustments permit nations with even high rates of inflation to engage in trade with
countries experiencing relatively stable prices. A depreciation in a nation's currency in the exchange
market compensates for the nation's inflation rate.

------------------------------

If labor-intensive textile products could be produced more cheaply in low-wage countries than in the U.S.,
the U.S. would gain if it

* used its resources to produce other items while importing textiles from foreigners.
* levied a tax on the domestic textile products to penalize the industry for inefficiency.
* levied a tariff on the goods produced by the cheap foreign labor.
* subsidized the domestic textile industry so it could compete in international markets.

That answer is correct!

Both high wage and low wage countries gain from the opportunity to specialize in those activities that,
relatively speaking, they do best. The comparative advantage of low wage countries is likely to be in the
production of labor intensive goods while the comparative advantage for countries like the U.S. will be in
the production of high technology manufacturing products. Thus, the U.S. should import textiles while
producing aircraft domestically.

------------------------------

Compared to the no-trade situation, when a country exports a good

* domestic consumers gain, domestic producers lose and the losses outweigh the gains.
* domestic consumers gain, domestic producers lose and the gains outweigh the losses.
* domestic producers gain, but domestic consumers lose an equal amount.
* domestic producers gain, domestic consumers lose and the gains outweigh the losses.

That answer is incorrect.


Correct answer:
domestic producers gain, domestic consumers lose and the gains outweigh the losses.

Domestic producers gain under export because only goods for which the international price is higher than
the domestic price will be exported. Thus, the producer will receive a higher price abroad than it is
receiving at home. Domestic consumers lose because producers will reduce their output for domestic
consumption which will cause the price to rise. The gains outweigh the losses because the country will
divert resources away from inefficient production to the efficient production of the exported goods.
Producers will be motivated to divert these resources by higher prices abroad.

------------------------------

How will an unanticipated shift to a more expansionary monetary policy tend to influence the foreign
exchange value of the dollar and the U.S. current account deficit or surplus?

The dollar will ________ and the current account will ________.

* depreciate; move toward a surplus


* appreciate; move toward a deficit
* depreciate; move toward a deficit
* appreciate; move toward a surplus

That answer is correct!

An unanticipated expansionary monetary policy lowers real interest rates. These factors lead to an
decrease in the demand for the nation's assets and to a decrease in the demand for the nation's currency.
This serves to cause the currency to depreciate. A currency deprecation will increase the demand for U.S.
exports which will move the current account toward a surplus.
------------------------------

Which of the following is the best example of a quota?

* a tax placed on all small cars sold in the domestic market


* a limit imposed on the number of men's suits that can be imported from a foreign country
* a $100-per-car excise tax levied on all small cars sold in the domestic market
* a subsidy from the U.S. government to domestic manufacturers of small cars so they can compete more
effectively with foreign producers of small cars
* a $100-per-car fee imposed on all small cars imported

That answer is incorrect.


Correct answer:
a limit imposed on the number of men's suits that can be imported from a foreign country

An import quota is a specific quantity (or value) of a good permitted to be imported into a country during a
given year.

------------------------------

For an upcoming sight-seeing visit to India, a U.S. resident recently purchased a hundred thousand Indian
Rupees. His action

* created a credit balance in the U.S. BOP account.


* created a debit balance in the U.S. BOP account.
* created a deficit in the U.S. trade account.
* none of these answers.

That answer is incorrect.


Correct answer:
created a debit balance in the U.S. BOP account.

This is a point of convention where much confusion exists. Before you answer it, you should remember
two things:

In International Finance, a country's reserve of foreign currency and foreign- produced goods is treated as
an asset.

In Accounting, by convention, increases in assets are recorded as "debit" entries and reductions are
treated as "credit" entries.

In the above example, there is an inflow of Indian Rupees and is hence recorded as a debit to the U.S.
BOP account.

Of course, it is also quite important to remember that the BOP account must balance out, just as surely as
you must have debits = credits in accounting. The subtle part in BOP accounting is to recognize the
offsetting credit entry that arises. Consider how this happens in the above example:

The dollars used to buy the Indian rupees represent a liability for the U.S. This debt will be extinguished
during the year through a supply of goods by U.S. residents in the form of exports or end up as a
recognition of liability on the Official Reserve account when the accounts are balanced at the end of an
accounting period (quarter end or year end). When this happens, the offsetting credit entry shows up on
the accounts and the national balance sheet balances out. Note that exports are thus treated as credits
and imports are treated as debits on the BOP account.

In summary,

1. Any inflow of domestic currency represents a credit and any outflow of domestic currency represents a
debit on the BOP account.

2. Any inflow of foreign currency represents a debit and any outflow of foreign currency represents a credit
on the BOP account.

------------------------------

Unanticipated expansionary monetary policy will increase economic growth and push inflation upward
while lowering real interest rates. This will cause a(n)

* increase in the demand for foreign currencies and a decline in the foreign exchange value of the dollar.
* none of these answers.
* increase in the demand for foreign currencies and an increase in the foreign exchange value of the
dollar.
* decrease in the demand for foreign currencies and a decline in the foreign exchange value of the dollar.
* decrease in the demand for foreign currencies and a decline in the foreign exchange value of the dollar.

That answer is correct!

An unanticipated expansionary monetary policy will increase economic growth, accelerate in the inflation
rate and lower real interest rates. These factors lead to an decrease in the demand for the nation's exports
and assets and consequently and decrease in the demand for the nation's currency. This serves to cause
the currency to depreciate.

------------------------------

During the early 1980s, the U.S. shifted to a more restrictive monetary policy (to fight inflation) and a more
expansionary fiscal policy (tax rates were reduced sharply). How does economic theory indicate that this
policy combination will influence the foreign exchange value of the dollar and the U.S. current account
deficit or surplus?

The dollar will ________, the current account will ________.

* appreciate; move toward a deficit


* appreciate; move toward a surplus
* depreciate; move toward a surplus
* depreciate; move toward a deficit

That answer is correct!

Restrictive monetary will cause the real interest rate to rise and as a result, the dollar will appreciate. This
will decrease U.S. exports and increase the current account deficit. Restrictive fiscal policy will cause
inflation to fall, the exchange rate to appreciate and the real interest rate to fall. Thus, monetary and fiscal
policy both lead to currency appreciation and a movement toward current account deficit.

------------------------------

The price for French francs is $0.1945-67. What is the percent spread for this currency?

* 0.99%
* 1.12%
* 11.3%
* 1.13%
* 11.2%

That answer is incorrect.


Correct answer:
1.12%

The percent spread is the (ask price - bid price)/ask price times 100. In this case, (0.1967 - 0.1945)/0.1967
x 100 = 1.12%

------------------------------

Which of the following factors will cause country A's currency to appreciate relative to the currency of
country B, all else equal?

I. A has higher income growth.


II. A has higher inflation.
III. B has higher real interest rate.

* II & III
* None of them
* I, II & III
* I only
* II only
* III only
* I & III

That answer is incorrect.


Correct answer:
None of them

A country's currency will appreciate relative to those of its trading partners if it has lower income growth
(which will cause the increase in imports to lag behind the increase in exports), has a lower inflation rate
or offers higher real interest rates.

------------------------------

Under a system of flexible exchange rates, an increase in demand for a nation's currency in the foreign
exchange market will
* make it more expensive for the nation to import goods.
* cause the nation's balance on current account to shift toward a deficit.
* cause the nation's currency to appreciate.
* make it less expensive for foreigners to buy the nation's goods.

That answer is incorrect.


Correct answer:
cause the nation's currency to appreciate.

Any increase in the demand for a country' currency causes the currency to appreciate. The exchange rate
is essentially the price of one currency in terms of another currency. Whenever the demand for a good
rises, the price must rise. Therefore the price of a currency rises whenever the demand for it rises.

------------------------------

Trade restrictions that limit the sale of low-price foreign goods in the U.S. market

* help channel more of our resources into producing goods for which we are a low-cost producer.
* benefit domestic producers in the protected industries at the expense of consumers and domestic
producers in export industries.
* increase the real income of Americans.
* reduce unemployment and increase the productivity of American workers.

That answer is incorrect.


Correct answer:
benefit domestic producers in the protected industries at the expense of consumers and domestic
producers in export industries.

Import restriction are essentially subsidies to domestic producers in protected import industries at the
expense of consumers and producers in export industries. This is because consumers pay a higher
market price while domestic producers enjoy a higher market price which increases their revenues.

------------------------------

Which of the following will tend to cause the U.S. dollar to appreciate on the foreign exchange market?

* the purchase of a Mexican shoe factory by a major U.S. shoe manufacturer


* your purchase of an automobile manufactured in Japan
* the purchase of a U.S. government security by a French investor
* expenditures by U.S. college students touring Europe

That answer is incorrect.


Correct answer:
the purchase of a U.S. government security by a French investor

In order to purchase a U.S. asset the French investor must exchange French francs for U.S. dollars since
the U.S. asset must be purchased with U.S. dollars. Therefore, the investor goes to the foreign exchange
market and trades francs for dollars. This transaction increases the demand for U.S. dollars and therefore
causes the dollar to appreciate.
------------------------------

Under a flexible exchange-rate system, which of the following will be most likely to cause an appreciation
in the exchange-rate of the dollar relative to the English pound?

* attractive investment opportunities in England inducing U.S. investors to buy stock in English firms
* higher real interest rates in England
* inflation in the U.S. while prices are stable in England
* an economic boom in England, inducing English consumers to buy more American-made automobiles,
trucks and computer products

That answer is incorrect.


Correct answer:
an economic boom in England, inducing English consumers to buy more American-made automobiles,
trucks and computer products

If English consumers experience an increase in their real incomes their demand for imported goods is
likely to rise. As a result, the demand for U.S. goods will increase as will the demand for U.S. dollars.
Since the exchange rate represents the "price" of U.S. dollars in pounds the U.S. dollar will become more
expensive under greater demand. The dollar is then said to appreciate relative to the English pound: it
now costs more British pounds to purchase one U.S. dollar.

------------------------------

From ________ most of the world operated under ________ exchange rates.

* 1910 to 1944, fixed


* none of these answers.
* 1944 to 1971, fixed
* 1944 to 1971, floating
* 1971 to 1980, fixed

That answer is incorrect.


Correct answer:
1944 to 1971, fixed

From 1944 to 1971 most of the world operated under fixed exchange rates where each nation "pegs" the
price of its currency to another currency for long periods of time.

------------------------------

If a nation wants to maintain a constant exchange rate at a time when supply and demand are causing an
excess of imports over exports, the nation might

* shift to a more restrictive monetary policy.


* shift to a more expansionary monetary policy and tax exports and subsidize imports
* tax exports and subsidize imports.
* reduce its trade barriers (tariffs and quotas)
* shift to a more expansionary monetary policy.
That answer is correct!

A more restrictive monetary policy will cause inflation to fall and interest rates to rise. As a result, the
demand for the nation's exports and assets will increase thus reducing the excess of imports over exports.

------------------------------

Which of the following would most likely cause a nation's currency to depreciate?

* an increase in exports coupled with a decline in imports


* lower real interest rates abroad
* a slower inflation rate than those of its trading partners
* lower domestic real interest rates

That answer is incorrect.


Correct answer:
lower domestic real interest rates

A decline in domestic real interest rates will discourage foreign investors from buying domestic assets. As
a result, foreigners will sell their investments and increase the supply of the nation's currency on the
foreign exchange market. This increased supply of (or diminished demand for) currency causes the
currency to depreciate.

------------------------------

The law of comparative advantage indicates that

* both specialization and exchange will permit trading partners to maximize their joint output and a nation
can gain from trade even when it is at an absolute disadvantage in producing all goods.
* all of these answers.
* specialization and exchange will permit trading partners to maximize their joint output.
* a nation cannot gain from trade when its trading partners are low-wage countries.
* a nation can gain from trade even when it is at an absolute disadvantage in producing all goods.

That answer is correct!

International trade leads to mutual gain because it allows each country to specialize more fully in the
production of those things that it does best. As long as trade is conducted according to the rules of
comparative advantage, total output will increase. Gains from comparative advantage are possible even if
one country has an absolute advantage in all goods. As long as the relative costs of producing the goods
differs across trading countries, gains from specialization and trade will be possible.

------------------------------

What type of foreign exchange transaction accounts for approximately 60 percent of the entire market?

* covered interest arbitrage


* swap transactions
* spot transactions
* currency arbitrage transactions
* forward transactions

That answer is incorrect.


Correct answer:
spot transactions

Spot transactions account for about 60% of the market, forward transactions account for 10%, and the
remaining 30% consists of swap transactions.

------------------------------

Which of the following product categories makes up the largest percent of U.S. imports?

* petroleum, automobiles and clothing


* construction equipment
* phosphoric fertilizers, grains (corn, wheat and soybeans) and fish
* coffee, aluminum and shoes

That answer is correct!

Petroleum, automobiles and clothing represent the greatest imports for the U.S. This reflects the relative
disadvantage the U.S. has in the production of these goods relative to other countries it trades with.

------------------------------

What is the annualized forward discount if French francs are selling for $0.1926 and the 90 day forward
market rate is $0.1920.

* 3.60%
* 1.25%
* 1.56%
* 2.34%
* 0.55%

That answer is incorrect.


Correct answer:
1.25%

The forward discount equals the forward rate minus the spot rate, all divided by the spot rate times 360
divided by the number of days of the forward contract, in this case, (0.192 - 0.1926)/0.1926 x 360/90 =
1.25%.

------------------------------

The record of all transactions with foreign nations that involve the exchange of merchandise goods and
services or unilateral gifts is called the

* balance of trade.
* balance of payments.
* official reserve account.
* capital account.
* current account.

That answer is incorrect.


Correct answer:
current account.

The current account is all payments related to the purchase or sale of goods and services and income
flows during the designated period.

------------------------------

The currency of a nation that is experiencing a high growth in income ________ relative to currency of a
nation that is experiencing a low growth in income, all else equal.

* will depreciate
* will not be affected
* may appreciate or depreciate
* will appreciate

That answer is correct!

An increase in income leads to an increased in demands for goods, including imported goods. This
demand for foreign currencies leads to a depreciation of the domestic currency.

------------------------------

A transaction that creates a demand for foreign currency and a supply of domestic currency in the foreign
exchange markets is recorded as ________.

* a credit to the BOP account


* none of these answers
* an increase in the Balance of Trade account
* a debit to the BOP account

That answer is incorrect.


Correct answer:
a debit to the BOP account

Remember the convention:

1. Any inflow of domestic currency represents a credit and any outflow of domestic currency represents a
debit on the BOP account.

2. Any inflow of foreign currency represents a debit and any outflow of foreign currency represents a credit
on the BOP account.

In this case, the supply of domestic currency to the forex markets represents either an outflow of the
domestic currency (if it originates domestically) or an inflow of foreign currency (if it originates abroad). In
either case, it appears as a debit to the BOP account.

------------------------------

Under a system of fixed exchange rates, an upward revaluation of a nation's currency will cause

* higher levels of employment within the country.


* the nation's exports to increase.
* the nation's investors to reduce their gold holdings.
* the nation's imports to increase.

That answer is incorrect.


Correct answer:
the nation's imports to increase.

An upward revaluation of a nation's currency implies that the purchasing power of the nation's currency
has increased relative to foreign currencies. AS a result, imports are less expensive and therefore the
demand for imports increases.

------------------------------

Which of the following belong to the Current account?

I. Donations by the domestic investors to the International Red Cross.


II. A direct purchase of a minority holding in a foreign company.
III. Supply of F-16 jets and weapons to a developing country.
IV. Purchase of foreign currency by the Central Bank.

* II & IV
* I only
* I & III
* II only
* I, II & IV
* I & II
* IV only
* III only

That answer is incorrect.


Correct answer:
I & III

II represents a Capital account investment. Further, all Central Bank transactions are excluded from the
Current and Capital accounts and charged to the Official Reserve account.

------------------------------
Restrictions on the import of tomatoes reduce the domestic supply in the U.S. Studies indicate that the
demand for tomatoes is elastic. Compared to a free-trade situation, the import restrictions cause

* higher tomato prices and a reduction in consumer expenditures on tomatoes in the U.S.
* lower tomato prices and a reduction in consumer expenditures on tomatoes in the U.S.
* higher tomato prices and an increase in consumer expenditures on tomatoes in the U.S.
* lower tomato prices and an increase in consumer expenditures on tomatoes in the U.S.

That answer is correct!

A reduction in the domestic supply of tomatoes will cause the price to rise. Since demand is elastic, total
expenditure on tomatoes will decrease.

------------------------------

The ________ is the price at which a currency dealer is willing to sell a currency.

* ask price
* bid price
* offer or ask price
* bid or offer price
* offer price

That answer is incorrect.


Correct answer:
offer or ask price

The ask or offer price is the exchange rate at which a dealer is willing to sell a currency.

------------------------------

The following chart indicates the production possibilities of food and clothing per worker day in the U.S.
and Japan.

United States Japan

Food 4 3
Clothing 12 6

Which of the following is true?

* Joint output would be maximized if the U.S. specialized in producing clothing and Japan in producing
food.
* Since Japanese workers produce more of both food and clothing than U.S. workers, no gains from trade
are possible.
* The Japanese are the high-cost producers of both food and clothing.
* Mutual gains from trade could be realized if the U.S. specialized in food production and Japan in
clothing production.
That answer is correct!

The U.S. has a clear comparative advantage in the production of clothing because it is the low opportunity
cost producer of clothing. The opportunity cost of one unit of clothing in the U.S. is 1/3 units of food; the
opportunity cost in Japan is 1/2. Therefore, the gains from specialization and comparative advantage are
realized if the U.S. produces clothing and Japan produces food.

------------------------------

Expansion in the size of the market:

* will always lead to an increase in the opportunity cost of production.


* will create differences in the opportunity costs across nations.
* will always lead to a decline in the opportunity cost of production.
* sometimes permits firms to realize economies of scale in production.
* will create a monopoly in the market for one firm.

That answer is incorrect.


Correct answer:
sometimes permits firms to realize economies of scale in production.

An expansion in the size of the market will permit firms to realize economies that accompany large scale
production, marketing and distribution. Under these circumstances, international trade will allow domestic
firms to produce large outputs and achieve lower costs than would be possible if they were unable to sell
abroad.

------------------------------

If the spot Japanese yen sold at $0.007960, whereas the 180-day forward yen were priced at $0.008184,
what would be the forward premium annualized?

* 5.89%
* 2.74%
* 5.63%
* 2.82%
* 5.47%

That answer is incorrect.


Correct answer:
5.63%

Forward premium annualized = [(0.008184 - 0.007960)/0.007960] x 360/180 = 0.0563 = 5.63%

------------------------------

A nation experiencing an excess of imports over exports can remedy this situation under a fixed rate
system by:

* devaluing its currency.


* heightening its trade barriers.
* following restrictive macroeconomic policy to promote deflation.
* all of these answers
* none of these answers.

That answer is incorrect.


Correct answer:
all of these answers

A country has three choices to remedy the situation of excess imports over exports: a devaluation can
restore equilibrium between the demand and supply of the currency in the exchange market. Enacting
trade barriers in the form of tariffs and quotas reduces imports but is in conflict with economic efficient.
Restrictive macroeconomic policy retards inflation and increases interest rates.

------------------------------

Which is the following statement is not true?

* In an efficient market with no transaction costs, the interest differential should be about equal to the
forward differential.
* According to the interest rate parity theory, the return on a hedged foreign investment will not equal the
domestic interest rate on investments of identical risk.
* If the covered interest differential between two money markets is nonzero, there is an arbitrage incentive
to move money from one market to the other.
* According to interest rate parity theory, the currency of the country with a lower interest rate should be
at a forward premium in terms of the currency of the country with the higher rate.
* Covered interest differentials tend to disappear through covered interest arbitrage.

That answer is incorrect.


Correct answer:
According to the interest rate parity theory, the return on a hedged foreign investment will not equal the
domestic interest rate on investments of identical risk.

Interest parity ensures that the return on a hedged foreign investment will equal the domestic interest rate
on investments of identical risk.

------------------------------

Currency quotations are given in pairs. The first number in the pair is the ________ and the second is the
________.

* ask rate; bid rate


* spot; distant
* bid rate; ask rate
* sell rate; buy rate
* domestic rate; foreign rate

That answer is incorrect.


Correct answer:
bid rate; ask rate
Quotes are given in pairs because customers could be either buyers or sellers. The first number is also
known as the buy rate and the second number is also known as the sell rate.

------------------------------

A depreciation in a currency implies:

* fewer units of the currency are now required to purchase on unit of a foreign currency.
* that foreign goods are now less expensive.
* more units of a foreign currency are now required to purchase the currency.
* none of these answers.
* the purchasing power of the currency has increased.

That answer is incorrect.


Correct answer:
none of these answers.

All of these answers suggest an depreciation in a currency. A depreciation in a currency implies a


decrease in the value of a domestic currency relative to foreign currencies. A depreciation decreases the
purchasing power of the domestic currency over foreign goods.

------------------------------

Suppose the U.S. imposed a tariff on television sets, preventing foreign-produced TVs from freely entering
the U.S. market. Which of the following would most likely occur?

* The price of televisions to U.S. consumers would fall and the demand for U.S. export products would
fall.
* The price of televisions to U.S. consumers would increase and the demand for U.S. export products
would rise.
* The price of televisions to the U.S. consumers would fall and the demand for U.S. export products would
rise.
* The price of televisions to U.S. consumers would increase and the demand for U.S. export products
would fall.

That answer is incorrect.


Correct answer:
The price of televisions to U.S. consumers would increase and the demand for U.S. export products would
fall.

By imposing a tariff on imported televisions into the U.S. market the government encourages domestic
consumers to purchase domestically produced televisions. This is because the tariff causes the price of
foreign TVs to increase; domestic producers will raise their prices also but will capture a greater portion of
the domestic demand. In response to the U.S. imposition of a tariff, trading partners may also restrict trade
and therefore demand for U.S. exports will fall.

------------------------------
Specialization and trade of products A and B will be mutually beneficial to two nations provided that

* gains from trade are identical for each of the countries.


* one country can produce both products with less labor than the other country.
* production is subject to increasing costs in the countries.
* production is subject to constant costs in the countries.
* comparative costs of the products differ between the countries.

That answer is incorrect.


Correct answer:
comparative costs of the products differ between the countries.

As long as the relative costs of producing the two goods differ in the two countries, gains from
specialization and trade are possible. Each county will find it cheaper to trade for goods that can be
produced only at a high opportunity cost.

------------------------------

Quotas are more harmful than tariffs because

* they prohibit foreign producers from selling additional units regardless of how much lower their costs are
relative to domestic producers and they generate no revenue for the government.
* they prohibit foreign producers from selling additional units regardless of how much lower their costs are
relative to domestic producers.
* restrict trade while tariffs do not restrict trade.
* they generate no revenue for the government.
* they encourage free trade while tariffs impose restrictions that increase prosperity domestically.

That answer is correct!

Quotas are more harmful than tariffs because the quota restricts supply domestically and diverts
resources from efficient to inefficient use. Additionally, a tariff transfers revenue from consumers to the
government while a quota transfers revenue from domestic consumers to foreign producers.

------------------------------

Under a depreciation of the U.S. dollar:

* the demand for U.S. dollars will rise as long as foreigners increase their demand for U.S. exports by less
than the percentage of dollar depreciation.
* total expenditure by foreigners on U.S. exports will increase if demand increases by a smaller
percentage than the dollar depreciated.
* foreigners will buy fewer U.S. exports and therefore total expenditure will decline.
* the demand for U.S. dollars will fall as long as foreigners increase their demand for U.S. exports by less
than the percentage of the dollar depreciation.
* total expenditure by foreigners on U.S. exports will decrease if demand increases by a larger
percentage than the dollar depreciated.

That answer is incorrect.


Correct answer:
the demand for U.S. dollars will fall as long as foreigners increase their demand for U.S. exports by less
than the percentage of the dollar depreciation.

The depreciation of the U.S. dollar will make U.S. exports cheaper to foreigners. Unless foreigners
increase the quantity they purchase by the same percentage as the currency depreciated, their demand
for dollars will decrease as a result of the depreciation.

------------------------------

An unanticipated shift to a more expansionary fiscal policy (a shift toward a larger budget deficit) will
cause

* a capital outflow.
* a shift toward a current account surplus.
* a temporary decline in GDP.
* deflation.
* an increase in real interest rates.

That answer is incorrect.


Correct answer:
an increase in real interest rates.

Expansionary fiscal policy pushes domestic interest rates upward due to the crowding out effect.

------------------------------

The foreign exchange market is comprised of:

* none of these answers.


* mostly floating exchange rate currencies.
* mostly fixed exchange rate currencies.
* an equal number of floating and fixed exchange rate currencies.

That answer is incorrect.


Correct answer:
mostly floating exchange rate currencies.

Compared to the size of the exchange rate market, fixed exchange rate transactions are relatively small.
The exchange rate value of the major currencies are determined primarily by market forces.

------------------------------

If the spot rate is FF/$ = 8.0000 and the forward exchange rate is FF/$ = 8.0200 then:

* none of these answers.


* the dollar quotes at a .0200 franc premium.
* the dollar quotes at a .0200 franc discount.
* there is no arbitrage in this market.
* the franc quotes at a .0200 franc premium.
That answer is incorrect.
Correct answer:
the dollar quotes at a .0200 franc premium.

A premium exists when the forward exchange rate is higher than the spot rate and a discount exists
otherwise.

If the one month forward exchange rate is FF/$ = 8.0200 and the spot rate is FF/$ = 8.0000 then the dollar
quotes with a premium of .0200 francs.

------------------------------

What is the difference between the balance of merchandise trade and the balance of payments?

* Only the value of goods imported and exported are included in the balance of merchandise trade, while
the balance of payments includes the value of all payments to and receipts from foreigners.
* The value of goods imported and exported is included in the balance of merchandise trade, while the
balance of payments includes only capital account transactions.
* "Balance of merchandise trade" and "balance of payments" both describe the same international
exchange transactions.
* The value of all goods, services and unilateral transfer is included in the balance of merchandise trade,
while the balance of payments includes both current account and capital account transactions.

That answer is correct!

Balance of merchandise trade is only one component of a nation's total balance of payments. Other
components are balance on goods and service, income from investments and unilateral transfers.

------------------------------

When a country lifts restrictions on exports, the price paid by the domestic consumers of the exported
goods ________.

* decreases or could remain the unaffected


* could remain unaffected
* decreases
* increases

That answer is incorrect.


Correct answer:
increases

When exports are restricted, the supply of the good in the domestic market is higher than it would be in
the absence of the restrictions. Hence, the price prevailing in the domestic market is lower in the presence
of restrictions. So when the restrictions are lifted, exports increase, the domestic supply of the good
decreases and price increases.

------------------------------
Dumping is

* generally beneficial to consumers and imposes costs on the producers of the nation receiving the
"dumped" goods.
* the sale of a good abroad at a cheaper price than what the same good sells for in the producer's
domestic market.
* all of these answers are true.
* a charge often made against foreign producers by the domestic producers of a nation who want to limit
competition from abroad.

That answer is incorrect.


Correct answer:
all of these answers are true.

Dumping is the sale of a good by a foreign supplier in another country at a price lower than the supplier
sells it in its home market. Dumping infuses an economy with low priced goods which benefits consumers
but hurts producers because their market share falls.

------------------------------

Currency A is currently selling at $0.123 and 180-day forward A's are priced at $0.128. Which of the
following is true?

* A is at a forward premium of 2.0%


* A is at a forward discount of 8.13%
* A is at a forward discount of 7.81%
* None of these answers
* A is at a forward premium of 8.13%

That answer is incorrect.


Correct answer:
A is at a forward premium of 8.13%

If the current rate (spot) is less than the forward rate, there is a forward premium, in this case of ((.128-
.123)/.123)(360/180) = 8.13%.

------------------------------

An important explanation for the current account deficit and capital account surplus in the U.S. is that

* we buy more goods from foreigners than they buy from us and Americans find foreign countries an
attractive place to invest.
* we buy more goods from foreigners than they buy from us and foreigners find the U.S. an attractive
place to invest.
* we buy fewer goods from foreigners than they buy from us and foreigners find the U.S. an attractive
place to invest.
* we buy fewer goods from foreigners than they buy from us and Americans find foreign countries an
attractive place to invest.
That answer is incorrect.
Correct answer:
we buy more goods from foreigners than they buy from us and foreigners find the U.S. an attractive place
to invest.

Since the U.S. has many investment opportunities they will tend to run a capital account surplus; this
surplus, however, must be counteracted by a current account deficit which is often seen in the U.S.

------------------------------

The case for fixed exchange rates is based primarily on the argument that fluctuating rates will

* cause a greater imbalance in the current account of a nation's balance of payments.


* cause exchange rates to fall.
* cause the demand for foreign exchange to increase.
* add to the uncertainties of international trade.

That answer is incorrect.


Correct answer:
add to the uncertainties of international trade.

A fixed exchange rate system implies that governments intervene in the foreign exchange market or alter
their economic policies in an effort to maintain the fixed value of their currency. In this way, the
government can completely determine the exchange rate between the nation and all others. Therefore, all
uncertainty has been removed.

------------------------------

Evaluate the following statement: If every time someone in Idaho buys an automobile made in Michigan,
Idaho is worse off. The people of Idaho would be better off if they bought only goods made in Idaho."

* This statement is incorrect because Idaho cannot produce automobiles.


* This statement is incorrect because the people of Idaho are better off specializing in goods like potatoes
that they can produce economically and trade for goods like automobiles that can be produced only at a
high per-unit cost.
* This statement is incorrect because only goods imported from other countries, not other states, harm
the people of Idaho.
* This statement is essentially correct.

That answer is incorrect.


Correct answer:
This statement is incorrect because the people of Idaho are better off specializing in goods like potatoes
that they can produce economically and trade for goods like automobiles that can be produced only at a
high per-unit cost.

The comparative advantage of these two states is bound to differ. It is probably true that the comparative
advantage for Idaho lies in the production of goods such as potatoes which require specific labor and
natural resources that are not as abundant in Michigan; similarly, Michigan has a comparative advantage
in the production of cars because of its specific labor and natural resource endowments. Both states are
better off if free trade exists between them and each can specialize in the area of their comparative
advantage.

------------------------------

The British pound sterling has a bid price of $1.4419 and an ask price of $1.4428. What is the bid-ask
spread?

* 0.62%
* 0.9%
* 0.09%
* none of these answers
* 0.062%

That answer is incorrect.


Correct answer:
0.062%

The bid-ask spread is computed as follows: Percent spread = (Ask price - Bid price)/Ask price X 100, or
1.4428-1.4419/1.4428 X 100 = 0.062%

------------------------------

When referring to foreign exchange trades, "European terms" refers to:

* a direct quote in the U.S.


* trades by Americans living abroad
* an indirect quote outside the U.S.
* the number of foreign currency units per U.S. dollar
* the number of U.S. dollars per unit of foreign currency

That answer is incorrect.


Correct answer:
the number of foreign currency units per U.S. dollar

"American terms" refers to the number of U.S. dollars per unit of foreign currency

------------------------------

Suppose the Brazilian real is quoted at R0.9955-1.0076/U.S.$ and the Thai baht is quoted at B25.2513.
What is the direct quote for the real in Bangkok?

* B30.5541-9456/R
* B25.1376-3654/R
* 24.9481
* B25.0608-5134/R
* B25.1376-4432/R

That answer is incorrect.


Correct answer:
B25.0608-5134/R

Bid cross rate for the real = 25.2513/1.0076 = B25.0608/R


Ask cross rate for the real = 25.3986/0.9955 = B25.5134/R
Therefore the direct quotes for the real in Bangkok are B25.0608-5134.

------------------------------

A U.S. merchandise trade ________ implies that Americans ________.

* deficit; supplying less dollars to the exchange market in order to purchase foreign made goods than
foreigners are demanding for the purchase of American goods
* none of these answers
* surplus; supplying less dollars to the exchange market in order to purchase foreign made goods than
foreigners are demanding for the purchase of American goods
* surplus; supplying more dollars to the exchange market in order to purchase foreign made goods than
foreigners are demanding for the purchase of American goods

That answer is incorrect.


Correct answer:
surplus; supplying less dollars to the exchange market in order to purchase foreign made goods than
foreigners are demanding for the purchase of American goods

A U.S. merchandise trade surplus occurs when the supply of U.S. dollars to the exchange market is less
than the amount of dollars demanded by foreigners to purchase American goods.

------------------------------

A country that fixes the foreign exchange value of its currency above the market rate and limits the
convertibility of its currency will

* improve its standard of living.


* reduce the ability of its citizens to gain from specialization and exchange.
* increase its ability to specialize.
* increase its ability to adopt mass-production techniques.

That answer is incorrect.


Correct answer:
reduce the ability of its citizens to gain from specialization and exchange.

At an artificially high exchange rate, a country's export good s will be extremely expensive to foreigners.
As a result, foreigners will purchase goods elsewhere and the country's exports will be small. A low level
of exports will make it extremely difficult for domestic resident to obtain foreign currency required to
purchase imports. Thus, citizens will be unable to export the goods for which they have a comparative
advantage and import all other goods.

------------------------------
A U.S. investor purchased 1,000 shares of a foreign company on the London Stock Exchange at the
beginning of the year. 6 months ago, it received dividends on these shares. The share purchase is
recorded on the ________ account. The dividends are recorded on the ________ account.

* Current, Capital
* Capital, Capital
* Capital, Current
* Current, Current

That answer is incorrect.


Correct answer:
Capital, Current

All non-Central Bank related capital investments, including portfolio investments, in a foreign country are
considered part of the Capital account. Income on such investments, however, are part of the Current
account. In passing, convince yourself that the share purchase will be a debit entry and the dividend
income will be a credit entry.

------------------------------

Which of the following is a current account transaction?

* a loan by a domestic bank to a foreigner


* a loan by a foreign bank to a domestic manufacturer
* the purchase of a foreign engineering firm by a domestic investor
* the import of shoes by a domestic retailer

That answer is incorrect.


Correct answer:
the import of shoes by a domestic retailer

The current account records all transaction with foreign nations that involve the exchange of merchandise
goods and services, current income derived from investment and unilateral gifts.

------------------------------

If FF/$ = 12.00 and DM/$ = 3.50 then:

* FF/DM = 1
* FF/DM = .292
* DM/FF = 3.43
* FF/DM = 3.43
* None of these answers

That answer is incorrect.


Correct answer:
FF/DM = 3.43

To find FF/DM when given FF/$ and DM/$ implies creating the fraction FF/DM. When this fraction is
created, we get: (FF/$)/(DM/$) or (12.00)/(3.5)=3.43 = FF/DM.
------------------------------

Fixing exchange rates and limiting the convertibility of currency will

* increase productivity and living standards.


* lead to black markets and less trade.
* improve international trade.
* increase the ability of people to gain from specialization.

That answer is incorrect.


Correct answer:
lead to black markets and less trade.

Fixing the exchange rate value of its currency above the market level leads to a shortage of foreign
exchange and a black market will develop.

------------------------------

Which foreign exchange participant uses forward contracts to eliminate or cover the risk of loss on export
of import orders that are denominated in foreign currencies?

* traders
* hedgers
* arbitrageurs
* brokers
* speculators

That answer is correct!

Generally, a forward-covering transaction is related to a specific payment or receipt expected at a


specified point in time.

------------------------------

How much will a trader make through covered interest arbitrage under the following exchange and interest
rate conditions. Borrow $100 at an interest rate of 4%. Convert the dollars to French francs at the spot rate
of $0.1920. Invest the francs in Paris at 8% interest. Sell the francs at the forward rate of 0.1879. What is
the profit on the transaction?

* $4.00
* $3.00
* $1.69
* $2.50
* $5.69

That answer is incorrect.


Correct answer:
$1.69

You borrow $100 and owe $104 at the end of a year. Convert the dollars to francs, at $100/.192 = FF
520.83. Invest the FF 520.83 at 8% interest, yielding FF 562.50 at the end of a year. Sell FF 562.5 forward
for on year at the one year forward rate of 0.1879, for 562.5 x .1879 = $105.69. Repay the loan for $104,
and earn a riskless profit of $1.69.

------------------------------

Suppose that there is a decline in the real interest rate differential between the U.S. and other nations.
This convergence of interest rates will result in a

* slight appreciation of the U.S. dollar.


* very large depreciation of the U.S. dollar.
* very large appreciation of the U.S. dollar.
* slight depreciation of the U.S. dollar.

That answer is correct!

As the U.S. real interest rate increases, the demand for U.S. assets increases as does demand for U.S.
currency. This increased demand causes the U.S. dollar to appreciate.

------------------------------

If the Deutsche mark is selling for $0.58 and the buying rate for the French franc is $0.17, what is the
DM/FF cross rate?

* DM 1 = FF 3.4
* DM 3.4 = FF 1
* 70.69%
* DM .58 = FF 0.17
* 2.41%

That answer is correct!

If the Deutsche mark is selling for $0.58 and the buying rate for the French franc is $0.17, the DM/FF
cross rate is DM 1 = FF 3.4 (.58/.17)

------------------------------

Under a flexible exchange-rate system, the rate that equates demand and supply in the exchange-rate
market also equates the

* debit and credit items on capital account transactions.


* value of the nation's purchases of goods, services and assets from foreigners with the value of the
nation's sales of these items to foreigners.
* value of the nation's merchandise exports with the value of its merchandise imports.
* debit and credit items on current account transactions.
That answer is incorrect.
Correct answer:
value of the nation's purchases of goods, services and assets from foreigners with the value of the
nation's sales of these items to foreigners.

The value of purchases and sales simply represents the price of each on the foreign exchange market. At
equilibrium in the exchange market, supply equals demand for foreign currency. This implies that a certain
amount of U.S. dollars were exchanged for a certain amount of foreign currency. Wherever this
equilibrium exists defines the exchange rate. It must be true in equilibrium then that the value of the
imports to the U.S. from a certain country equals the values of U.S. exports to that country. Without this
equality there would be no equilibrium in the market.

------------------------------

If the value of a nation's merchandise exports exceeds merchandise imports, then the nation is running a

* balance of payments surplus.


* merchandise trade deficit.
* merchandise trade surplus.
* balance of payments deficit.

That answer is incorrect.


Correct answer:
merchandise trade surplus.

The balance of merchandise trade refers to the difference between the value of a country's merchandise
exports and the value of its merchandise imports. A surplus implies that merchandise exports exceed
merchandise imports.

------------------------------

State the formula for calculating a forward premium annualized.

* Forward premium = [(Forward rate - Spot rate) / Spot rate] X 360/Forward contract no. of days
* Forward premium = [(Spot rate - Forward rate) / Spot rate] X 360/Forward contract no. of days
* Forward premium = [(Forward rate - Spot rate) / Spot rate]
* Forward premium = [(Forward rate - Spot rate) / Forward rate] X 360/Forward contract no. of days
* Forward premium = [(Forward rate - Spot rate) / Forward rate]

That answer is correct!

For example, if French francs are currently selling at $0.1339 and 180-day forward FF are priced at a
premium at $0.1350, the forward premium annualized would equal: Forward premium annualized =
[(0.1350 - 0.1339/0.1339)] X 360/180 = 0.0164 = 1.64%

------------------------------

Which one of the following would supply dollars to the foreign exchange market?
* the sale of a U.S. corporation to a Saudi Arabian investor
* the sale of U.S. automobiles to a Mexican consumer
* the purchase of Canadian oil by a U.S. consumer
* the spending of British tourists in the U.S.

That answer is incorrect.


Correct answer:
the purchase of Canadian oil by a U.S. consumer

Since the purchase of oil from a Canadian firm by an American increases the supply of U.S. dollars on the
foreign exchange market, this transaction is accounted for as a debit on the U.S. current account.

------------------------------

The IMF serves to:

* make currency loans to nations experiencing difficulties with their balance of payments.
* control the world supply of money.
* maintain a fixed rate system of currency exchange.
* all of these answers.
* none of these answers.

That answer is correct!

The IMF does not control the world money supply but does hold currency reserves for member nations
and makes currency loans to national central banks when they have difficulties with their balance of
payments.

------------------------------

The ________ for spot transactions is the date on which the monies must be paid to the parties involved.

* trading date
* liquidation date
* settlement date
* valuation date
* value date

That answer is incorrect.


Correct answer:
value date

It is set as the second working day after the date on which the transaction is concluded.

------------------------------

I. If the value of a nation's exports increases relative to the value of its imports, the nation's currency will
tend to depreciate.

II. Other things constant, an increase in a nation's inflation rate will tend to cause the nation's currency to
appreciate in the foreign exchange market.

* Both statements are true.


* Both statements are false.
* I is false, II is true.
* I is true, II is false.

That answer is incorrect.


Correct answer:
Both statements are false.

Under a flexible exchange rate system, it must be true that the capital account plus the current account
sum to zero. Thus, if the nation has a current account surplus this is counteracted with a capital account
deficit. Thus, the exchange rate is determined by this equilibrium. An increase in a nation's inflation rate
will decrease the demand for its exports and consequently cause the currency to depreciate.

------------------------------

An appreciation in a currency implies:

* less units of a foreign currency are now required to purchase the currency.
* the purchasing power of the currency has decreased.
* none of these answers.
* that foreign goods are now more expensive.
* fewer units of foreign currency are now required to purchase on unit of the currency.

That answer is incorrect.


Correct answer:
none of these answers.

All of these answers reflect a depreciation in a currency. An appreciation implies that there has been an
increase in the value of a domestic currency relative to foreign currencies. An appreciation increases the
purchasing power of the domestic currency over foreign goods.

------------------------------

Given the following information, what is the cross rate of yen per won?

Japanese yen: Y135.62/U.S.$


South Korean won: W763.89/U.S.$

* Y5.63258/W
* Y0.28884/W
* Y9.54440/w
* Y0.01540/W
* Y0.17754/W
That answer is incorrect.
Correct answer:
Y0.17754/W

(Y135.62/U.S.$) / (W763.89/U.S.$) = Y0.17754/W

------------------------------

The ________ is the exchange rate at which the dealer is willing to buy a currency.

* spot price
* none of these answers
* offer price
* bid price
* ask price

That answer is incorrect.


Correct answer:
bid price

The bid price is the price that a foreign exchange dealer is willing to buy a currency; it is almost always
true that the bid price will differ from the ask price which is the price that the dealer is willing to sell the
currency for.

------------------------------

If French francs are currently selling at $0.1350 and 180-day forward FF are priced at a discount of
$0.1339, what is the forward discount annualized?

* 0.0164
* 1.64%
* -1.63%
* 1.63%
* 1.46%

That answer is incorrect.


Correct answer:
-1.63%

Forward discount annualized = [(0.1339 - 0.1350/0.1350)] X 360/180 = -0.0163 = -1.63%

------------------------------

The law of comparative advantage indicates that

* specialization and exchange will permit trading partners to maximize their joint output.
* a nation can gain from trade even if its trading partners are low-wage countries.
* all of these answers.
* a national can gain from trade even when it is at an absolute disadvantage in producing all goods.
That answer is incorrect.
Correct answer:
all of these answers.

If trade occurs according to the law of comparative advantage each country will gain by specializing in the
production of goods that it can produce economically and using the proceeds to import goods that would
be expensive to produce domestically. Joint output is maximized since the lowest cost producer produces
each good: this necessarily implies that output will increase because prior to trade each country had to
make each good even if it was an inefficient producer of that good. As long as the relative costs of
producing the two goods differ in the two countries, gain from specialization and trade will be possible
because each country will find it cheaper to trade for goods that can be produced only at a high
opportunity cost. Thus, a country with an absolute disadvantage in all goods can still gain from trade
because it will end up producing the good that it is least bad at. This also implies that countries can gain
from trade with countries with low wage labor. This is because the country with high wage labor will
produce those goods that do not require a large labor input; the low wage country will produce the highly
labor intensive good.

------------------------------

As the result of specialization and trade, according to the law of comparative advantages, total output will

* decline because the less-developed countries will be: driven from the international markets because
they are less able to compete.
* increase since resources will be better directed toward their highest-valued use.
* rise, only when there is an accompanying decline in the total output of one's trading partners.
* rise if a nation is a net exporter and fall if the nation is a net importer of goods and services.
* decline because specialization is costly.

That answer is incorrect.


Correct answer:
increase since resources will be better directed toward their highest-valued use.

International trade leads to mutual gain because it allows each country to specialize more fully in the
production of those things that it does best. As long as trade is conducted according to the rules of
comparative advantage, total output will increase.

------------------------------

The worldwide convention in quoting short-term interest rates is to quote:

* none of these answers.


* there is no single convention: quotes may be three-month, six-month or annual rates.
* simple or linear annualized rates.
* the simple or linear six month rate.
* the simple three-month rate.

That answer is incorrect.


Correct answer:
simple or linear annualized rates.
Short-term interest rate quotes are made according to the simple or linear annualized rate. The rate to be
paid over the period is simply equal to the annual rate multiplied by the length of the period, as a
proportion of a year.

------------------------------

U.S. demand for imports and foreign demand for U.S. exports will be ________.

* highly elastic in the short run


* highly elastic in the long run
* unit elastic as long as the exchange rate is constant
* highly inelastic in the long run
* highly inelastic in the short run

That answer is incorrect.


Correct answer:
highly elastic in the long run

Since American and foreign produced goods are excellent substitutes for one another, there is good
reason to expect that both the U.S. demand for imports and foreign demand for U.S. exports will be highly
elastic in the long run.

------------------------------

Suppose a U.S. firm buys good from Thailand with a payment of Thai Baht 2,000,000 due in 90 days. The
current price of the baht is U.S. 0.02532. The U.S. firm wishes to hedge its currency risk by entering into a
90 day forward contract with a bank at a price of $0.02545. In 90 days the spot rate for Thai baht is
$0.02528.

* The firm can choose not to fulfill the contract because the spot rate at the time baht payment is due is
less than the forward contract price.
* This means the firm will pay the bank $50,560 to fulfill its part of the forward contract.
* This means the firm will pay the bank $50,640 to fulfill its part of the forward contract.
* The firm's loss on the contract is directly related to the current spot rate of $0.02532.
* This means the firm will pay the bank $50,900 to fulfill its part of the forward contract.

That answer is incorrect.


Correct answer:
This means the firm will pay the bank $50,900 to fulfill its part of the forward contract.

The firm pays the bank the price agreed on in the forward contract: 0.02545 x 2,000,000 or $50,900. The
firm experiences a loss on the contract because the forward price is greater than the spot rate at the time
the baht payment must be made. If it had paid at the spot rate at the time the money was due, it would
only have paid 2,000,000 x 0.02528 = $50,560.

------------------------------

The following chart indicates the production possibilities of food and clothing per workday in the U.S. and
Japan.

United States Japan

Food 6* 2
Clothing 2 1

*Units of Output Per Workday

Which of the following is true?

* Since Japanese workers are producing more of both food and clothing, no gains from trade are
possible.
* Joint output would be maximized if the U.S. specialized in producing clothing from Japan in producing
food.
* Mutual gains from trade could be realized if the U.S. specialized in food production and Japan in
clothing production.
* The Japanese are the high-cost producers of both food and clothing.

That answer is incorrect.


Correct answer:
Mutual gains from trade could be realized if the U.S. specialized in food production and Japan in clothing
production.

The U.S. has a clear comparative advantage in the production of food: this is because the opportunity cost
for the U.S. in producing one unit of food is 1/3 units of clothing; the opportunity cost for Japan for
producing one unit of food is 1/2 unit of clothing. The U.S. is the low opportunity cost producer of food.
Therefore, the gains from specialization and comparative advantage are realized if the U.S. produces food
and Japan produces clothing.

------------------------------

Which of the following increase the gains from trade?

* all of these answers


* market determined exchange rates
* a reduction in tariffs
* good weather
* good roads

That answer is correct!

Physical obstacles like bad roads and stormy weather that increase transaction costs will retard the gains
from trade. Additionally, tariffs, quotas, exchange rate controls and other man made trade restrictions
have similar effects. Thus, good roads and weather and reductions in trade restrictions increase the gains
available from free trade.

------------------------------

The law of comparative advantage explains why a nation will benefit from trade when
* it exports goods for which it is a high-opportunity cost producer, while importing those for which it is a
low-opportunity cost producer.
* it exports goods for which it is a low-opportunity cost producer, while importing those for which it is a
high-opportunity cost producer.
* it exports more than it imports.
* its trading partners are experiencing offsetting losses.

That answer is incorrect.


Correct answer:
it exports goods for which it is a low-opportunity cost producer, while importing those for which it is a high-
opportunity cost producer.

The law of comparative advantage implies that trading partners can be made better off if each specializes
in the production of goods for which it is a low opportunity cost producer and trades for those goods for
which it is a high opportunity cost producer. A nation benefits from trade because it imports high
opportunity cost goods and produces low opportunity cost goods. As a result, there is greater output
overall and each nation is at least as well off as under no trade. With international trade, each country can
gain by specializing in the production of goods that it can produce economically and using the proceeds to
import goods that would be expensive to produce domestically.

------------------------------

Central banks will buy and sell currencies in an attempt to:

* dramatically decrease their exchange rate in a short period of time.


* dramatically increase their exchange rate in a short period of time.
* reduce the effects of high price inflation.
* none of these answers.
* reduce sharp swings in the exchange rate.

That answer is incorrect.


Correct answer:
reduce sharp swings in the exchange rate.

Nations sometimes have their central banks buy and sell currencies in an attempt to reduce sharp swings
in the exchange rate. During some years, a nation might build up holdings of foreign currencies while in
other years it might permit them to be drawn out.

------------------------------

Under a system of fixed exchange rates, an upward revaluation of a nation's currency will cause

* a nation's imports to decline.


* a nation's exports to increase.
* higher levels of employment within the country.
* a nation's imports to increase.
* a nation's exports to increase and a nation's imports to decline.

That answer is incorrect.


Correct answer:
a nation's imports to increase.

An upward revaluation implies that the purchasing power of the nation's currency has increased.
Therefore, the imports of the nation will increase since they are relatively less expensive after the
revaluation.

------------------------------

The best source for statistic on the foreign exchange markets is:

* the Eurocurrency market


* none of these answers
* the Federal Exchange Commission.
* the Bank for International Settlement.
* the International Monetary Fund.

That answer is incorrect.


Correct answer:
the Bank for International Settlement.

Although not easy to get, the best source for statistic on the foreign exchange markets is the BIS. Their
survey suggests that a majority of the trades are not motivated by commercial transaction but are
interbank trades.

------------------------------

The bid-ask spread is computed as follows:

* Percent spread = (Ask price - Bid price)/Bid price X 100


* Percent spread = (Ask price - Bid price)/Ask price X 50
* Percent spread = (Bid price - Ask price)/ Ask price X 100
* Percent spread = (Ask price - Bid price)/Ask price X 100
* None of these answers

That answer is incorrect.


Correct answer:
Percent spread = (Ask price - Bid price)/Ask price X 100

The bid-ask spread is the difference between bid and ask rates for a currency and relies on market
conditions for that specific currency.

------------------------------

An increase in the domestic income of a nation's residents will:

* discourage residents from spending additional income on imports so that imports will fall.
* cause exports to fall.
* encourage residents to spend a portion of their additional income on imports so that imports will rise.
* none of these answers.
* cause exports to rise.

That answer is incorrect.


Correct answer:
encourage residents to spend a portion of their additional income on imports so that imports will rise.

An increase in the domestic income will encourage residents to spend a portion of their additional income
on imports. When the income of a nation increases rapidly, the nation's imports tend to rise rapidly as well.

------------------------------

FF/$ is defined as:

* the number of dollars per French franc.


* neither of these answers.
* the number of French francs per dollar.

That answer is incorrect.


Correct answer:
the number of French francs per dollar.

This notation refers to the number of French francs per dollar. Similarly, $/FF refers to the number of
dollars per French franc.

------------------------------

According to the method for calculating short-term interest rates from annualized rates, a three-month
interest rate of 9% means that the interest rate paid after three months will be ________.

* 2.25%
* 1.25%
* 3%
* 4.5%
* none of these answers

That answer is correct!

The rate to be paid over the period is equal to the annual rate multiplied by the length of the period, as a
proportion of a year. Thus: 9%(3/12) = 2.25%

------------------------------

The ________ is the difference between bid and ask rates for a currency and relies on market conditions
for that specific currency.

* spot rate
* cross rate
* ask-bid spread
* none of these answers
* bid-ask spread

That answer is incorrect.


Correct answer:
bid-ask spread

This spread is more narrow for widely traded currencies and for less volatile currencies.

------------------------------

According to the balance of payments, any transaction that:

* supplies the nation's domestic currency in the foreign exchange market is recorded as a credit or plus
item.
* reduces the supply of the nation's domestic currency in the foreign exchange market is recorded as a
debit or minus item.
* creates a demand for foreign currencies in the foreign exchange market is recorded as a credit or plus
item.
* supplies the nation's domestic currency in the foreign exchange market is recorded as a debit or minus
item.
* none of these answers.

That answer is incorrect.


Correct answer:
supplies the nation's domestic currency in the foreign exchange market is recorded as a debit or minus
item.

Balance of payments accounts record any transaction that supplies the nation's domestic currency (or
creates a demand for foreign currency) in the foreign exchange market is recorded as a debit, or minus,
item.

------------------------------

NAFTA will allow:

* the greater movement of goods and investment among the countries of North America.
* all of these answers.
* consumer to have access to a greater variety of goods.
* greater economies of scale in production.
* many products to be produced at a lower per unit cost.

That answer is incorrect.


Correct answer:
all of these answers.

NAFTA will expand the free movement of goods and investment among the countries of North America
and will allow domestic producers access to a larger market. As a result, products will be produced at a
lower per unit cost because producers will be able to plan for and produce a larger volume of output.
Economies of scale are also then more likely. Elimination of trade barrier will provide consumers with
greater variety and an expanded choice among suppliers.

------------------------------

Foreign exchange quotations are usually made:

* in terms of the amount of foreign currency required to purchase one unit of local currency.
* none of these answers.
* in terms of the amount of local currency required to purchase one unit of foreign currency.
* in terms of the amount of local currency that the French franc purchases.
* in terms of how much one unit of foreign currency purchases in terms of the domestic currency.

That answer is incorrect.


Correct answer:
in terms of the amount of local currency required to purchase one unit of foreign currency.

Foreign exchange quotations are usually shown by how much local currency is needed to purchase one
unit of foreign currency. The French will quote the Deutsche mark exchange rate as 3.20 FF/DM while the
Germans quote 0.3125 DM/FF.

------------------------------

An import quota on a product protects domestic industries by

* increasing the foreign supply to the domestic market and thereby lowering the domestic price.
* providing the incentive for domestic producers to improve the efficiency of their operation and thereby
reduce their per-unit costs of production.
* increasing the domestic demand for the product and thereby increasing its price.
* reducing the foreign supply to the domestic market and thereby raising the domestic price.

That answer is incorrect.


Correct answer:
reducing the foreign supply to the domestic market and thereby raising the domestic price.

Foreign supply under a quota is restricted to some maximum amount. Unsatisfied demand will be supplied
by domestic producers who will enjoy a higher price since supply is now restricted. Whenever supply is
restricted the price will rise. Domestic producers are protected in the sense that they enjoy a higher price
incentive to produce more output.

------------------------------

The agreement of the U.S., Canada and Mexico to eliminate tariffs on the shipment of most products
among the three countries by 2003 is called the

* General Agreement on Tariffs and Trade.


* North American Free Trade Agreement.
* Tariff Reduction Act of 1992.
* Uruguay Round.
That answer is incorrect.
Correct answer:
North American Free Trade Agreement.

As a result of the NAFTA agreement, tariffs on the shipment of most products between the three countries
will be eliminated during the next ten years. The agreement will also remove limits on financial
investments, liberalize trade in services such as banking and establish uniform legal requirements for the
protection of intellectual property.

------------------------------

Which of the following provides the foundation of the case for free trade (i.e., no government restrictions
on international trade)?

* the public choice theory


* the equation of exchange
* the law of comparative advantage
* the free rider theorem

That answer is incorrect.


Correct answer:
the law of comparative advantage

The law of comparative advantage implies that trading partners can be made better off if each specializes
in the production of goods for which it is a low opportunity cost producer and trades for those goods for
which it is a high opportunity cost producer. This will minimize the cost of production and leads to
maximum output. Trade restrictions will distort the true production costs of countries and therefore will not
allow the benefits of comparative advantage to be realized. Countries without a comparative advantage in
the production of some goods will end up producing those goods because of the distortion created by
trade restrictions.

------------------------------

If a portfolio manager wishes to exchange Deutsche marks for francs which is the best bank?

* None of these answers


* Bank C: FF/$ = 8.0020-20, DM/$ = 2.5000-30
* Bank B: FF/$ = 8.0000-25, DM/$ = 2.5500-20
* Bank D: FF/$ 8.0025-21, DM/$ = 2.5500-40
* Bank A: FF/$ = 8.0000-25, DM/$ = 2.4500-20

That answer is incorrect.


Correct answer:
Bank A: FF/$ = 8.0000-25, DM/$ = 2.4500-20

To convert marks francs implies selling marks for dollars and then dollars into francs. The best bank is
Bank A because the manager gets 8.0025/2.4500=3.27 francs per mark.

------------------------------
According to the method for calculating short-term interest rates from annualized rates, a three-month
interest rate of 2% means that the interest rate paid after three months will be ________.

* none of these answers


* .5%
* 1%
* .25%
* 2%

That answer is incorrect.


Correct answer:
.5%

The rate to be paid over the period is equal to the annual rate multiplied by the length of the period, as a
proportion of a year. Thus: 2%(3/12) = .5%.

------------------------------

When a nation's currency depreciates, its current account deficit initially widens and eventually shrinks.
This is known as:

* the Lucas effect.


* the Q-curve equilibrium.
* the Phillips curve effect.
* the J-curve effect.

That answer is incorrect.


Correct answer:
the J-curve effect.

The effect of a currency depreciation on the current-account deficit can be broken up into two parts: Price
effect and quantity effect. When the currency depreciates, the sudden fall in the value of the domestic
currency means that the value of the exports measured in the domestic currency falls while that of the
imports rises. Thus, the current account deficit widens immediately, simply because the currency has
depreciated. Over time, when import and export quantities adjust to this currency revaluation, the actual
amounts of import fall and those of exports rise since the currency depreciation has caused exports to
become cheaper and imports costlier. Eventually, an equilibrium is reached where the current account
deficit shrinks from its pre-depreciation level.
This swing in the deficit over time is likened by fanciful economists to "sliding down the hook of the letter
"J" before rising up its stem." Hence the name, "J-curve effect."

------------------------------

Countries that impose high international trade barriers, on average, have

* high rates of economic growth.


* a large export sector.
* low rates of economic growth.
* a large import sector.
That answer is incorrect.
Correct answer:
low rates of economic growth.

The average annual growth of per capita GDP in low restriction countries was 4.7 percent during 1980-91.
In contrast the growth rate of the ten countries with high trade restrictions was minus 0.7 percent.

------------------------------

Nations often adopt trade restrictions because:

* they wish to encourage the dumping of very low priced goods into their economy
* they understand the implications of the law of comparative advantage
* none of these answers
* of a desire to protect domestic infant industries
* of a desire to reduce national security

That answer is incorrect.


Correct answer:
of a desire to protect domestic infant industries

Valid arguments such as the infant industry, national defense and anti-dumping arguments lead to the
protection of specific industries through trade restriction. Economic illiteracy or a failure to comprehend the
implication of the law of comparative advantage and the linkage between exports and imports lead people
to fallaciously believe that trade restrictions increase employment and help keep the wages of Americans
high.

------------------------------

The dollar would have appreciated relative to the peso if

* it had been exchanging for 20 pesos but can now be exchanged for 25.
* it had been exchanging for 10 pesos but can now be exchanged for four.
* the U.S. had a balance of trade deficit under a system of flexible exchange-rates but now has a trade
surplus.
* the U.S. had a balance of trade surplus under a system of flexible exchange-rates but now has a
balance of trade deficit.
* under fixed exchange-rates, the number of dollars demanded in the foreign exchange market increased.

That answer is correct!

If the purchasing power of one currency relative to another increases then the currency is said to have
appreciated. Here, the dollar now purchases more pesos per dollar and so the dollar has appreciated.

------------------------------

"Tariffs and other trade restrictions increase the domestic scarcity of products by reducing the supply from
abroad. Such policies benefit domestic producers of the restricted products at the expense of domestic
consumers."

This statement

* contains one error: the trade restraints do not reduce the domestic supply of products in the protected
industries.
* contains one error: domestic producers gain at the expense of foreign producers rather than domestic
consumers.
* contains two errors: trade restraints do not increase the domestic scarcity of products and neither do
they harm domestic consumers.
* is essentially correct.

That answer is incorrect.


Correct answer:
is essentially correct.

Tariffs benefit domestic producers and the government at the expense of domestic consumers. Producers
do not pay the tariff so they will expand their output in response to the higher market price that prevails
under the tariff. Consumers must pay a higher price. The government gains through tariff revenue paid by
consumers.

------------------------------

The major types of current account transactions are:

* income from investments, service exports and merchandise trade transactions


* service exports, merchandise trade transactions and unilateral transfers
* unilateral transfers and merchandise trade transactions
* service exports and merchandise trade transactions
* all of these answers

That answer is incorrect.


Correct answer:
all of these answers

The major types of current account transactions are income from investments, service exports,
merchandise trade transactions and unilateral transfers.

------------------------------

The currency of a nation that is experiencing a low rate of inflation ________ relative to currency of a
nation that is experiencing a high rate of inflation, all else equal.

* will appreciate
* will not be affected
* may appreciate or depreciate
* will depreciate

That answer is correct!


If country A is experiencing a higher inflation rate than B, then consumers in A increase their demand for
goods from country B, increasing the imports. This demand for foreign currency in A leads to a
depreciation of A's currency. Thus, the currency of a nation that is experiencing a low rate of inflation
appreciates relative to currency of a nation that is experiencing a high rate of inflation, all else equal.

------------------------------

The purchase of a foreign bond is represented as a _______ entry in the _______ account.

* credit, current
* credit, capital
* debit, capital
* debit, current

That answer is incorrect.


Correct answer:
debit, capital

All non-Central Bank related capital investments, including portfolio investments, in a foreign country are
considered part of the Capital account.

To determine whether it is a debit or a credit entry, remember that by convention,

1. Any inflow of domestic currency represents a credit and any outflow of domestic currency represents a
debit on the BOP account.

2. Any inflow of foreign currency represents a debit and any outflow of foreign currency represents a credit
on the BOP account.

Since the investment represents an outflow of domestic currency, it is a debit entry.

------------------------------

Which foreign exchange participant seeks to earn risk-free profits by taking advantage of differences in
interest rates among countries?

* brokers
* hedgers
* arbitrageurs
* traders
* speculators

That answer is incorrect.


Correct answer:
arbitrageurs

Arbitrageurs use forward contracts to eliminate the exchange risk involved in transferring their funds from
one nation to another.
------------------------------

Suppose the per capita income in U.S. were to unexpectedly increase by 5% relative to the per capita
income in Japan. Which of the following best describes the effect of this on the dollar-yen exchange rate?

* The dollar will appreciate by an unknown amount relative to the yen.


* The dollar will appreciate 5% relative to the yen.
* The dollar will depreciate by an unknown amount relative to the yen.
* The dollar will depreciate 5% relative to the yen.

That answer is incorrect.


Correct answer:
The dollar will depreciate by an unknown amount relative to the yen.

If the per capita income in U.S. jumped up relative to that in Japan, then part of this income would be
spent on Japanese goods, increasing the Japanese imports in the U.S. This increased demand for
Japanese goods would raise the value of the yen relative to the U.S. dollar, causing the dollar to
depreciate, till an equilibrium is reached at a higher dollar value of the yen. However, this depreciation is
determined by the amount by which the demand for yen increases and this information is not given in the
problem.

------------------------------

Which of the following is a current account transaction?

* a loan by a foreign bank to a domestic manufacturer.


* a loan by a domestic bank to a foreigner.
* the sale of a domestic asset to a foreign investor.
* the export of shoes by a domestic producer and the import of shoes by a domestic retailer.
* the purchase of a foreign asset by a domestic investor.

That answer is incorrect.


Correct answer:
the export of shoes by a domestic producer and the import of shoes by a domestic retailer.

The current account includes the exchange of merchandise.

------------------------------

Mutual gain from trade is feasible:

* only if one country has an absolute advantage in the production of both goods.
* only if neither country has an absolute advantage in the production of one of the goods.
* even if the resource difference among nations is not large.
* only if the resource differences among nations is very large.
* there is no resource difference among nations.

That answer is incorrect.


Correct answer:
even if the resource difference among nations is not large.

Trade and specialization expand joint output and lead to mutual gain when the resource bases of regions
differ. This difference need not be huge; the only requirement for mutual gain is that a difference exists.

------------------------------

If the value of a nation's merchandise exports exceeds merchandise imports, then the nation is running a

* capital account surplus.


* balance of payments surplus.
* balance of payments deficit.
* balance of trade deficit.
* balance of merchandise trade surplus.

That answer is incorrect.


Correct answer:
balance of merchandise trade surplus.

The balance of merchandise trade is the difference between the value of a country's merchandise exports
and the value of its merchandise imports. Whenever this difference is positive the balance of merchandise
trade is said to be in surplus.

------------------------------

Which of the following is the premise of a fixed rate system of exchange rates?

* The exchange rate is never manipulated with macroeconomic policy.


* The exchange rate is set by the market equilibrium.
* A nation pays for its imports with exports.
* The exchange rate is never devalued.

That answer is incorrect.


Correct answer:
A nation pays for its imports with exports.

The premise of a fixed rate system is that a nation pays for its imports with exports. Countries may
temporarily experience periods where imports exceed exports. During such period, nations can draw down
their reserve balance.

------------------------------

When the government puts restrictions on exports, which of the following holds?

* The domestic producers of the export goods are hurt and the domestic consumers of the export goods
gain.
* The domestic consumers of the export goods are hurt.
* The domestic producers of the export goods are hurt.
* The domestic consumers of the export goods gain.
That answer is correct!

When exports are restricted, the supply of the good in the domestic market is higher than it would be in
the absence of the restrictions. Hence, the price prevailing in the domestic market is lower in the presence
of restrictions. Thus, domestic consumers of the exported good gain at the expense of the domestic
producers of the good, who would have reaped a higher price with exports.

------------------------------

Currencies are traded in the ________ market, where deliveries generally occur within two business days
after the transaction has been concluded.

* foreign exchange
* stock
* spot
* options
* interbank

That answer is incorrect.


Correct answer:
spot

In the forward market, currencies are bought and sold for future delivery.

------------------------------

Which of the following does not restrict the volume of international trade?

* tariffs
* quotas
* high transportation costs
* a stable international monetary framework

That answer is incorrect.


Correct answer:
a stable international monetary framework

Barriers to trade include quotas, tariffs and high transportation costs. Thus, these three restrict the volume
of international trade. A stable international monetary framework encourages trade because countries can
easily exchange goods through with stable exchange rates.

------------------------------

An unanticipated shift to a more restrictive monetary policy will cause

* real interest rates to decline.


* a capital outflow.
* the dollar to appreciate.
* a shift toward a current account surplus.
* the growth rate of GDP to increase.

That answer is incorrect.


Correct answer:
the dollar to appreciate.

Unanticipated restrictive monetary policy will lead to a contraction in growth, a deceleration in inflation and
higher interest rates. As a result the demand for the nation's exports and assets will increase as will the
demand for the currency. This will cause the current to move toward a surplus and dollar will appreciate.

------------------------------

The following chart indicates the production possibilities for food and clothing per workday in the U.S. and
South Korea.

United States Korea

Food 2* 6
Clothing 1 2

*Units of Output Per Workday

Which of the following is true?

* Mutual gains could be realized if the U.S. specialized in producing food and South Korea in producing
clothing.
* South Korean workers are the low-cost producers of food.
* Mutual gains could be realized if the U.S. specialized in clothing production and South Korea in food
production.
* Both mutual gains could be realized if the U.S. specialized in clothing production and South Korea in
food production and South Korean workers are the low-cost producers of food.

That answer is incorrect.


Correct answer:
Both mutual gains could be realized if the U.S. specialized in clothing production and South Korea in food
production and South Korean workers are the low-cost producers of food.

The U.S. has a clear comparative advantage in the production of clothing. The opportunity cost of
producing a unit of clothing in the Us is 2 units of food; the opportunity cost of one unit of clothing in Korea
is three units of food. Therefore, the gains from specialization and comparative advantage are realized if
the U.S. produces clothing and Korea produces food. Korea is the low wage country because both food
and clothing production it has a lower opportunity cost in the production of both of these goods which are
highly labor intensive.

------------------------------

A student group from your university touring the United Kingdom

* causes the U.S. dollar to appreciate.


* causes the English pound to depreciate.
* creates a demand for English pounds and a supply of dollars in the foreign currency market.
* creates a demand for dollars and a supply of English pounds in the foreign currency market.

That answer is incorrect.


Correct answer:
creates a demand for English pounds and a supply of dollars in the foreign currency market.

The group must exchange its home currency for British pounds. This creates a demand for British pounds
and a supply of foreign currency in the foreign exchange market.

------------------------------

When a country fixes the price of foreign exchange (in terms of the domestic currency) below equilibrium,
which of the following will result?

* A depreciation of the currency will restore equilibrium in the foreign exchange market.
* Residents of that country can buy as much foreign currency as they want.
* There will be a surplus of foreign exchange.
* There will be a shortage of foreign exchange.

That answer is incorrect.


Correct answer:
There will be a shortage of foreign exchange.

Fixing the exchange rate value of a currency below the market level will lead to a shortage of foreign
exchange.

------------------------------

A nation experiencing an excess of imports over exports can remedy this situation under a fixed rate
system by:

* all of these answers.


* upwardly revaluing its currency.
* following restrictive monetary policy to induce deflation.
* following expansionary policy.
* none of these answers.

That answer is incorrect.


Correct answer:
following restrictive monetary policy to induce deflation.

A country has three choices to remedy the situation of excess imports over exports: a devaluation can
restore equilibrium between the demand and supply of the currency in the exchange market. Enacting
trade barriers in the form of tariffs and quotas reduces imports but is in conflict with economic efficient.
Restrictive macroeconomic policy retards inflation and increases interest rates.

------------------------------
The primary organizers of the foreign exchange market are:

* individual consumers of foreign goods and services.


* national governments.
* none of these answers.
* domestic banks and currency brokers.
* international commercial banks and currency brokers.

That answer is incorrect.


Correct answer:
international commercial banks and currency brokers.

The foreign exchange market is a widely dispersed, highly organized market in which the currencies of
different countries are bought and sold. Commercial banks and currency brokers around the world are the
primary organizers of the market.

------------------------------

If FF/$ = 8.00 and DM/$ = 2.50 then:

* FF/DM = .3125
* FF/DM = 3.20
* None of these answers
* FF/DM = 1
* DM/FF = 3.20

That answer is incorrect.


Correct answer:
FF/DM = 3.20

To find FF/DM when given FF/$ and DM/$ implies creating the fraction FF/DM.

When this fraction is created, we get: (FF/$)/(DM/$) or (8.00) / (2.5) = 3.20 = FF/DM.

------------------------------

If a portfolio manager wishes to exchange French francs for Deutsche marks which is the best bank?

* Bank D: FF/$ 8.0025-20, DM/$ = 2.5000-40


* Bank A: FF/$ = 8.0020-25, DM/$ = 2.4500-20
* None of these answers
* Bank C: FF/$ = 8.0020-20, DM/$ = 2.5500-30
* Bank B: FF/$ = 8.0000-25, DM/$ = 2.5500-20

That answer is incorrect.


Correct answer:
Bank C: FF/$ = 8.0020-20, DM/$ = 2.5500-30
To convert francs to marks implies selling francs for dollars (so you want the ask price to be low) and then
buying marks with dollars (so you want to bid price to be high). The best bank is Bank C because the
manager gets 2.5500/8.0020=.319 mark per franc.

------------------------------

The ________ elasticity of domestic demand for imports and foreign demand for exports is ______.

* long-run; elastic
* long-run; inelastic
* short-run; unit-elastic
* none of these answers
* short-run; elastic

That answer is correct!

The short run elasticity of domestic demand for imports and foreign demand for exports is inelastic. So a
depreciation initially increases import expenditures and export sales. In the long run, the demand for both
imports and exports is elastic.

------------------------------

International loanable funds:

* tend to move toward areas where the expected real rate of return (after compensation for differences in
risk) is lowest.
* tend to move toward areas where the expected rate of is highest.
* none of these answers.
* tend to move away from areas where the expected real rate of return (after compensation for
differences in risk) is highest.
* tend to move toward areas where the expected real rate of return (after compensation for differences in
risk) is highest.

That answer is incorrect.


Correct answer:
tend to move toward areas where the expected real rate of return (after compensation for differences in
risk) is highest.

International loanable funds tend to move toward areas where the expected real rate of return (after
compensation for differences in risk) is highest. An increase in the real interest rate in the U.S. will attract
investors from all over the world to invest in the U.S.

------------------------------

If a nation wants to maintain a fixed exchange rate at a time when supply and demand are causing the
exchange rate of its currency to appreciate, the nation might

* shift to a more restrictive monetary policy.


* expand the money supply to push its price level (or inflation rate) upward.
* increase its tariffs and/or adopt more restrictive quotas.
* reduce taxes and run a budget deficit to push domestic interest rates up.

That answer is incorrect.


Correct answer:
expand the money supply to push its price level (or inflation rate) upward.

An expansionary monetary policy will create inflation which will drive down the demand for exports and
consequently the demand for the nation's currency. The result will be a depreciation in the exchange rate
value of the currency.

------------------------------

Gains from exchange depend on the law of ________ advantage.

* comparative
* none of these answers
* absolute
* experience

That answer is correct!

The law of comparative advantage implies that both countries involved in trade can benefit from free trade.
This is because each country imports the good which has a high opportunity cost domestically and exports
the good which has a low opportunity cost domestically.

------------------------------

If a portfolio manager wants to buy francs with dollars which is the best quotation from his point of view?

* Bank B: FF/$ = 7.9985-20


* None of these answers
* Bank C: FF/$ = 8.0000-25
* Bank D: FF/$=7.9995-35
* Bank A: FF/$ = 7.9955-30

That answer is incorrect.


Correct answer:
Bank C: FF/$ = 8.0000-25

The manager will choose the bank that offer the most francs per dollar which is Bank C.

------------------------------

The ________ relation states that the forward discount or premium is equal the interest rate differential
between two currencies.

* all of these answers


* currency
* balance of payments
* exchange rate parity
* interest rate parity

That answer is incorrect.


Correct answer:
interest rate parity

The interest rate parity relation derives from the fact that arbitrage exists. If it did not, riskless arbitrage
would occur.

------------------------------

With time, an appreciation in the value of the nation's currency in the foreign exchange market would
cause

* the nation's exports to increase and imports to decline.


* both imports and exports to rise.
* both imports and exports to decline.
* the nation's imports to increase and exports to decline.

That answer is incorrect.


Correct answer:
the nation's imports to increase and exports to decline.

Appreciation in currency simply implies that the currency has become more expensive for foreigners to
buy. Thus, under a currency appreciation a nation's exports will decrease because the demand from
abroad will fall (this is because the nation's output is now more expensive to foreigners). Similarly, foreign
currency becomes less expensive for the nation's citizens and therefore their demand for imported goods
rises because they are now relatively less expensive. As a result the imports rise and the exports decline.

------------------------------

Which is the following statement is not true?

* Covered interest differentials tend to disappear through covered interest arbitrage.


* If the covered interest differential between two money markets is nonzero, there is an arbitrage incentive
to move money from one market to the other.
* According to interest rate parity theory, the currency of the country with a lower interest rate should be
at a forward premium in terms of the currency of the country with the higher rate.
* According to the interest rate parity theory, the return on a hedged foreign investment will not equal the
domestic interest rate on investments of identical risk.
* In an efficient market with no transaction costs, the interest differential should be about equal to the
forward differential.

That answer is incorrect.


Correct answer:
According to the interest rate parity theory, the return on a hedged foreign investment will not equal the
domestic interest rate on investments of identical risk.
Interest parity ensures that the return on a hedged foreign investment will equal the domestic interest rate
on investments of identical risk.

------------------------------

If a German student pays her way to attend Harvard University, her actions will

* cause the U.S. dollar to depreciate.


* create a supply of dollars and a demand for marks in the foreign currency market.
* create a supply of marks and a demand for dollars in the foreign currency market.
* cause the German mark to appreciate.

That answer is incorrect.


Correct answer:
create a supply of marks and a demand for dollars in the foreign currency market.

A German must pay for a U.S. education using U.S. dollars. Therefore, the student must exchange her
German marks on the foreign exchange market for U.S. dollars. This transaction implies supplying
German marks and demanding U.S. dollars.

------------------------------

Which of the following restricts the volume of international trade?

* a stable international monetary framework


* tariffs
* all of these answers
* quotas
* both quotas and tariffs

That answer is incorrect.


Correct answer:
both quotas and tariffs

Both quotas and tariffs represent barriers to trade in the sense that they distort the price level and amount
of imports in an economy. Trade barriers restrict the volume of international trade.

------------------------------

I. The wages of U.S. workers would decline if we traded freely with low-wage countries such as China.

II. If the wages of U.S. steel workers were three times the wages of steel workers in another country, then
the labor cost per unit of output in the U.S. must also be higher than the per-unit labor cost in the other
country.

* Both statements are true.


* I is true, II is false.
* I is false, II is true.
* Both statements are false.

That answer is incorrect.


Correct answer:
Both statements are false.

Under free trade with China the U.S. would export goods in which it has a comparative advantage. It is
likely that the U.S. will have such an advantage in the production of capital intensive goods since labor is
more expensive in the U.S. than in China. The wages of U.S. workers will not fall because the price of the
good which they produce for export (capital intensive goods) will increase through trade with China. The
wages of U.S. steel workers may be higher than foreign workers but because of the presence of capital
investment in the U.S. the per unit cost labor cost for each unit of steel may be lower than that abroad.
U.S. workers may be more efficient than foreign workers due to capital investments.

------------------------------

Which of the following is/are true about Balance of Payments accounting?

I. Imports are treated as debits to the BOP account.


II. Sale of foreign currency is entered as a credit to the BOP account.
III. Payments of coupons to foreign bond holders are entered as debits to the BOP account.

* I & III
* I only
* II only
* II & III
* I, II & III
* III only
* I & III

That answer is incorrect.


Correct answer:
I, II & III

In BOP accounting, by convention:

1. Any inflow of domestic currency represents a credit and any outflow of domestic currency represents a
debit in the BOP account.

2. Any inflow of foreign currency represents a debit and any outflow of foreign currency represents a credit
in the BOP account.

Thence, imports and coupon payments to foreigners, which represent outflows of domestic currency, are
treated as debits while the sale of foreign currency represents a credit to the BOP account.

------------------------------

Which foreign exchange participant engages in forward contracts to protect the home currency value of
various foreign currency-denominated assets and liabilities on their balance sheets?

* hedgers
* brokers
* traders
* speculators
* arbitrageurs

That answer is correct!

Hedgers engage in forward contracts to protect the home currency value of various foreign currency-
denominated assets and liabilities on their balance sheets that are not to be realized over the life of the
contracts.

------------------------------

Under a system of flexible exchange rates, in the long run a nation's balance on current and capital
account transactions will tend to

* net out to zero, indicating a balance between the debit and credits.
* move toward a deficit and increase if the nation is running a balance of trade surplus and decrease if it
is running a balance of trade deficit.
* move toward a surplus.
* move toward a deficit.
* increase if the nation is running a balance of trade surplus and decrease if it is running a balance of
trade deficit.

That answer is correct!

If a nation is experiencing a deficit on its current account balance, it must experience an offsetting surplus
on its capital account and vice versa. By definition, the balance of payments must be in balance so that
the capital account offsets the current account.

------------------------------

Most U.S. dollar trades in the foreign exchange market are cleared through the ________ which is a
computerized network for the transfer of international dollar payments.

* central bank
* foreign exchange market
* Clearing House Interbank Payments System (CHIPS)
* clearing house
* exchange

That answer is incorrect.


Correct answer:
Clearing House Interbank Payments System (CHIPS)

The Clearing House Interbank Payments System is also known as CHIPS.

------------------------------
An import quota on a product protects domestic industries by

* increasing the domestic demand for the product and thereby increasing its price.
* forcing domestic industries to compete in the international market.
* providing the incentive for domestic producers to improve their efficiency and reduce their per-unit cost
of production.
* increasing the foreign supply to the domestic market and thereby lowering the domestic price.
* reducing the foreign supply to the domestic market and thereby raising the domestic price.

That answer is incorrect.


Correct answer:
reducing the foreign supply to the domestic market and thereby raising the domestic price.

Domestic industries are protected under import quotas since foreign competitors are restricted to a certain
level of imports. As a result, restricted supply in the domestic economy increases the price of the good
and allows the domestic producer to supply a greater share of the market. The domestic producer is
protected in the sense that he enjoys a higher market price and a greater market share under the import
quota.

------------------------------

As the result of specialization and trade, according to the law of comparative advantage, total output will

* decline because specialization is costly.


* increase since resources will be better directed toward their highest-valued use.
* rise only when there is an accompanying decline in the total output of one's trading partners.
* rise if a nation is a net exporter and fall if the nation is a net importer of goods and services.

That answer is incorrect.


Correct answer:
increase since resources will be better directed toward their highest-valued use.

The law of comparative advantage predicts an increase in total output because each country will use more
of its resources to produce those goods that it can produce at a relatively low cost. Opportunity cost
reveals the low cost producer of each good. The reallocation of resources according to the lowest cost
producer in each good will cause total output to increase.

------------------------------

Which of the following are components of the BOP account?

I. Current account.
II. Capital account.
III. Official Reserve account.
IV. Foreign exchange account.

* II only
* III only
* I & II
* IV only
* III & IV
* I only
* I, II & III
* I, II, III & IV

That answer is incorrect.


Correct answer:
I, II & III

You should very carefully study the section. You can expect 2 to 3 questions on this subject in the exam.

------------------------------

A U.S. merchandise trade ________ implies that Americans ________.

* none of these answers


* surplus; supplying more dollars to the exchange market in order to purchase foreign made goods than
foreigners are demanding for the purchase of American goods
* deficit; supplying more dollars to the exchange market in order to purchase foreign made goods than
foreigners are demanding for the purchase of American goods
* deficit; supplying less dollars to the exchange market in order to purchase foreign made goods than
foreigners are demanding for the purchase of American goods

That answer is incorrect.


Correct answer:
deficit; supplying more dollars to the exchange market in order to purchase foreign made goods than
foreigners are demanding for the purchase of American goods

A U.S. merchandise trade deficit occurs when the supply of U.S. dollars to the exchange market is greater
than the amount of dollars demanded by foreigners to purchase American goods.

------------------------------

Economically speaking, tariffs are

* obstacles that limit voluntary exchange.


* equivalent to import quotas in their effects on economic efficiency.
* necessary to keep the basic industries of an economy healthy.
* a means to promote economy efficiency.

That answer is correct!

A tariff is a trade-restricting device because it limits the ability of foreigners to compete for a share of a
domestic market. The consumer will purchase the lowest price good (assuming equivalence across all
goods), A tariff that increases the price of an imported good artificially thus restricts a voluntary exchange
between the consumer and producer.

------------------------------
The ________ market is an electronically linked network of banks, foreign exchange brokers and dealers
who unite buyers and sellers of foreign exchange.

* foreign exchange
* stock
* financial
* options
* interbank

That answer is correct!

The purpose of the foreign exchange market is to trade one currency for another currency, i.e., it permits
people to exchange purchasing power that is denominated in different currencies.

------------------------------

If real interest rates in the U.S. are higher than those of our trading partners, what will tend to happen to
the foreign exchange value of the dollar and the U.S. current account deficit or surplus?

Dollar will: Current account will:

I. depreciate move toward a deficit


II. depreciate move toward a surplus
III. appreciate move toward a deficit
IV. appreciate move toward a surplus

* II.
* IV.
* I.
* III.

That answer is incorrect.


Correct answer:
III.

Higher U.S. real interest rates will cause the demand for U.S. dollars to increase as investors try to
purchase the higher yield assets in the U.S. As a result the U.S. dollar will cause the dollar to appreciate.
An appreciating dollar will decrease the demand for U.S. exports and thus the current account will move
toward a deficit.

------------------------------

Domestic producers are currently producing an equilibrium quantity of 1,000 units of a good on which
there are no import restrictions. If the government were to now impose a 5% tariff on imports, the domestic
output will ________ and the domestic consumption will ________.

* increase, also increase


* decrease, increase
* decrease, also decrease
* increase, decrease
That answer is incorrect.
Correct answer:
increase, decrease

The higher price will lead to a higher output from suppliers but a lower demand from consumers.

------------------------------

If the exchange-rate of the English pound goes from $1.75 to $1.50, then the pound has

* depreciated and the English will find U.S. goods cheaper.


* depreciated and the English will find U.S. goods more expensive.
* appreciated and the English will find U.S. goods cheaper.
* appreciated and the English will find U.S. goods more expensive.

That answer is incorrect.


Correct answer:
depreciated and the English will find U.S. goods more expensive.

Prior to the depreciation of the English pound Americans had to spend $1.75 to purchase one pound.
Afterwards, Americans had to only spend $1.50 to purchase one pound. Americans are now required to
pay less for English pounds; conversely, the English must now spend more to purchase one U.S. dollar (in
terms of pounds). The buying power of the dollar has increased relative to the pound which implies that
the English currency has depreciated and the English will find U.S. goods more expensive.

------------------------------

A country must either ________ if its currency is going to be fully convertible with other currencies.

* suffer a capital account or current account surplus


* suffer a capital account or current account deficit
* give up its monetary independence or fix its exchange rate
* maintain its monetary independence or allow its exchange rate to fluctuate
* give up its monetary independence or allow its exchange rate to fluctuate

That answer is incorrect.


Correct answer:
give up its monetary independence or allow its exchange rate to fluctuate

country can either follow an independent monetary policy and allow its exchange rate to fluctuate or tie its
monetary policy to the maintenance of the fixed exchange rate.

------------------------------

If a nation maintained a system of fixed exchange rates, a devaluation of its currency would be expected
to cause
* travel abroad to increase.
* an increase in the nation's investments abroad.
* an increase in exports.
* an increase in imports.

That answer is incorrect.


Correct answer:
an increase in exports.

A fixed exchange rate devaluation implies that the purchasing power of foreign currencies relative to the
fixed currency increased. As a result the demand for the nation's exports would increase because they are
relatively less expensive for foreigners now. The level of the nation's exports would increase.

------------------------------

Which of the following is the most likely impact of an exchange rate depreciation on a nation's current
account deficit? The depreciation will cause the current account deficit to

* widen in both the short run and long run.


* shrink in both the short run and long run.
* widen in the short run but shrink in the long run.
* shrink in the short run but widen in the long run.

That answer is incorrect.


Correct answer:
widen in the short run but shrink in the long run.

According to the J-Curve effect, a nation's current account deficit widens initially before it shrinks in
response to an exchange rate depreciation.

------------------------------

If U.S. lumber producers were prohibited from exporting their product, which of the following would most
likely occur?

* U.S. consumers would gain at the expense of foreign producers.


* Lumber prices in the U.S. would fall, but U.S. households would have less foreign exchange with which
to buy goods, services and assets abroad.
* The incomes of U.S. lumber producers would rise.
* Lumber prices in the U.S. would rise.

That answer is incorrect.


Correct answer:
Lumber prices in the U.S. would fall, but U.S. households would have less foreign exchange with which to
buy goods, services and assets abroad.

An export restriction would imply that the domestic producers would have to sell all of their output
domestically. This would force them to lower their prices. However, since the exporters will not have any
foreign currency since they have not sold to foreign countries, U.S. individuals will not have the necessary
currency to buy imports.
------------------------------

International trade can benefit the domestic economy because:

* markets consumers can purchase a wide diversity of goods at economic prices.


* competition is promoted in domestic.
* their market will expand and will allow them economies of scale in production.
* all of these answers.
* none of these answers.

That answer is incorrect.


Correct answer:
all of these answers.

Competition from abroad helps keep domestic producers on their toes. Domestic producers that otherwise
might have few rivals will have to constantly be seeking ways to improve quality and keep costs low.
Simultaneously, the diversity of goods that is available from abroad provides consumers with a broader
array of choices than would be available in the absence of international trade. An expansion in the size of
the market will permit firms to realize economies that accompany large scale production, marketing and
distribution. Under these circumstances, international trade will allow domestic firms to produce large
outputs and achieve lower costs than would be possible if they were unable to sell abroad.

------------------------------

How much of the world's output is now sold in a country different from the one in which it was produced?

* approximately 20 percent
* approximately 10 percent
* just over 50 percent
* less than 5 percent
* approximately 34 percent

That answer is correct!

Approximately 21 percent of the world's total output is now sold in a country other than that in which it was
produced. This is double the figure of three decades ago.

------------------------------

The wholesale market in which major banks trade currencies with each other is referred to as the
________.

* forward market
* spot market
* Society for Worldwide Interbank Financial Telecommunications
* Clearing House Interbank Payments System
* interbank market
That answer is incorrect.
Correct answer:
interbank market

95% of foreign exchange transactions are channeled through the interbank market.

------------------------------

An appreciation in a currency implies:

* more units of a foreign currency are now required to purchase the currency.
* the purchasing power of the currency has increased.
* all of these answers.
* that foreign goods are now less expensive.
* fewer units of the currency are now required to purchase on unit of a foreign currency.

That answer is incorrect.


Correct answer:
all of these answers.

All of these answers reflect the same idea: an appreciation in a currency implies an increase in the value
of a domestic currency relative to foreign currencies. An appreciation increases the purchasing power of
the domestic currency over foreign goods.

------------------------------

Suppose the dollar falls from 125 to 105 yen. As a result,

* exports to Japan will likely increase.


* Japanese tourists will be less likely to visit the U.S.
* imports from Japan will likely increase.
* U.S. consumers will more likely buy Japanese-made automobiles.
* U.S. businesses will more likely use Japanese shipping lines to transport their products.

That answer is correct!

AS the result of the depreciation in the U.S. dollar (the purchasing power of the dollar has decreased) U.S.
exports are now relatively less expensive. Thus, the exports to Japan will increase. This is seen by
observing that the yen has moved from being able to purchase $1/125 dollars to being able to purchase
$1/105 dollars. The increased purchasing power of the yen makes U.S. exports more attractive to
Japanese consumers.

------------------------------

How will a depreciation in the dollar relative to the German mark influence the cost of BMWs to American
consumers?

* The dollar cost of BMWs will decline.


* The dollar cost of BMWs will stay the same.
* None of these answers.
* The dollar cost of BMWs will increase.

That answer is incorrect.


Correct answer:
The dollar cost of BMWs will increase.

A depreciation in the dollar implies that the purchasing power of the dollar declines; that is, one U.S. dollar
now buys fewer German marks. Since a BMW must be purchased using German marks the exchange
rate is relevant to a potential buyer (even though he or she may pay for the car in dollars, the amount of
dollars needed to pay for the car is dependent on the exchange rate between the dollar and the mark).
Thus, the dollar buys fewer marks and therefore the price of a BMW to an American is now relatively
higher.

------------------------------

Which transaction comprises the largest portion of a nation's balance of payments account?

* none of these answers


* service exports and imports
* merchandise-trade transactions
* income from investments
* unilateral transfers

That answer is incorrect.


Correct answer:
merchandise-trade transactions

The export and import of merchandise goods comprise by far the largest portion of a nation's balance-of-
payments account.

------------------------------

When prices in country A are increasing at a faster rate than the prices in country B, which of the following
hold(s) in country A?

I. The demand curve for B's currency moves left.


II. The demand curve for B's currency moves right.
III. The supply curve for B's currency moves right.
IV. The supply curve for B's currency moves left.

* I only
* II & IV
* IV only
* III only
* II only
* I & III

That answer is incorrect.


Correct answer:
II & IV

When prices are rising in A faster than in B, people in country A will demand more of goods from country
B. This will move the demand curve for B's currency to the right. On the other hand, the amount of A's
goods demanded by people in country B will decline. This will reduce the supply of B's currency in the
foreign exchange market, causing the supply curve of currency B to move to the left. The result of these
two movement will be that the price of currency B will increase relative to A's currency. Therefore, A's
currency will depreciate relative to B's currency.

------------------------------

Which of the following hold(s) in the case of import restrictions?

I. The government generates revenues when it imposes import quotas.

II. Domestic producers always gain with import restrictions.

III. Domestic consumers suffer with import tariffs but not with import quotas.

IV. Foreign producers always gain with import restrictions.

* II & III
* III only
* I only
* II only
* III & IV
* I & III
* IV only

That answer is incorrect.


Correct answer:
II only

Import restrictions serve to raise the prices of imported goods, thus helping domestic producers obtain
higher prices at the expense of domestic consumers. If the restriction is in the form of a tariff, then the
government gains through the extra tax revenues. If the restriction is in the form of a quota, then the
government does not gain or lose but foreign producers receive higher revenues.

------------------------------

________ provide the buying power that makes it possible for a nation to ________ goods.

* Tariffs; import
* Mutual gains; export
* Tariffs; export
* Production possibilities; import
* Exports; import

That answer is incorrect.


Correct answer:
Exports; import

A nation without exports would have no way of generating the foreign currency necessary to purchase
imports. Thus, exports represent the buying power that allows consumers to purchase imports.

------------------------------

If the exchange-rate between the U.S. dollar and the French franc were 0.20 (20 cents = one franc), what
would be the price in dollars of a bottle of French wine selling for 400 francs?

* $80
* $800
* $2,000
* $20

That answer is correct!

Since a French franc costs an American 20 cents or 1/5 of a dollar, the dollar price of a 400 franc bottle of
wine is: $(400/5) or $80.

------------------------------

When the real rate of interest in country A is higher than that in country B, which of the following hold(s)?

I. The supply curve for A's currency moves right.


II. The demand curve for A's currency moves left.
III. The demand curve for A's currency moves right.
IV. The supply curve for A's currency moves left.

* II only
* IV only
* I & II
* I & III
* III & IV
* III only
* II & IV
* I only

That answer is incorrect.


Correct answer:
III & IV

When the real rate of interest in A is higher than in B, investors in country B will move funds into A. This
will move the demand curve for A's currency to the right. On the other hand, investors in A will reduce the
investment in country B. This will reduce the supply of A's currency in the foreign exchange market,
causing the supply curve of currency A to move to the left. The result of these two movement will be that
the price of currency A will increase relative to B's currency. Therefore, A's currency will appreciate
relative to B's currency.

------------------------------
In the above problem, the capital account shows a balance of ________ at the end of the year.

* +546
* -1,191
* -1,482
* +793

That answer is incorrect.


Correct answer:
-1,191

All non-Central Bank related capital investments, including portfolio investments, in a foreign country are
considered part of the Capital account. Note that income on foreign investments is not a part of the Capital
account but of the Current account. Using the concept of net balance = cash inflows - cash outflows, you
can directly calculate the Capital account balance of -793 + 284 - 682 = -1,191.

------------------------------

Which of the following provides the foundation for the case for free trade (no government restriction on
international trade)?

* the law of comparative advantage


* the infant-industry argument
* the anti-dumping argument
* the law of diminishing marginal utility
* the industrial diversity argument

That answer is correct!

The benefits of comparative advantage cannot be realized in the absence of free trade. This is because
trade restrictions distort the relative costs across countries and therefore the law of comparative
advantage is not respected. The benefits of comparative advantage are great and therefore free trade
should be advocated in order to realize these gains.

------------------------------

In response to an exchange rate depreciation, a nation's current account deficit will often initially widen
before it shrinks in the long run. Economists refer to this time path of adjustment as

* the Laffer curve.


* devaluation.
* the balance of trade surplus.
* the J-curve effect.
* the official reserve effect.

That answer is incorrect.


Correct answer:
the J-curve effect.
The J-curve effect (the tendency for a nation's current account deficit to widen initially before it shrinks in
response to an exchange rate depreciation, occurs because the short-run elasticity of domestic demand
for imports and foreign demand for exports is inelastic. In the long run, however, the demand for both
imports and exports is elastic.

------------------------------

Under a pure flexible exchange-rate system, the rate that equates demand and supply in the exchange-
rate market will also lead to a balance of

* current and capital account transactions.


* merchandise exports and merchandise imports.
* current account transactions.
* capital account transactions.

That answer is correct!

At equilibrium the balance of payments accounts must balance (that is, the sum of the current account
balance and capital account balance must be zero). A current account deficit means that in aggregate, the
citizens of a nation are buying more goods and services from foreigners than they are selling to foreigners.
Under a pure flexible exchange system, this excess of expenditures relative to receipts is paid for by
borrowing from and selling assets to foreigners. This leads to a capital account surplus which in the
balance of payments negates the current account deficit. The reverse holds for a current account surplus.

------------------------------

Free trade between two countries will lead to gains for both countries as long as:

* both countries have an absolute advantage in one of the goods.


* there is a difference between the relative costs of production of goods in the two countries.
* neither country has a comparative advantage in either good.
* none of these answers.
* one country does not have an absolute advantage in the production of both goods.

That answer is incorrect.


Correct answer:
there is a difference between the relative costs of production of goods in the two countries.

As long as the relative production costs of goods differ among nations, the nations will be able to gain
from free trade. A difference in the relative costs of production imply that the countries will differ in the
opportunity cost of production of each good and therefore the countries can split production according to
high and low opportunity costs.

------------------------------

Under a system of flexible exchange rates, which of the following will most likely cause a nation's currency
to appreciate on the foreign exchange market?
* stable domestic prices while the nation's trading partners are experiencing inflation and a decrease in
domestic interest rates
* stable domestic prices while the nation's trading partners are experiencing inflation
* an increase in foreign interest rates
* a domestic inflation rate of 10 percent while the nation's trading partners are experiencing stable prices
* a decrease in domestic interest rates

That answer is incorrect.


Correct answer:
stable domestic prices while the nation's trading partners are experiencing inflation

Stable domestic prices in an international environment of rising prices makes the nation's exports
relatively less expensive to foreign consumers. As a result, the demand for the nation's exports increases
as does the demand for the nation's currency. This increase in demand causes the currency to appreciate.

------------------------------

The domestic demand Q for a good A at a price P is given by Q = 500 - 5P while the supply function is
given by 300 + 3P. The world price for good X is 19. If the government places import restrictions on the
good, the revenues of domestic producers will ________.

* decrease by 367
* increase by 2,592
* decrease by 1,192
* increase by 873

That answer is incorrect.


Correct answer:
increase by 2,592

Without restrictions, the producers will be able to produce 300 + 3 * 19 = 357 units and have total
revenues of 357 * 19 = 6,783. If there are import restrictions, then the domestic equilibrium price is given
by the supply = demand condition in the domestic market. This price satisfies 500 - 5P = 300 + 3P, giving
P = 25. Thus, the import restrictions lead to a higher price. At this price, the domestic producers produce
300 + 3 * 25 = 375 units and have total revenues of 375 * 25 = 9,375. Thus, their revenues increase by
9,375 - 6,783 = 2,592.

------------------------------

Consider the situation where the currency of a country has suddenly devalued against the rest of the
world. Which of the following would happen over the longer term?

I. The quantity of exports will increase.


II. The quantity of imports will increase.
III. Domestic inflation will increase.

* III only
* I, II & III
* I & III
* I & II
* I only
* II only

That answer is incorrect.


Correct answer:
I & III

The immediate impact of the devaluation is not a quantity adjustment. In the very short run, imports and
exports stay constant, changing only over slightly longer term. However, given time, the imports will fall
since they are costlier and demand will shift to domestic products. Exports will also rise since domestic
goods are now cheaper to foreigners. This rise in domestically produced goods will lead to higher inflation
and a further devaluation of the currency, unless offsetting market forces come into play, with or without
changes in fiscal and monetary policies.

------------------------------

Whether a nation runs a trade deficit or surplus depends upon:

* the attractiveness of the nation's exports relative to the nation's saving rate.
* the investment opportunities in the country relative to the attractiveness of the nation's exports.
* the investment opportunities in the country relative to the nation's preferences toward imports.
* the investment opportunities in the country relative to the nation's saving rate.
* the attractiveness of the nation's exports relative to the nation's preferences.

That answer is incorrect.


Correct answer:
the investment opportunities in the country relative to the nation's saving rate.

Under a flexible rate exchange system, countries with more attractive investment opportunities and a
lower saving rate than their trading partners will run current account deficits. This is because domestic
investors will not take advantage of all of the domestic opportunities and thus foreign investors will invest
their capital in the domestic economy. This will lead to a capital account surplus. Since the balance of
payments must balance, there must be a trade deficit to counteract this surplus. When a country
possesses less attractive investment opportunities and a higher savings rate, it will run capital account
deficits and trade surpluses. Thus, the investment opportunities of the country relative to its savings rate is
important in determining if there is a trade deficit or surplus.

------------------------------

The following chart indicates the production possibilities of food and clothing per workday in the U.S. and
Japan.

United States Japan

Food 2* 2
Clothing 4 3

*Units of output per workday

Which of the following is true?

* The Japanese are the low-cost producers of both food and clothing.
* Mutual gains from trade could be realized if the U.S. specialized in food production and Japan in
clothing production.
* U.S. workers will not be able to gain from trade with Japan.
* Joint output would be maximized if the U.S. specialized in producing clothing and Japan in producing
food.
* Since Japanese workers produce more of both food and clothing than U.S. workers, no gains from trade
are possible.

That answer is incorrect.


Correct answer:
Joint output would be maximized if the U.S. specialized in producing clothing and Japan in producing food.

The U.S. has a comparative advantage in the production of clothing. This is because the opportunity cost
of U.S. production of one unit of clothing is 1/2 unit of food. The opportunity cost of Japan for one unit of
clothing is 2/3 which is greater than the U.S. opportunity cost. Therefore, the U.S. should produce clothing
and the Japanese should produce food.

------------------------------

I. The U.S. current account deficit is a financial obligation of the federal government and if it is not
reduced, foreigners will be reluctant to loan money to the U.S. government.

II. The current account deficit does not belong to any individual or institution, it is a purely statistical
aggregate, like the number of eggs laid in the U.S. or the number of bald-headed men living in the U.S.

* Both statements are true.


* I is true, II is false.
* Both statements are false.
* I is false, II is true.

That answer is incorrect.


Correct answer:
I is false, II is true.

The U.S. current account deficit is not the financial obligation of the federal government; it is simply a
function of the import and export pattern existent in the market for merchandise, services, income from
investments and transfers. As in II, it is simply a statistical aggregate that is not the liability of any
individual, firm or government.

------------------------------

If French francs are currently selling at $0.1339 and 180-day forward FF are priced at a premium at
$0.1350, what is the forward premium annualized?

* 1.64%
* 1.46%
* -1.63%
* 1.63%
* 0.0164
That answer is correct!

Forward premium annualized = [(0.1350 - 0.1339/0.1339)] X 360/180 = 0.0164 = 1.64%

------------------------------

Which of the following will enter as a credit in the U.S. balance of payments capital account?

* the purchase of a Japanese electronic plant by an American industrialist


* the sale of Japanese electronics to an American
* the sale of an American baseball team to a Japanese industrialist
* the purchase of a Japanese automobile by a U.S. consumer

That answer is incorrect.


Correct answer:
the sale of an American baseball team to a Japanese industrialist

A transaction that decreases the supply of U.S. dollars on the foreign exchange market is recorded as a
credit on the balance of payments. The sale of an American baseball team to a Japanese citizen implies
that the demand for dollars increased. Therefore, this transaction reduced the supply of dollars and is
recorded as a credit.

------------------------------

Expansionary fiscal policy implies:

* the government increases the supply of money in the economy.


* that the government reduces spending.
* the government decreases the supply of money in the economy.
* that the government increases spending.
* none of these answers.

That answer is incorrect.


Correct answer:
that the government increases spending.

Expansionary fiscal policy implies that the government reduces a budget surplus, increases a budget
deficit or balances the budget by spending more money.

------------------------------

A reduction in the tariff on imported steel would most likely benefit

* the domestic producers of steel.


* foreign producers at the expense of domestic consumers.
* workers in the steel industry.
* the domestic consumers of steel.
That answer is incorrect.
Correct answer:
the domestic consumers of steel.

The imposition of a tariff benefits domestic producers and the government at the expense of consumers.
Therefore, the reduction in a tariff benefits the domestic consumers of the good. Consumers lose because
the price of the good rises in the domestic market.

------------------------------

The record of all transactions with foreign nations that involve the exchange of merchandise goods and
services or unilateral gifts is known as a(n) ________ account.

* payment
* exchange
* none of these answers
* current
* service

That answer is incorrect.


Correct answer:
current

All payments (and gifts) related to the purchase or sale of goods and services and income flows during the
designated period are included in the current account. There are four major types of current account
transactions: the exchange of merchandise goods, the exchange of services, income from investments
and unilateral transfers.

------------------------------

When the government imposes import quotas,

I. domestic producers gain.


II. foreign producers lose due to extra taxes.
III. the government gains.
IV. comparative advantage is not fully exploited.

* I, II & III
* IV only
* I, II, III & IV
* I & IV
* II only
* I & III
* I only
* III only

That answer is incorrect.


Correct answer:
I & IV

With quotas, the government gains nothing directly (unlike the case of tariffs, though it could gain or lose
indirectly through tax collections). However, the quotas serve to raise domestic prices and domestic
consumers suffer. The gains are reaped by domestic producers and foreign producers who obtain permits
to sell the goods at higher prices than those prevailing in the world market.

------------------------------

An increase in the tariff on foreign-produced automobiles would most likely harm ________.

* the producers of automobiles


* the domestic consumers of automobiles
* workers in the automobile industry
* steel producers, who supply steel to the domestic automobile industry

That answer is incorrect.


Correct answer:
the domestic consumers of automobiles

A tariff is a tax on foreign imports. Imposing a tariff on foreign produced automobiles involving raising the
price that consumers pay for the automobile. The result will be that domestic consumers will pay more for
an equivalent foreign automobile. Tariffs benefit the domestic producers and the government at the
expense of consumers. Domestic producers do not pay the tariff but benefit from the higher market price.
Thus, the tariff is a subsidy to domestic producers.

------------------------------

The infant-industry argument about tariffs implies that

* permanent tariffs should be levied on foreign products that compete with those produced by newly
established domestic industries.
* tariffs should be levied on foreign products that compete with new domestic industries only in the short
run.
* it is unfair to levy tariffs on items intended for use by infants.
* if a newly established domestic industry can survive in the short run, a tariff should be levied to protect it
from long run foreign competition.

That answer is incorrect.


Correct answer:
tariffs should be levied on foreign products that compete with new domestic industries only in the short
run.

Advocates of the infant-industry argument hold that new domestic industries should be protected from
older, established foreign competitors. As the new industry matures, it will be able to stand on its own feet
and compete effectively with foreign producers, at which time protection can be removed.

------------------------------

I. Nations export goods so they can import goods, services and ownership rights from other nations.

II. Policies that restrict imports simultaneously reduce the ability of foreigners to buy a nation's export
products.

* I is true, II is false.
* I is false, II is true.
* Both statements are true.
* Both statements are false.

That answer is incorrect.


Correct answer:
Both statements are true.

Exports such as goods, services and assets provide the buying power that makes it possible for a nation
to import other goods. Without exports, a nation would not have the foreign currency that is required for
the purchase of imports. Thus, policies which restrict imports reduce the ability of foreigners to buy a
nation's exports because it will not have the foreign currency necessary to do so.

------------------------------

If a U.S. dollar exchanges for 2.5 German marks, then the dollar price of a mark is

* 40 cents.
* 250 cents.
* 25,000 cents.
* 400 cents.
* 2,500 cents.

That answer is correct!

The following equality holds: $1 = 2.5 marks. Divide both sides of the equation by 2.5 marks implies that 1
mark = .4 dollars or 40 cents.

------------------------------

An organization composed of most non-Communist countries designed to set the rules for the conduct of
international trade and reduce the barriers to trade among nations is called ________.

* IOM
* IMF
* None of these answers
* NAFTA
* GATT

That answer is incorrect.


Correct answer:
GATT

GATT is an organization composed of 115 countries designed to set the rules for the conduct of
international trade and reduce barriers to trade among nations.
------------------------------

Which of the following is not a form of import restriction?

* voluntary import restraints.


* voluntary export restraints.
* exchange rate controls.
* percentage tariffs.

That answer is correct!

Voluntary Export Restraints are agreements by foreign producers not to exceed specified export limits.
Other forms of import restrictions include forcing the official exchange rate above the market rate for trade
purposes, imposing quotas and tariffs, etc.

------------------------------

If the exchange-rate between the U.S. dollar and the German mark were 0.50 (50 cents = one mark), what
would be the price in dollars of a German automobile that cost 100,000 marks?

* $20,000
* $200,000
* $10,000
* $50,000

That answer is incorrect.


Correct answer:
$50,000

Since the exchange rate between marks and dollars is that each mark costs an American 50 cents the
dollar cost of a 10,000 mark car is $(10,000/2) or $5,000.

------------------------------

A ________, in the interbank market, is the forward differential (the difference between forward and spot
rates), quoted by dealers only, as a discount from, or a premium on, the spot rate.

* premium rate
* all of these answers
* discount rate
* swap rate
* outright rate

That answer is incorrect.


Correct answer:
swap rate

Forward rates can be expressed in two ways: Outright rate is the actual price which is usually quoted to
commercial customers. Swap rate, in the interbank market, is the forward differential (the difference
between forward and spot rates), quoted by dealers only, as a discount from, or a premium on, the spot
rate.

------------------------------

An appreciation in the value of the U.S. dollar would

* make it more expensive for U.S. citizens to travel abroad.


* increase the number of dollars it takes to buy a Swiss franc.
* encourage foreigners to make investments in the U.S.
* make U.S. goods more expensive to foreign consumers.

That answer is incorrect.


Correct answer:
make U.S. goods more expensive to foreign consumers.

An appreciation in the value of a nation's currency means that fewer units of the currency are now
required to purchase one unit of a foreign currency. Or conversely, it now takes more units of foreign
currency to purchase one unit of the appreciated currency. If prices are constant but a currency
appreciates then the good will be more expensive to foreigners because it will take more foreign currency
to pay for the constant-priced good.

------------------------------

Which of the following will most likely result from an unanticipated shift to a more restrictive monetary
policy?

* lower real interest rates and an appreciation in the exchange rate of the nation's currency
* higher real interest rates and an appreciation in the exchange rate of the nation's currency
* lower real interest rates and a depreciation in the exchange rate of the nation's currency
* higher real interest rates and a depreciation in the exchange rate of the nation's currency

That answer is incorrect.


Correct answer:
higher real interest rates and an appreciation in the exchange rate of the nation's currency

Unanticipated restrictive monetary policy will lead to a contraction in growth, a deceleration in inflation and
higher interest rates. As a result the demand for the nation's exports and assets will increase as will the
demand for the currency. This will cause the current to move toward a surplus and dollar will appreciate.

------------------------------

What gives the home currency price of a certain quantity of the foreign currency quoted?

* direct quotation
* cross rate
* indirect quotation
* bid-ask spread
* spot rate
That answer is correct!

For example, the price of foreign currency is expressed in French francs in France and in Deutsche marks
in Germany. Thus, in France, the Deutsche mark might be quoted at FF 4, whereas in Germany, the franc
would be quoted at DM 0.25.

------------------------------

________ utilize forward contracts to rid themselves of exchange risk in the currency markets.

* Traders
* Hedgers
* Speculators
* Arbitrageurs
* All of these answers

That answer is incorrect.


Correct answer:
Arbitrageurs

Traders utilize forward contracts to end (or cover) the chance of loss on export or import orders that are
denominated in foreign currencies. Hedgers utilize forward contracts to safeguard the value (in home
currency terms) of assets denominated in a foreign currency on its balance sheet. Speculators choose to
be exposed to currency risk (unlike the three above who attempt to minimize/eliminate risk) by using
forward contracts to benefit from exchange rate fluctuations.

------------------------------

Under a flexible exchange rate system, if a nation is experiencing a deficit on its current account
transactions, the

* nation must also experience a deficit on its capital account transactions.


* nation must experience an offsetting surplus on its capital account transactions.
* nation's currency must appreciate.
* nation's currency must depreciate.

That answer is incorrect.


Correct answer:
nation must experience an offsetting surplus on its capital account transactions.

Since the balance of payments must balance it must be true that if the current account is in deficit that the
capital account is in surplus or vice versa. This implies that the sum of the capital account and the current
account must equal zero and thus cancel each other out.

------------------------------

Consider the international trade relationship between U.S. and Germany. The demand for German Marks
(DEM) by U.S. consumers reflects the demand for German goods arising from U.S. and vice versa. In this
case, a simple model indicates that when the exchange rate is "S" Deutchemark per U.S. dollar, the
quantity of DEM demanded by U.S. investors is given by Q = 3.1 - 1.17 S (expressed in billions). At the
same time, the quantity of Deutchemark that the Germans are ready to supply equals Q = 0.9 + 0.78 S.
The equilibrium exchange rate that would prevail in a freely operating foreign exchange market equals
________.

* 1.128
* 1.182
* 1.397
* 1.436

That answer is correct!

This example is to illustrate the fact that the foreign exchange market is exactly like any other commodities
market when there are no artificial government restrictions (in principle). The value of one currency in
terms of another is determined by the market forces of supply and demand. In the current example, the
demand gets equated with supply when Q = 3.1 - 1.17S = 0.9 + 0.78S. Solving this gives the equilibrium
exchange rate of S = (3.1-0.9)/(1.17+0.78) = 2.2/1.95 = 1.128 DEM per USD.

------------------------------

If a foreign supplier sells a good in another country at a cheaper price than it sells the good in its home
market, the

* foreign supplier will gain a monopoly in the foreign market.


* receiving country can gain from buying the foreign-produced good if it is cheaper than the cost of
producing the good domestically.
* receiving country will be harmed by the dumping of the good into its domestic market.
* usual implications of the law of comparative advantage do not hold in this case, particularly if the low-
cost supplier is subsidized by a foreign government.

That answer is incorrect.


Correct answer:
receiving country can gain from buying the foreign-produced good if it is cheaper than the cost of
producing the good domestically.

Dumping, which refers to the sale of a good by a foreign supplier in another country at a price lower than
the supplier sells it in its home country, generally benefits domestic consumers and imposes costs on
domestic producers of goods for which the imports are good substitutes. Lower prices permit consumers
to obtain the goods more economically than they are available from domestic producers.

------------------------------

A depreciation in the value of the dollar would

* encourage U.S. consumers to buy more foreign goods.


* reduce the number of dollars that could be purchased with a Mexican peso.
* make U.S. goods more expensive to foreigners.
* discourage U.S. consumers from traveling abroad.
That answer is incorrect.
Correct answer:
discourage U.S. consumers from traveling abroad.

A depreciation in the value of the dollar suggests that the purchasing power of the dollar relative to other
currencies has declined. Therefore foreign goods and services are now more expensive for Americans to
purchase. Americans will be less likely to travel abroad because the goods and services they purchase
there will be relatively more expensive given the depreciation in the dollar.

------------------------------

The interest rate parity relation ensures that ________.

* the forward premium is greater than the interest rate differential between two currencies
* forward exchange rate equals the spot exchange rate
* none of these answers
* the forward premium is smaller than the interest rate differential between two currencies
* arbitrage exists

That answer is incorrect.


Correct answer:
arbitrage exists

The interest rate parity relation derives from the fact that arbitrage exists. If it did not, riskless arbitrage
would occur. The relation states that the forward discount or premium is equal to the interest rate
differential between two currencies.

------------------------------

_______ occasionally intervene in the foreign exchange market to smooth out exchange rate fluctuations
or to maintain target exchange rates

* Major commercial banks


* Commercial customers
* None of these answers- it is illegal
* Foreign exchange brokers
* Central banks

That answer is incorrect.


Correct answer:
Central banks

The major players in the foreign exchange market are: Major commercial banks, Foreign exchange
brokers in the interbank market, Commercial customers, who are primarily multinational corporations, and
Central banks, which occasionally intervene to smooth out exchange rate fluctuations or to maintain target
exchange rates.

------------------------------
The record of all transactions with foreign nations that involve the exchange of merchandise goods and
services or unilateral gifts is called the

* capital account.
* current account.
* balance of payments.
* balance of trade.

That answer is incorrect.


Correct answer:
current account.

All payments related to the purchase or sale of goods and services and income flows during the
designated period are included in the current account. There are four major types of current account
transactions: the exchange of merchandise goods, the exchange of services, income from investments
and unilateral transfers.

------------------------------

A country that fixes the foreign exchange value of its currency above the market rate and limits
convertibility of its currency will

* reduce the ability of its citizens to gain from specialization and exchange.
* increase its ability to specialize.
* improve its standard of living.
* increase its ability to adopt mass-production techniques.

That answer is correct!

This practice ends up fixing the price of foreign currencies below the market level which makes it difficult
for domestic residents to convert their domestic currency to foreign exchange. Thus, domestic residents
find it difficult to acquire foreign currencies and are less able to trade with foreigners.

------------------------------

How will an unanticipated shift to a more restrictive fiscal policy (a shift toward budget surplus) tend to
affect domestic real interest rates and a nation's current account?

Real interest rates will ________ and the current account will ________.

* fall; move toward a deficit


* rise; move toward a surplus
* rise; move toward a deficit
* fall; move toward a surplus

That answer is incorrect.


Correct answer:
fall; move toward a surplus
A shift to a more restrictive fiscal policy retard demand and reduces interest rates. Imports tend to fall
shifting a nation's current account toward a surplus.

------------------------------

Which of these countries would not be considered a leading trading partner of the U.S.?

* France
* Canada
* India
* Italy
* Japan

That answer is incorrect.


Correct answer:
India

India is the only country listed above that both imports less than 10 billion dollars worth of U.S. goods and
exports less than 10 billion dollars worth of goods to the U.S.

------------------------------

Suppose a video craze in the U.S. makes a particular Japanese-produced video game very popular. This
would tend to

* increase the balance of trade surplus of the U.S.


* reduce any existing balance of trade deficit in the U.S.
* reduce any existing balance of trade deficit in the U.S. and increase the balance of trade surplus of the
U.S.
* increase the balance of trade deficit of the U.S.
* affect the U.S. balance of payments but not the balance of trade.

That answer is incorrect.


Correct answer:
increase the balance of trade deficit of the U.S.

Since Americans want to purchase the new video game the level of imports into the U.S. from Japan will
increase. The balance of trade reflects all economic transactions between the U.S. and other countries.
Since an import shows up on the balance of trade as a debit, the increase in Japanese imports will cause
the balance of trade to move closer to or further into deficit.

------------------------------

The major players in the ________ market are:

1. Major commercial banks


2. Foreign exchange brokers in the interbank market
3. Commercial customers, who are primarily multinational corporations
4. Central banks, which occasionally intervene to smooth out exchange rate fluctuations or to maintain
target exchange rates

* currency
* forward
* interbank
* foreign exchange
* spot

That answer is incorrect.


Correct answer:
foreign exchange

The major commercial banks continuously deal in foreign exchange for their own accounts and, therefore,
are market makers.

------------------------------

The argument that trade restrictions expand employment in import-competing industries is

* correct, but it fails to consider that the restrictions also reduce the efficiency of resource use and retard
employment in export industries.
* incorrect; trade restrictions increase the efficiency of resource use, but they do not enlarge employment
in import-competing industries.
* incorrect; trade restrictions generally destroy jobs in import-competing industries.
* correct; therefore, it is correct to conclude that the restrictions also expand aggregate employment.

That answer is correct!

Trade restrictions increase employment in import-competing industries by raising the price in those
industries and thus increasing the domestic supply. Increased production is achieved through increased
employment. However, by ignoring the law of comparative advantage which requires free trade to hold, a
country imposing restrictions is losing out on the potential benefits associated with importing those goods
which the country is a high opportunity cost producer and exporting those goods for which it is a low
opportunity cost producer.

------------------------------

If the exchange-rate of the English pound goes from $1.80 to $1.60, then the pound has

* depreciated and the English will find U.S. goods more expensive.
* appreciated and the English will find U.S. goods cheaper.
* appreciated and the English will find U.S. goods more expensive.
* depreciated, but this will have no effect on the cost of U.S. goods to the English because the U.S. goods
are priced in dollars.
* depreciated and the English will find U.S. goods cheaper.

That answer is correct!

Originally an English pound could be purchased with $1.80 U.S. dollars. After the depreciation a pound
could be purchased for $1.60. This implies that Americans have to spend less to buy a single English
pound. This is one indication that the English pound has depreciated. A second way to verify this result is
to convert the dollar amount into the price of American dollars in pounds. Thus, the exchange rate implies
that originally $1.80=1 pound or $1 = .5556 pounds. After the depreciation $1=.625 pounds. Thus, the
purchasing power of the U.S. dollar has increased which implies that the purchasing power of the pound
has decreased. This in turn suggests a depreciation in the English pound. As a result, the diminished
purchasing power of the pound makes U.S. goods more expensive to the English.

------------------------------

What is the approximate share of GDP in the U.S. that results from international trade?

* 11 percent
* 25 percent
* 20 percent
* 5 percent

That answer is correct!

Exports as a percent of the total output in the U.S. is 11 percent. The U.S. is considered a large country
according to the size of its trade sector.

------------------------------

Which foreign exchange participant seeks to earn risk-free profits by taking advantage of differences in
interest rates among countries?

* traders
* hedgers
* speculators
* brokers
* arbitrageurs

That answer is incorrect.


Correct answer:
arbitrageurs

Arbitrageurs use forward contracts to eliminate the exchange risk involved in transferring their funds from
one nation to another.

------------------------------

The following chart indicates the production possibilities of food and clothing per workday in the U.S. and
Japan.

United States Japan

Food 4* 2
Clothing 2 3
*Units of output per workday

Which of the following is true?

* Joint output would be maximized if the U.S. specialized in producing clothing and Japan in producing
food.
* Since Japanese workers produce more of both food and clothing than U.S. workers, no gains from trade
are possible.
* Mutual gains from trade could be realized if the U.S. specialized in food production and Japan in
clothing production.
* The Japanese are the low-cost producers of both food and clothing.
* Japanese workers will find their welfare reduced if trade with the U.S. is allowed.

That answer is incorrect.


Correct answer:
Mutual gains from trade could be realized if the U.S. specialized in food production and Japan in clothing
production.

The opportunity cost for the U.S. to make one unit of food is 1/2 unit of clothing; the same opportunity cost
for Japan is 1.5 units of clothing. Therefore the U.S. is the low opportunity cost producer of food and
therefore has a comparative advantage in its production. The Japanese therefore have a comparative
advantage in the production of food.

------------------------------

If a nation wants to maintain a constant exchange rate at a time when supply and demand are causing the
exchange rate of its currency to appreciate, the nation might

* shift to a more expansionary monetary policy.


* decrease its tariffs and/or eliminate restrictive quotas.
* shift to a more expansionary monetary policy and decrease its tariffs and/or eliminate restrictive quotas.
* shift to a more restrictive monetary policy.
* reduce taxes and run a budget deficit to push domestic interest rates up.

That answer is incorrect.


Correct answer:
shift to a more expansionary monetary policy and decrease its tariffs and/or eliminate restrictive quotas.

Expansionary monetary policy will cause the nation's inflation to increase and interest rates to fall. This in
turn will reduce the demand for the nation's exports and assets which will reduce the demand for the
nation's currency. The fall in demand will cause the currency to depreciate. By lifting tariffs and restrictive
quotas more imports will be demanded which will drive down the exchange rate value of the currency
even further.

------------------------------

How much profit will a trader make in the following triangular currency arbitrage transaction? Begin with
$100 U.S. and buy marks at $0.6010/DM. Sell the DM for DM1 = 3.5 French francs. Then resell the francs
in the U.S. at FF 5.2/$.

* $7.89
* $6.47
* $10.45
* $12.45
* $11.99

That answer is incorrect.


Correct answer:
$11.99

The trader first converts $ into marks: $100/0.6010 = 166.3894 DM. Then convert the DM to francs:
166.3894 x 3.5 = 582.3627 FF. Then convert the francs into dollars: 582.3627/5.2 = $111.99. Subtract the
original $100, for a profit of $11.99.

------------------------------

If the exchange-rate between the U.S. dollar and the West German mark were 0.60 (60 cents = one
mark), what would be the price in dollars of a German automobile that costs 50,000 marks?

* $80,000
* $30,000
* $5,000
* $8,333
* $300,000

That answer is incorrect.


Correct answer:
$30,000

Since the exchange-rate between the U.S. dollar and the West German mark were 0.60 (60 cents = one
mark), to determine the price of a German car in dollar terms: .6 dollars = 1 mark which implies that one
dollar = 1.666 marks. Therefore the German car costs $(50,000/1.666) or $30,012.

------------------------------

If the U.S. imports low-cost goods produced in low-wage countries instead of producing the goods
domestically,

* dollars that leave the U.S. will not return to buy goods produced by high-wage American workers.
* the availability consumption of goods in the U.S. will be reduced.
* the U.S. will lose jobs.
* the U.S. will gain and domestic resources will be employed more productively.

That answer is incorrect.


Correct answer:
the U.S. will gain and domestic resources will be employed more productively.

A high wage country like the U.S. gains from the opportunity to specialize in the production of capital
intensive goods. This is where the comparative advantage of the U.S. lies. The U.S. will import labor
intensive goods since their labor is expensive and the opportunity cost of producing these goods is high.
Low wage countries are better at producing such goods.
------------------------------

Which of the following is not a function of foreign exchange brokers?

* Ensure adequate supplies of foreign currencies


* Assist banks in minimizing their contacts with other traders
* Provide rates to banks that buy and sell currencies
* Provide anonymity to parties until a rate is agreed upon
* Working to make the foreign exchange market run more smoothly

That answer is correct!

Foreign exchange brokers are intermediaries, smoothing the transaction process for customers, generally
large banks.

------------------------------

State the formula for calculating a forward discount annualized.

* Forward discount = [(Forward rate - Spot rate) / Spot rate] X 360/Forward contract no. of days
* Forward discount = [(Spot rate - Forward rate) / Spot rate] X 360/Forward contract no. of days
* Forward discount = [(Forward rate - Spot rate) / Spot rate]
* Forward discount = [(Forward rate - Spot rate) / Forward rate]
* Forward discount = [(Forward rate - Spot rate) / Forward rate] X 360/Forward contract no. of days

That answer is correct!

For example, if French francs are currently selling at $0.1350 and 180-day forward FF are priced at a
discount of $0.1339, the forward discount annualized would equal: Forward discount annualized =
[(0.1339 - 0.1350/0.1350)] X 360/180 = -0.0163 = -1.63%

------------------------------

The difference between the value of a country's merchandise exports and merchandise imports is known
as the

* balance of payments.
* official reserve account.
* balance on capital account.
* balance on current account.
* balance of merchandise trade.

That answer is incorrect.


Correct answer:
balance of merchandise trade.

The balance of merchandise trade is the difference between the value of a country's merchandise exports
and the value of its merchandise imports.

------------------------------

A basic flaw in the infant-industry argument is that

* most industries need protection when they are mature, not when they are first established.
* once a tariff is granted, political pressure will likely prevent the withdrawal of the tariff even when the
industry matures.
* the amount of the tariff is unlikely to have much impact on the success of an infant industry.
* domestic consumers will continue to buy the foreign products anyway, regardless of the tariff.

That answer is incorrect.


Correct answer:
once a tariff is granted, political pressure will likely prevent the withdrawal of the tariff even when the
industry matures.

The infant industry argument suggests that protection be temporary until the infant industry is mature
enough to compete effectively with foreign competitors. However it is generally difficult to remove such
protection once it is applied due to political considerations.

------------------------------

Which one of the following would supply dollars to the foreign exchange market?

* the purchase of Japanese televisions by an American distributor


* the sale of U.S. automobiles to a Mexican consumer
* the spending of British tourists in the U.S.
* the sale of a U.S. corporation to a Saudi Arabian investor
* the purchase of 1,000 shares of IBM stock by a Latin American investor

That answer is correct!

Since the American buys Japanese televisions with Japanese yen he or she must exchange U.S. dollars
for the yen on the foreign exchange market. As a result there is an increase in the supply of U.S. dollars
on the market.

------------------------------

I. If a nation's imports exceeded its exports, a depreciation in the nation's currency on the foreign
exchange market would restore equilibrium if the nation's demand for foreign goods (and the demand of
foreigners for the nation's goods) were inelastic.

II. Since the demand of foreigners for a nation's products (and the nation's demand for imports) may be
inelastic in the short run, a depreciation in the nation's currency may not immediately restore equilibrium in
the foreign exchange market.

* Both statements are true.


* I is false, II is true.
* I is true, II is false.
* Both statements are false.

That answer is incorrect.


Correct answer:
I is false, II is true.

Since consumer demand is inelastic, the depreciation of the nation's currency would not affect their
spending patterns and they would continue to consume the same amount of imports. However, it is true
that if foreign demand is inelastic in the short run then they may not increase their demand for the nation's
exports once the nation's currency depreciates.

------------------------------

What would the bid-ask spread be for pound sterling quoted at $14419-28?

* 0.63%
* 6.4%
* 6.2%
* 0.64%
* 0.062%

That answer is incorrect.


Correct answer:
0.062%

Percent spread = [(1.4428 - 1.4419)/1.4428] x 100 = 0.062%

------------------------------

If the exchange-rate of the dollar goes from .50 English pounds to .75 pounds, then the dollar has

* depreciated relative to the pound and English goods have become more expensive to U.S. consumers.
* appreciated relative to the pound and English goods have become more expensive to U.S. consumers.
* appreciated relative to the pound and English goods have become less expensive to U.S. consumers.
* depreciated relative to the pound and English goods have become less expensive to U.S. consumers.

That answer is incorrect.


Correct answer:
appreciated relative to the pound and English goods have become less expensive to U.S. consumers.

Prior to the appreciation Americans could purchase .50 pounds for $1. After the appreciation they were
able to purchase .75 pounds for $1. Thus, the purchasing power of the dollar relative to the pound
increased and this is known as a currency appreciation. As a result of this appreciation English goods
become relatively less expensive for Americans; it takes fewer dollars to purchase a given English item.

------------------------------
Under a system of flexible exchange-rates, an increase in the foreign demand for the U.S. dollar in the
foreign exchange market will cause the

* U.S. trade deficit to decrease.


* dollar to appreciate.
* dollar to depreciate.
* U.S. inflation rate to increase.

That answer is incorrect.


Correct answer:
dollar to appreciate.

Increased demand will cause the U.S. dollar to appreciate. This is because the exchange rate is
essentially the price of a U.S. dollar in terms of a foreign currency. When the demand for U.S. dollars
increase the price of the dollar also increases. When the price of the dollar increases it is said to
appreciate.

------------------------------

The domestic demand Q for a good A at a price P is given by Q = 800 - 6P while the supply function is
given by 400 + 4P. The world price for good X is 44. If the government places export restrictions on the
good, the revenues of domestic producers will:

* decrease by 3,662.
* decrease by 2,944.
* increase by 2,199.
* increase by 1,215.

That answer is incorrect.


Correct answer:
decrease by 2,944.

Without restrictions, the producers will produce 400 + 4 * 44 = 576 units and have total revenues of 576 *
44 = 25,344. If there are export restrictions, then the domestic equilibrium price is given by the supply =
demand condition in the domestic market. This price satisfies 800 - 6P = 400 + 4P, giving P = 40. Thus,
the export restrictions lead to a lower price. At this price, the domestic producers produce 400 + 4 * 40 =
560 units and have total revenues of 560 * 40 = 22,400. Thus, their revenues decrease by 25,344 - 22,400
= 2,944. Note that we have implicitly used the fact that the domestic producers do not have to pay the
tariff and pocket the entire higher price.

------------------------------

According to international trade theory, a country can gain if it

* imports goods when they can be purchased at a lower price from foreign producers than from domestic
producers.
* exports goods when foreigners are willing to pay higher prices than domestic consumers.
* all of these answers.
* specializes in producing those things it does best (produces at a low cost).
That answer is incorrect.
Correct answer:
specializes in producing those things it does best (produces at a low cost).

A country gains from trade when it is able to import goods that can be produced domestically only a high
opportunity cost. By avoiding the high opportunity cost of production the country can divert resources
toward the production of low opportunity cost goods. This benefits the country and the total economy

------------------------------

Suppose the government institutes an unexpected restrictive fiscal policy. In the short run,

I. inflation will increase.


II. the domestic currency will depreciate.
III. real interest rates will increase.

* III only
* I only
* II only
* II & III
* I, II & III
* I & II
* III only

That answer is incorrect.


Correct answer:
II only

When the government institutes a restrictive fiscal policy, the aggregate demand decreases and the
inflation rate decreases. Savings increase and the supply of loanable funds rises. Further, since the
government is not as active as a borrower, the demand for loanable funds decreases. This leads to a
reduction in the real rates of interest. Consequently, foreign investment decreases, leading to a decreased
demand for the domestic currency. The domestic currency therefore depreciates.

If you look at the situation more closely, you will see two conflicting effects. Since aggregate demand
decreases, imports will also tend to decrease and exports will tend to rise. This will place an upward
pressure on the domestic currency and make it appreciate. however, the decreased foreign investment
referred to above will cause a depreciation. Since capital movements are much faster than changes in
goods movements, the latter effect dominates and the currency depreciates. At the same time, imports
decrease and exports rise.

------------------------------

The currency of a nation that has a low real interest rate prevailing in the economy ________ relative to
currency of a nation that has a high real interest rate prevailing, all else equal.

* will depreciate
* will appreciate
* will not be affected
* may appreciate or depreciate
That answer is correct!

Capital funds flow from regions of low real rates of interest to regions of high real rates of interest, all else
equal. Hence, the demand for the low interest rate currency will decrease, causing it to depreciate. It
should be noted that the interest rates referred to here are risk-adjusted rates.

------------------------------

The exchange rate of a country's currency will ________ if the income of the country and its trading
partners declines.

* increase
* none of these answers
* This answer depends on which income declines the fastest: for countries that are similar in size and
propensity to import, the country that is contracting the fastest will decrease its demand for imports
relatively more than its trading partner, resulting in an increase in the value of the more rapidly growing
nation's currency.
* decrease

That answer is incorrect.


Correct answer:
This answer depends on which income declines the fastest: for countries that are similar in size and
propensity to import, the country that is contracting the fastest will decrease its demand for imports
relatively more than its trading partner, resulting in an increase in the value of the more rapidly growing
nation's currency.

For countries similar in size and propensity to import, the country that is contracting the fastest will
decrease its demand for imports relatively more than its trading partner, resulting in an increase in the
value of the more rapidly growing nation's currency. Sluggish growth of income relative to one's trading
partners tends to cause the slow-growth nation's currency to appreciate.

------------------------------

Those who advocate fixed, rather than flexible, exchange rates base their arguments primarily on the view
that fixed rates

* eliminate uncertainty about future prices of international currencies.


* automatically create an equilibrium price for each nation's currency in the foreign exchange market.
* will generally lead to a balance of payments deficit.
* cause uncertainty about the prices of goods traded internationally.

That answer is correct!

A fixed exchange rate system implies that governments intervene in the foreign exchange market or alter
their economic policies in an effort to maintain the fixed value of their currency. In this way, the
government can completely determine the exchange rate between the nation and all others. Therefore, all
uncertainty has been removed.

------------------------------
A nation experiencing an excess of imports over exports can remedy this situation under a fixed rate
system by:

* upwardly revaluing its currency.


* none of these answers.
* following expansionary policy.
* all of these answers.
* heightening its trade barriers.

That answer is incorrect.


Correct answer:
heightening its trade barriers.

A country has three choices to remedy the situation of excess imports over exports: a devaluation can
restore equilibrium between the demand and supply of the currency in the exchange market. Enacting
trade barriers in the form of tariffs and quotas reduces imports but is in conflict with economic efficient.
Restrictive macroeconomic policy retards inflation and increases interest rates.

------------------------------

________ are recorded as a ________ item on the balance of payments account.

* Imports, credit
* None of these answers
* Exports, debit
* Imports, debit

That answer is incorrect.


Correct answer:
Imports, debit

Balance of payments accounts record any transaction that supplies the nation's domestic currency (or
creates a demand for foreign currency) in the foreign exchange market is recorded as a debit, or minus,
item. Since imports create a supply of domestic currency on the exchange market it is recorded as a debit
on the balance of payments account.

------------------------------

Devaluation of a nation's currency

* will tend to reduce the exports of the devaluating country.


* will tend to increase imports of the devaluating country.
* will tend to reduce the imports of the devaluating country.
* can only occur under a system of flexible exchange rates.

That answer is incorrect.


Correct answer:
will tend to reduce the imports of the devaluating country.
Devaluation occurs under a fixed exchange system and implies that the purchasing power of the nation's
currency is reduced relative to other currencies. Imports are now more expensive and therefore the
demand for imports decreases.

------------------------------

How will an unanticipated shift to a more restrictive monetary policy tend to influence the foreign exchange
value of the dollar and the U.S. current account deficit or surplus?

The dollar will: The current account will:

I. depreciate move toward a deficit


II. depreciate move toward a surplus
III. appreciate move toward a deficit
IV. appreciate move toward a surplus

* II.
* IV
* I.
* III.

That answer is incorrect.


Correct answer:
IV

Unanticipated restrictive monetary policy will lead to a contraction in growth, a deceleration in inflation and
higher interest rates. As a result the demand for the nation's exports and assets will increase as will the
demand for the currency. This will cause the current to move toward a surplus and dollar will appreciate.

------------------------------

The most likely consequence of trying to maintain monetary independence while fixing the exchange rate
value of a currency is:

* an above market fixed exchange rate value of the currency.


* a below market fixed exchange rate value of the currency.
* low inflation.
* none of these answers.
* high inflation.

That answer is correct!

Several countries have tried to maintain monetary independence while tying their exchange rate to
another country. This strategy generally results in the exchange rate value of the country's currency being
fixed above the market level.

------------------------------

The primary source of purchasing power used to import goods is


* the balance of payments deficit.
* the exports of a nation.
* taxation and other revenue-generating activities.
* the monetary sector.

That answer is incorrect.


Correct answer:
the exports of a nation.

Exports such as goods, services and assets provide the buying power that makes it possible for a nation
to import other goods. Without exports, a nation would not have the foreign currency that is required for
the purchase of imports.

------------------------------

Fixing the price of a currency in the exchange market:

* equilibrates the market in terms of supply and demand.


* results in surpluses and shortages of currency.
* implies adjustment of the exchange rate daily.
* none of these answers.
* is referred to as floating exchange rates.

That answer is incorrect.


Correct answer:
results in surpluses and shortages of currency.

Fixing the price of a currency in the exchange market implies that as market conditions change there is no
adjustment to alter the exchange rate to equal supply with demand. Thus, shortages and surpluses
develop.

------------------------------

When interest parity ensures that the return on a hedged foreign investment will equal the domestic
interest rate on investments of identical risk, the covered interest differential equals ________.

* cannot be computed with the information given


* -1
* non-zero
*1
*0

That answer is incorrect.


Correct answer:
0

As an example, suppose an investor is deciding whether to invest $1 million in U.S. dollars at an annual
rate of 12% (3% for 90 days) or invest $1 million in pounds sterling (ps) at an annual rate of 8% (2% for 90
days). Assume the spot rate is ps 0.6812/$ and the 90-day forward rate is ps 0.6746/$. The answer for the
investor is that the returns will be identical. The $1 million invested in dollars for 90 days will yield
$1,000,000 x 1.03 = $1,030,000.

------------------------------

In the short run, it is likely that a 10 percent depreciation in the U.S. dollar will lead to:

* a less than 10 percent increase in export demand.


* a greater than 10 percent increase in export demand.
* a less than 10 percent decrease in export demand.
* a greater than 10 percent decrease in export demand.

That answer is correct!

Since American and foreign produced goods are excellent substitutes for one another, there is good
reason to expect that both the U.S. demand for imports and foreign demand for U.S. exports will be highly
elastic in the long run. In the short run however, it will take time for foreign consumers to adjust their
consumption to now cheaper U.S. products.

------------------------------

Given the following:

I. The law of comparative advantage suggests that, if two people shared the lawn-mowing and gardening
chores, the lawn should be mowed by the person who could do it quickest.

II. If a lawyer was also the fastest typist in town, she clearly could not gain by hiring a typist.

* Both statements are false


* Both statements are true
* I is false, II is true
* I is true, II is false

That answer is correct!

The law of comparative advantage suggests that if the there are two people involved in these chores the
person who is relatively best at mowing should do so. This implies that if the first person is worse than the
second at both chores, he will mow if he is relatively less worse at mowing. For the lawyer who is a good
typist: if the lawyer is relatively better than the typist at interpreting the law and representing clients then
the lawyer should not type. It is possible that the lawyer is absolutely better at both types of tasks (law and
typing) than the typist. However, it is likely that the lawyer is better at being a lawyer relative to his or her
typing skills than the typist is. Thus, the typist should type and the lawyer should practice law since the
lawyer is "more efficient" at being a lawyer than at being a typist relative to the typist.

------------------------------

Which of the following provides the foundation of the case for free trade (no government restriction on
international trade)?
* the anti-dumping argument
* the law of comparative advantage
* the infant-industry argument
* the industrial diversity argument

That answer is incorrect.


Correct answer:
the law of comparative advantage

The law of comparative advantage implies that trading partners can be made better off if each specializes
in the production of goods for which it is a low opportunity cost producer and trades for those goods for
which it is a high opportunity cost producer. This situation will lead to a minimization of costs and a
maximization of output. Free trade will allow the opportunity for countries to specialize in production
according to comparative advantage. Under restricted trade (such as tariffs or quotas) resources that
could have been used to produce goods that the country produces efficiently are diverted to the less
efficient production of other goods. Free trade is necessary in order to realize the gains due to
comparative advantage.

------------------------------

A(n) ________ is the actual price which is usually quoted to commercial customers.

* swap rate
* premium rate
* all of these answers
* discount rate
* outright rate

That answer is incorrect.


Correct answer:
outright rate

Forward rates can be expressed in two ways: Outright rate is the actual price which is usually quoted to
commercial customers. Swap rate, in the interbank market, is the forward differential (the difference
between forward and spot rates), quoted by dealers only, as a discount from, or a premium on, the spot
rate.

------------------------------

Which of the following would tend to cause a nation's currency to appreciate?

* lower domestic interest rates


* an increase in a nation's exports
* higher foreign interest rates
* a more rapid domestic inflation rate than those of its trading partners

That answer is incorrect.


Correct answer:
an increase in a nation's exports
An increase in a nation's exports implies that foreign demand for the nation's goods has increased. In
order to purchase the nation's exports, foreigners must use the nation's currency; thus demand for the
currency increases. As the demand rises, the currency appreciates.

------------------------------

Which one of the following is a debit in the U.S. current account?

* the purchase of insurance from Lloyd's of London by an American


* a trip to the U.S. by a Mexican student
* the purchase of a U.S. car by a German
* the purchase of Rockefeller Center by Japanese investors

That answer is correct!

Since the purchase of insurance from Lloyd's by an American increases the supply of U.S. dollars on the
foreign exchange market, this transaction is accounted for as a debit on the U.S. current account.

------------------------------

In a purely floating exchange rate economy, a shift toward a restrictive monetary policy will move the
capital account toward a ________. The current account will move toward a ________.

* surplus; surplus also


* none of these answers
* deficit; surplus
* deficit; deficit also
* surplus; deficit

That answer is incorrect.


Correct answer:
surplus; deficit

A restrictive monetary policy will slow down the economic growth, reduce inflation and increase real
interest rates by reducing the money supply. This increase in real rates will attract foreign funds, causing
the domestic currency to appreciate. The inflow of funds will move the capital account toward a surplus (or
a smaller deficit). The current account will move toward a deficit since the changes in current and capital
accounts in a purely floating economy must equal zero. Over a slightly longer run, the appreciation in the
domestic currency will reduce exports and increase imports, moving the current account into further
deficit.

Note that the slower economic growth tends to reduce imports initially. This may lead you to conclude that
the current account moves toward a surplus and the capital account toward a deficit. This is incorrect
since capital movements are much faster than the speed of changes in demand for goods.

While analyzing questions about current and capital account, always remember that capital account
effects will win out over current account effects due to the high fluidity in the financial markets. Therefore,
always look at the effects on the capital account first.

------------------------------
A devaluation of a nation's currency

* will tend to reduce the imports of the devaluating country.


* will tend to increase exports of the devaluating country.
* will tend to reduce the imports of the devaluating country and will tend to increase exports of the
devaluating country.
* can only occur under a system of flexible exchange rates.
* will tend to reduce the exports of the devaluating country.

That answer is incorrect.


Correct answer:
will tend to reduce the imports of the devaluating country and will tend to increase exports of the
devaluating country.

A devaluation implies that the purchasing power of the nation's currency has been reduced. Therefore, the
imports of the nation will decrease since they are relatively more expensive after the devaluation. The
exports of the nation will increase because the purchasing power of foreign nations' currencies has
increased and thus exports are now relatively less expensive to them.

------------------------------

An unanticipated shift to a more restrictive fiscal policy (a shift toward a budget surplus) will cause

* an increase in GDP growth.


* a capital outflow.
* inflation.
* a shift toward a current account deficit.
* an increase in real interest rates.

That answer is incorrect.


Correct answer:
a capital outflow.

A shift toward a restrictive fiscal policy causes the real interest rate to decline since the government frees
up loanable funds for private use. This will lead to a capital outflow because investors will liquidate their
assets and move them elsewhere for a higher yield.

------------------------------

Currency A is currently selling @ $.2115 and 180-day forward Contract A's are priced at a premium at
.2195. The forward premium annualized would equal ______.

* 3.78%
* 7.29%
* None of these answers
* 3.64%
* 7.56%
That answer is incorrect.
Correct answer:
7.56%

forward premium annualized = (Forward rate - spot rate)/spot rate * (360/180) = 7.56%.

------------------------------

Compared to the no-trade situation, if the U.S. imports shoes, then

* domestic shoe makers will increase both their prices and their profits
* U.S. consumers will be harmed.
* domestic shoe makers will increase their profits.
* the price of domestic shoes will decline.
* domestic shoe makers will increase their prices.

That answer is incorrect.


Correct answer:
the price of domestic shoes will decline.

If the U.S. decides to import shoes it must be because the U.S. is a high opportunity cost producer of
shoes. This implies that the domestic price for shoes will be higher than the international price; once
shoes are imported then, the domestic price will fall since imported goods are subject to the international
price.

------------------------------

A trade policy that restricts the sale of foreign goods in the U.S. market will

* reduce the demand for U.S. export goods, since foreigners will be less able to buy our goods if they
cannot sell to us.
* diminish economic efficiency by allocating more resources to the areas of their relative comparative
disadvantage.
* benefit producers in industries that export goods.
* increase the nation's income since it protects domestic jobs.

That answer is incorrect.


Correct answer:
diminish economic efficiency by allocating more resources to the areas of their relative comparative
disadvantage.

Trade restrictions promote inefficiency because they send the wrong signals to producers when foreigners
can produce the good more cheaply. Under an import restricting policy, domestic producers of this good
will be able to sell more of their output at a higher price (reduced foreign supply will cause the price of the
good to rise domestically).

This encourages domestic production of the good (in which the producers do not have a comparative
advantage). Thus, the consequence of the trade restriction was a reallocation of the nation's resources
toward the production of goods in which it does not have a comparative advantage. If there was a
comparative advantage in the good, the country would not have imported it in the first place.
------------------------------

The last digits of a foreign exchange quotation (i.e. 8.0025 francs per dollar) are called:

* the points.
* the ask.
* the spread.
* the bid.
* the offer.

That answer is correct!

To make quotations faster, only the last digits, called the points, of the foreign exchange price are quoted.

------------------------------

Devaluation differs from depreciation primarily in that

* devaluation can be a depreciation, but a depreciation can never be a devaluation.


* devaluation is an official government act under a system of fixed exchange-rates, whereas depreciation
may occur under either fixed or flexible rates.
* depreciation involves a change in the price of gold in domestic terms.
* depreciation applies to imports, whereas devaluation pertains only to exports.

That answer is incorrect.


Correct answer:
devaluation is an official government act under a system of fixed exchange-rates, whereas depreciation
may occur under either fixed or flexible rates.

Depreciation of a currency refers to a reduction in the value of a domestic currency relative to foreign
currencies. A depreciation reduces the purchasing power of the domestic currency over foreign goods. A
devaluation refers to an official act that changes the level of the fixed exchange rate downward in terms of
other currencies.

------------------------------

Which foreign exchange participant uses forward contracts to eliminate or cover the risk of loss on export
of import orders that are denominated in foreign currencies?

* brokers
* arbitrageurs
* hedgers
* speculators
* traders

That answer is incorrect.


Correct answer:
traders
Generally, a forward-covering transaction is related to a specific payment or receipt expected at a
specified point in time.

------------------------------

How will an unanticipated shift to a more expansionary fiscal policy tend to affect domestic real interest
rates and a nation's current account?

Real interest rates will ________ and the current account will ________.

* rise; move toward a surplus


* fall; move toward a deficit
* rise; move toward a deficit
* fall; move toward a surplus

That answer is incorrect.


Correct answer:
rise; move toward a deficit

A rise in real interest rates causes the capital account to move toward a surplus which necessarily implies
that the current account must move toward a deficit.

------------------------------

For which of the following reasons do producers export goods to other nations?

* They export goods because they seek to increase the standard of living of persons in other countries.
* They export goods because they plan to use the compensation received to buy goods, services and
ownership rights that they will import from others.
* They export goods because they believe in the law of comparative advantage.
* They export goods so they can obtain the currency of other nations, which they will attempt to hold
indefinitely.

That answer is incorrect.


Correct answer:
They export goods because they plan to use the compensation received to buy goods, services and
ownership rights that they will import from others.

Exports such as goods, services and assets provide the buying power that makes it possible for a nation
to import other goods. Without exports, a nation would not have the foreign currency that is required for
the purchase of imports.

------------------------------

Which of the following retard the gains from trade?

* expenditure on physical infrastructure


* exchange rate controls
* GATT
* good weather
* a reduction in tariffs

That answer is incorrect.


Correct answer:
exchange rate controls

Physical obstacles like bad roads and stormy weather that increase transaction costs will retard the gains
from trade. Additionally, tariffs, quotas, exchange rate controls and other man made trade restrictions
have similar effects.

------------------------------

Which of the following would be a debit in the U.S. balance of payments?

* a trip to Japan by an American student


* a short-term loan extended to a South American country by the U.S.
* the purchase of a German car by an American
* the purchase of insurance from Lloyd's of London by a U.S. resident

That answer is incorrect.


Correct answer:
a short-term loan extended to a South American country by the U.S.

Since the extension of the loan increases the supply of U.S. dollars on the foreign exchange market, this
transaction is accounted for as a debit on the U.S. current account.

------------------------------

International trade can benefit domestic firms because:

* they will experience a fall in their labor costs.


* their market will expand and will allow them economies of scale in production.
* they will be subject to a lower interest rate.
* none of these answers.
* they will increase their opportunity costs.

That answer is incorrect.


Correct answer:
their market will expand and will allow them economies of scale in production.

An expansion in the size of the market will permit firms to realize economies that accompany large scale
production, marketing and distribution. Under these circumstances, international trade will allow domestic
firms to produce large outputs and achieve lower costs than would be possible if they were unable to sell
abroad.

------------------------------
When considering the effects of fiscal policy on the exchange rate:

* the effect of a capital inflow or outflow will most likely dominate in the long run.
* the effect of a capital inflow or outflow will most likely dominate in the short run.
* none of these answers.
* the effect is ambiguous in the long and short run.
* the effect of inflation on the level of imports and exports is likely to dominate in the short run.

That answer is incorrect.


Correct answer:
the effect of a capital inflow or outflow will most likely dominate in the short run.

To the extent that a more restrictive fiscal policy places downward pressure on interest rates, the outflow
of capital is likely to dominate in the short run. To the extent that expansionary fiscal policy allows interest
rates to rise, the inflow of capital is likely to dominate in the short run too.

------------------------------

________ choose to be exposed to currency risk (unlike the three above who attempt to
minimize/eliminate risk) by using forward contracts to benefit from exchange rate fluctuations.

* Hedgers
* Traders
* Speculators
* Arbitrageurs
* All of these answers

That answer is incorrect.


Correct answer:
Speculators

Arbitrageurs utilize forward contracts to rid themselves of exchange risk in the currency markets. Traders
utilize forward contracts to end (or cover) the chance of loss on export or import orders that are
denominated in foreign currencies. Hedgers utilize forward contracts to safeguard the value (in home
currency terms) of assets denominated in a foreign currency on its balance sheet.

------------------------------

What is the working balance that is maintained to facilitate delivery and receipt of currencies?

* Clearing House Interbank Payment System


* contra account
* position sheet
* fed funds
* nostro account

That answer is incorrect.


Correct answer:
nostro account
The nostro account is a working balance account that is maintained with the correspondent to facilitate
delivery and receipt of currencies.

------------------------------

When the dollar depreciates, initially the current account deficit will often widen because

* domestic demand for imports and foreign demand for exports is generally inelastic in the short run.
* long-run domestic demand for imports and foreign demand for exports is highly inelastic.
* domestic demand for imports and foreign demand for exports is generally elastic in the short run.
* none of these answers.

That answer is correct!

The J-curve effect (the tendency for a nation's current account deficit to widen initially before it shrinks in
response to an exchange rate depreciation, occurs because the short-run elasticity of domestic demand
for imports and foreign demand for exports is inelastic. In the long run, however, the demand for both
imports and exports is elastic.

------------------------------

Which of the following most accurately characterizes the level of tariffs in the U.S.?

* The average tariff rate has declined substantially since the 1930s and now represents about 4 percent
of the value of duty-eligible imports.
* The average tariff rate declined during World War II, but has increased steadily since then.
* The average tariff has increased substantially since the 1930s and now represents about 30 percent of
the value of duty-eligible imports.
* The average tariff is approximately the same as in the 1930s.

That answer is correct!

After rising slightly in the 1950s, U.S. tariffs have trended downward since 1960. In 1991, the average
tariff rate on merchandise imports as 3.3 percent.

------------------------------

Which of the following is the best example of a tariff?

* a $100-per-car fee imposed on all small cars imported.


* a tax placed on all small cars sold in the domestic market.
* a subsidy from the U.S. government to domestic manufacturers of small cars so they can compete more
effectively with foreign producers of small cars.
* a limit imposed on the number of small cars that can be imported from a foreign country.

That answer is correct!


A tariff is a tax levied on goods imported into a country. Thus, this $100 per car fee represents a tariff
since it is imposed on each unit imported into the country.

------------------------------

The difference between the value of a country's merchandise exports and merchandise imports is known
as the balance

* on current account.
* of merchandise trade.
* of payments.
* on capital account.

That answer is incorrect.


Correct answer:
of merchandise trade.

The balance of merchandise trade refers to the difference between the value of a country's merchandise
exports and the value of its merchandise imports.

------------------------------

Since the ________ is easily deduced by observing two exchange rates with the dollar, all currencies are
traded only against the U.S. dollar.

* none of these answers


* cross exchange rate
* spot exchange rate
* spread
* pure exchange rate

That answer is incorrect.


Correct answer:
cross exchange rate

Since with n currencies only n-1 exchange rates are sufficient to deduce the exchange rates of any
currency pair. All cross-exchange rates can be deduced simply by quoting exchange rates between all
currencies and the U.S. dollar.

------------------------------

The wholesale market in which major banks trade currencies with each other is referred to as the
________.

* Clearing House Interbank Payments System


* Society for Worldwide Interbank Financial Telecommunications
* interbank market
* spot market
* forward market
That answer is incorrect.
Correct answer:
interbank market

95% of foreign exchange transactions are channeled through the interbank market.

------------------------------

When a nation, as the result of its previous experience and/or natural endowments, can produce more
output at the same cost than another nation, it is said to have a(n) ________ advantage.

* comparative
* experience
* absolute
* none of these answers

That answer is incorrect.


Correct answer:
absolute

An absolute advantage implies that one country is more efficient than another in the production of both
goods considered. This may be due to prior experience or higher skill levels in one country.

------------------------------

In response to terrorist activities believed to be sponsored by the Iranian government, the U.S.
government has frozen all Iranian assets in the U.S. In the Iranian BOP account, this event is recorded as
a _______ entry on the _______ account.

* debit; current
* credit; capital
* debit; capital
* credit; current

That answer is incorrect.


Correct answer:
credit; current

Unrequited transfers like foreign government expropriations are part of the Current account.
To determine whether it is a debit or a credit entry, remember that by convention,

1. Any inflow of domestic currency represents a credit and any outflow of domestic currency represents a
debit on the BOP account.

2. Any inflow of foreign currency represents a debit and any outflow of foreign currency represents a credit
on the BOP account.

When the Iranian investors had originally invested in U.S. property, the transaction got recorded as a debit
(to the capital account). Now that this property has been lost to expropriation, there has to be an offsetting
credit entry.

------------------------------

A decrease in the domestic income of a nation's residents will:

* cause exports to rise.


* cause exports to fall.
* discourage residents from spending on imports so that imports will fall.
* none of these answers
* encourage residents to spend income on imports so that imports will fall.

That answer is incorrect.


Correct answer:
discourage residents from spending on imports so that imports will fall.

A decrease in the domestic income will discourage residents from spending on imports since they will
spend less on all types of goods. A fall in the domestic income of a nation will thus reduce imports.

------------------------------

The domestic demand Q for a good X at a price P is given by Q = 400 - 2P while the supply function is
given by 100 + 4P. The world price for good X is 45 in international markets. If the government imposes a
quota of 20 units on imports, the price paid by the domestic consumers equals ________.

* 42.78
* 45.22
* 47.24
* 44.82
* 45.95
* 46.76

That answer is incorrect.


Correct answer:
46.76

At the world price of 45, the producers supply a quantity equal to 100 + 4 * 45 = 280 units and domestic
consumers demand 400 - 2 * 45 = 310 units. Thus, net imports without the quota equal 310 - 280 = 30. If
the government imposes an import quota of 20 units, then the domestic price paid is no longer 45 but a
higher price, M. At this price, the producers produce 100 + 4M units and consumers demand a larger
amount, 400 - 2M units. The difference between these two equals the total imports allowed. Therefore,
(400 - 2M) - (100 + 4M) = 20. Solving for M gives M = 46.76.

------------------------------

Under a purely flexible exchange rate system,

I. the current account deficit is offset by the changes in the Official Reserves account.
II. the official reserves account has a zero balance.
III. the Current and Capital accounts together balance out.
IV. there is no activity in the Official Reserves account.

* II, III & IV


* I, II & IV
* III & IV
* I only
* II only
* III only
* IV only

That answer is incorrect.


Correct answer:
III & IV

At all times, the BOP account must balance out to zero. Under a purely floating exchange rate system, the
official reserves activity is zero (though the reserves are almost certainly non-zero). Hence, under this
system, the Current and Capital accounts together balance out.

------------------------------

The Central Bank of a country has decided not to intervene in foreign exchange markets to affect
exchange rates for the next fiscal year. In this case, which of the following is/are FALSE over the next
fiscal year?

I. The Overall Balance equals zero.


II. The total BOP equals zero.
III. The Official reserves account equals zero.
IV. The current account deficit equals zero.

* II & III
* II only
* IV only
* I, II & III

That answer is incorrect.


Correct answer:
IV only

The BOP account must always balance out. So II is always true. Due to the Central Bank's policy, the
changes in the Official Reserves account will be zero over the next fiscal year, though there's no reason
why the Reserves themselves should be zero (in fact, they most likely won't be zero).
The Overall Balance equals the sum of the Current and Capital account balances. Since the sum of
Current, Capital and the change in Official Reserves account should be zero, in the current example, the
Overall Balance must also be zero.

------------------------------

What type of foreign exchange transaction accounts for approximately 60 percent of the entire market?

* forward transactions
* covered interest arbitrage
* currency arbitrage transactions
* spot transactions
* swap transactions

That answer is incorrect.


Correct answer:
spot transactions

Spot transactions account for about 60% of the market, forward transactions account for 10%, and the
remaining 30% consists of swap transactions.

------------------------------

An official act that changes the level of the "fixed" exchange-rate downward in terms of other currencies is
________.

* reduction
* revaluation
* none of these answers
* devaluation

That answer is incorrect.


Correct answer:
devaluation

Devaluation is a one step reduction in the value of a nation's currency under a fixed rate system.

------------------------------

Under a system of flexible exchange rates, in the long run a nation's balance on current account and
capital account transactions will

* increase continuously.
* decrease continuously.
* tend to net out to zero, indicating a balance between the debit and credits.
* tend to increase if the nation is running a balance of trade surplus and decrease if it is running a balance
of trade deficit.

That answer is incorrect.


Correct answer:
tend to net out to zero, indicating a balance between the debit and credits.

Since the balance of payments must balance it must be true that if the current account is in deficit that the
capital account is in surplus or vice versa. This implies that the sum of the capital account and the current
account must equal zero and thus cancel each other out.

------------------------------
If the French franc is quoted as $0.2051-82 and the German Deutsche mark is quoted as $0.6024-45,
what is the ask cross rate for the French franc?

* FF 2.9371/DM
* FF 2.9473/DM
* FF 2.8821/DM
* FF 2.8934/DM
* FF 2.9035/DM

That answer is incorrect.


Correct answer:
FF 2.9473/DM

The ask cross rate equals the ask rate for the mark divided by the bid rate for the franc, in this case,
0.6045 divided by 0.2051.

------------------------------

I. In 1992 U.S. merchandise exports were greater than merchandise imports.

II. In 1992 U.S. service exports were greater than service imports.

* Both statements are false.


* Both statements are true.
* I is false, II is true.
* I is true, II is false.

That answer is incorrect.


Correct answer:
I is false, II is true.

In 1992 the U.S. exported $440 billion of merchandise goods and imported $536 billion. In 1992 the U.S.
ran a $66 billion service surplus.

------------------------------

Which of the following arguments in support of trade restrictions limiting U.S. dependence on foreign
suppliers of petroleum and aircraft is most valid when applied to the U.S.?

* the industrial diversity argument


* the national defense argument
* the infant-industry argument
* the comparative advantage argument

That answer is incorrect.


Correct answer:
the national defense argument

The national defense argument suggests that certain industries are vital to national defense and therefore
should be protected from foreign competitors so that a domestic supply of necessary materials would be
available in case of an international conflict.

------------------------------

An unanticipated shift to a more expansionary monetary policy will most likely

* reduce imports and decrease real interest rates, causing the nation's currency to appreciate.
* reduce imports and increase real interest rates, causing the nation's currency to appreciate.
* increase imports and increase real interest rates, causing the nation's currency to depreciate.
* increase imports and decrease real interest rates, causing the nation's currency to depreciate.

That answer is incorrect.


Correct answer:
increase imports and decrease real interest rates, causing the nation's currency to depreciate.

When the effects are not fully anticipated, expansionary monetary policy will lead to more rapid economic
growth, an acceleration in the inflation rate and lower real interest rates. Higher inflation and lower real
interest rates will lead to a decline in the demand for the nation's exports and assets which will cause the
demand for the nation's currency to decline. This in turn will cause the currency to depreciate.

------------------------------

Trade permits nations to:

* none of these answers.


* decrease inflation at the expense of increased unemployment.
* expand consumption possibilities and increase total output.
* increase total output.
* extract economic rents from the imposition of trade tariffs and quotas.

That answer is incorrect.


Correct answer:
expand consumption possibilities and increase total output.

Because of the law of comparative advantage, trade between nations will lead to an expansion in total
output and mutual gain for each trading partner when each country specializes in the production of goods
it can produce at a relatively low cost and uses the proceeds to buy goods that it could produce only at a
high cost.

------------------------------

The Mexican peso has a bid price of $0.1093 and an ask price of $0.10943. What is the bid-ask spread?

* 0.13%
* 0.013%
* 0.1189%
* none of these answers
* 0.1187%
That answer is incorrect.
Correct answer:
0.1187%

The bid-ask spread is computed as follows: Percent spread = (Ask price - Bid price)/Ask price X 100, or
.10943 - .1093 X 100 = 0.1187%

------------------------------

I. The concept of comparative advantage applies equally to domestic trade between individuals,
businesses and regions and to foreign trade between nations.

II. The law of comparative advantage suggests that, if television sets could be produced more cheaply in
Japan than in the U.S., the U.S. should subsidize its television manufacturers to minimize unemployment
resources in the domestic television manufacturing industry.

* Both statements are false.


* I is true, II is false.
* Both statements are true.
* I is false, II is true.

That answer is incorrect.


Correct answer:
I is true, II is false.

The law of comparative advantage can be applied to the domestic economy as well as to international
trade. Individuals or firms may enjoy comparative advantage in the production of some goods. This simply
implies that relative to individuals or firms which they can trade goods with they may be more efficient in
the production of one good over another. As long as the relative costs of production between individuals or
firms differs, benefits from comparative advantage can be realized. The law of comparative advantage
requires that free trade exists; thus the imposition of a subsidy by the U.S. would distort free trade and
prevent the benefits of comparative advantage from being realized.

------------------------------

A country such as the U.S., which has a low saving rate and attractive investment opportunities, will tend
to experience a long-term

* outflow of capital and a current account surplus.


* inflow of capital and a current account surplus.
* outflow of capital and a current account deficit.
* inflow of capital and a current account deficit.

That answer is incorrect.


Correct answer:
inflow of capital and a current account deficit.

Since the U.S. has many investment opportunities they will tend to run a capital account surplus; this
surplus, however, must be counteracted by a current account deficit which is often seen in the U.S.
------------------------------

In a managed flexible exchange rate system, which of the following hold(s)?

I. Currencies float freely in the world markets.


II. Central Banks do not intervene in affecting exchange rates.
III. Exchange rates are determined by market forces.

* II only
* III only
* III only
* I & III
* I only
* I, II & III

That answer is incorrect.


Correct answer:
I & III

The flexible exchange rate system allows the currencies to float freely against the world markets. Market
forces determine the value of each currency in terms of other currencies, given the stated and expected
fiscal and monetary policies of each nation. The system becomes managed when Central Banks of
various nations intervene in the markets to increase or decrease the demand for and supply of particular
currencies. Such interventions are usually rare and are much different from the case where governments
set exchange rates by decree (of course, maintaining such decrees in the face of market forces is
extremely hard and is what lead to the collapse of the Bretton Woods agreement).

------------------------------

Which of the following is true?

* If the wage rates of a nation are low, the nation will also have low labor costs per unit of output.
* Restrictions limiting imports will also help domestic producers who export goods.
* A country can reduce unemployment and increase the real income of its citizens by imposing tariffs and
quotas on goods produced by foreign producers.
* When domestic producers have a comparative advantage in producing a good, they can compete
effectively in a competitive world market.

That answer is incorrect.


Correct answer:
When domestic producers have a comparative advantage in producing a good, they can compete
effectively in a competitive world market.

If a nation has a comparative advantage in the production of a good this implies that it can produce the
good at a lower opportunity cost. Other nation's can only produce the good at a higher opportunity cost
and therefore will demand the first nation's output. The first country's comparative advantage good will be
highly demanded because other nation's will prefer to import the good rather than produce it domestically.

------------------------------
When a nation's purely floating currency suddenly appreciates,

I. its current account deficit initially widens due to the price effect.
II. its current account deficit increases in the long run.
III. its capital account deficit increases in the long run.

* I & III
* II & III
* II only
* I only

That answer is incorrect.


Correct answer:
II only

The effect of a currency depreciation on the current-account deficit can be broken up into two parts: Price
effect and quantity effect. When the currency depreciates, the sudden fall in the value of the domestic
currency means that the value of the exports measured in the domestic currency falls while that of the
imports rises. Thus, the current account deficit widens immediately, simply because the currency has
depreciated. Over time, when import and export quantities adjust to this currency revaluation, the actual
amounts of import fall and those of exports rise since the currency depreciation has caused exports to
become cheaper and imports costlier. Eventually, an equilibrium is reached where the current account
deficit shrinks from its pre-depreciation level.

In a purely floating currency regime, changes in current and capital accounts must some up to zero.
Therefore, a reduction in the current account deficit induces an increase in the capital account deficit (or a
reduction in its surplus).

------------------------------

A rate of inflation that is lower than that of one's trading partners will cause a nation's currency to
________.

* appreciate
* cannot be determined
* depreciate
* stay the same

That answer is correct!

Lower relative inflation will increase the demand for the nation's exports. This is because the exports are
now relatively cheaper for the rest of the world. Heightened demand for exports increases the demand for
the nation's currency. As a result, the currency appreciates.

------------------------------

An import quota tariff on French wine that raises the prices for wine will probably hurt

* domestic wineries, which will lose business as a result of the higher prices.
* both domestic wine drinkers and domestic wineries, but this will be more than offset by a reduction in
driving fatalities.
* domestic wine drinkers but help domestic wineries, which will gain from the higher prices.
* both domestic wine drinkers and domestic wine producers because of a reduction in competition.

That answer is incorrect.


Correct answer:
domestic wine drinkers but help domestic wineries, which will gain from the higher prices.

The price for wine will increase domestically under the import tariff. As a result, consumers are worse off
but protected producers are better off because they enjoy higher prices.

------------------------------

The following quotation:

FF/$ = 8.0000-25

implies ________.

* the trader is willing to sell a dollar for 8.0000 francs or to buy a dollar for 8.0025 francs
* the trader is willing to sell a dollar for 8.0000 francs or to buy a dollar for 8.25 francs
* the trader is willing to buy a dollar for 8.0000 francs or to sell a dollar for 8.0025 francs
* the trader is willing to buy a dollar for 8.0000 francs or to sell a dollar for 8.25 francs
* the trader is willing to buy a dollar for 8.0000 francs minus a commission of .0025 francs

That answer is incorrect.


Correct answer:
the trader is willing to buy a dollar for 8.0000 francs or to sell a dollar for 8.0025 francs

This quotation implies that the trader will buy a dollar at 8.0000 francs per dollar or sell a dollar for 8.0025
francs. The difference between the bid and ask price is the spread, which is a form of remuneration for the
dealer.

------------------------------

Suppose trade restrictions that limit the sale of goods between the U.S. and Mexico are removed. As
markets adjust, the removal of the trade restrictions will

* reduce real income in both the U.S. and Mexico.


* increase real income in both the U.S. and Mexico.
* increase real income in Mexico but reduce real income in the U.S.
* increase real income in the U.S. but reduce real income in Mexico.

That answer is incorrect.


Correct answer:
increase real income in both the U.S. and Mexico.

The removal of trade restrictions between any two countries will cause the real income in both places to
increase. This is because under free trade each country can directs its resources toward the areas of their
competitive advantage. This increases the total output of the total economy and raises income in both
places.

------------------------------

I. The demand for dollars on the exchange-rate market originates from the demand of foreigners for U.S.
goods, services and assets.

II. The supply of dollars on the exchange-rate market originates from the demand of Americans for foreign
goods, services and assets.

* Both statements are true.


* I is true, II is false
* I is false, II is true
* Both statements are false.

That answer is correct!

Since U.S. goods must be purchased with U.S. dollars the demand for U.S. dollars is generated by
foreigners wishing to buy U.S. goods, services and assets. Similarly, U.S. citizens must purchase foreign
goods with foreign currency. Therefore, the supply of U.S. dollars to the foreign exchange market is
generated by citizens wishing to purchase foreign goods, services or assets.

------------------------------

Interest rate parity holds when the interest differential is ________ the forward differential.

* all of these answers


* lower than
* higher than
* approximately equal to
* a fraction of

That answer is incorrect.


Correct answer:
approximately equal to

Interest rate parity will hold when there are no covered interest arbitrage opportunities.

------------------------------

In the ________ market, currencies are bought and sold for future delivery.

* spot
* forward
* stock
* interbank
* foreign exchange
That answer is incorrect.
Correct answer:
forward

Currencies are traded in the spot market, where deliveries generally occur within two business days after
the transaction has been concluded.

------------------------------

For which of the following reasons do producers export goods to other nations?

* They export goods because they plan to use the compensation received to buy goods, services and
ownership rights that they will import from others.
* They export goods because trade is often viewed as a form of international war and because nations
with the biggest surpluses will be in a better position economically in the long run.
* They export goods because they believe in the law of comparative advantage.
* They export goods so they can obtain the currency of other nations, which they will attempt to hold
indefinitely.
* They export goods because they seek to increase the standard of living in other countries.

That answer is correct!

Exports such as goods, services and assets provide the buying power that makes it possible for a nation
to import other goods. Without exports, a nation would not have the foreign currency that is required for
the purchase of imports.

------------------------------

Which of the following is true?

* A fixed exchange rate below the market rate will create a foreign currency shortage.
* None of these answers.
* By fixing the exchange rate value of a country's currency below the market rate a country's exports
become more expensive.
* By fixing the exchange rate value of a country's currency above the market rate a country's exports
become more expensive.
* A fixed exchange rate above the market rate will create a foreign currency surplus.

That answer is incorrect.


Correct answer:
By fixing the exchange rate value of a country's currency above the market rate a country's exports
become more expensive.

An artificially high exchange rate will cause a country's export goods to be extremely expensive to
foreigners because they are getting less domestic currency for an equal amount of their foreign currency.
Even if the price of an export is constant, if the exchange increases the real cost of the good increases.

------------------------------
A nation can gain from international trade when:

I. the relative prices of the nation's products differ from those of other countries.

II. it imports goods for which it is a high-opportunity cost producer while exporting goods it produces at low
opportunity cost.

* Both statements are true.


* I is false, II is true.
* Both statements are false.
* I is true, II is false.

That answer is correct!

Both of these statements are true. The law of comparative advantage implies that trading partners can be
made better off if each specializes in the production of goods for which it is a low opportunity cost
producer and trades for those goods for which it is a high opportunity cost producer. This situation will lead
to a minimization of costs and a maximization of output. There must be differences in the relative prices of
products across countries in order for this comparative advantage to be realized. This is because the price
of a good reflects the production costs. If all production costs were equal across countries then there
would be no incentive to trade since each country can produce with equal efficiency.

------------------------------

A foreign currency is ________ if the forward rate expressed in dollars is less than the spot rate.

* at a forward discount
* out-of-the-money
* positively correlated
* at a forward premium
* in-the-money

That answer is correct!

There is a forward premium if the forward rate is above the spot rate.

------------------------------

Relative to a no-trade situation, if the U.S. exported wheat, the U.S. domestic price of wheat

* would rise, but domestic output would fall.


* would rise, as would domestic output.
* would decline, as would domestic output.
* would decline, but domestic output would rise.

That answer is incorrect.


Correct answer:
would rise, as would domestic output.
Since the U.S. is exporting wheat it must be true that it has a comparative advantage in its production. As
a result, the international price must be higher for wheat than the domestic price. As producers export
wheat the price will rise in the U.S. due to the price pressure from abroad. The price domestically will rise
to meet the international price.

------------------------------

Which of the following is false concerning foreign currency bid-ask spreads.

* If a dealer holds an excess supply of a currency, bid-ask spreads are likely to be lower.
* An anticipated change in interest rates is not likely to affect bid-ask spreads.
* If trading volume increases, spreads tend to decrease.
* If a currency is heavily traded, spreads tend to be lower.
* Longer term contracts tend to have higher spreads.

That answer is incorrect.


Correct answer:
An anticipated change in interest rates is not likely to affect bid-ask spreads.

Foreign currency spreads vary according to market conditions, including supply and demand for the
currency, information availability, expected changes in government policy or interest rates, whether the
dealer has excess supply of a currency, trading volume and length of the foreign exchange contract.

------------------------------

The ________ market, which handles approximately 95% of the foreign exchange transactions, is the
wholesale market in which major banks trade with one another.

* options
* stock
* interbank
* foreign exchange
* financial

That answer is incorrect.


Correct answer:
interbank

The foreign exchange market is an electronically linked network of banks, foreign exchange brokers and
dealers who unite buyers and sellers of foreign exchange.

------------------------------

According to the international trade theory, a country can gain

* if it maximizes the employment in domestic industries that face competition from foreign producers who
have lower costs.
* by importing goods when they can be obtained more economically from foreign producers.
* if it protects domestic industries from low-wage foreign producers.
* only if the trade harms its trading partners.

That answer is incorrect.


Correct answer:
by importing goods when they can be obtained more economically from foreign producers.

A country gains from trade when it is able to import goods that can be produced domestically only a high
opportunity cost. By avoiding the high opportunity cost of production the country can divert resources
toward the production of low opportunity cost goods. This benefits the country and the total world
economy.

------------------------------

If the exchange-rate between the U.S. dollar and French franc were 1.50 ($1.50 = one franc), what would
be the price in dollars of a bottle of French wine selling for 200 francs?

* $30
* $13.33
* $75
* $150
* $300

That answer is incorrect.


Correct answer:
$300

Since 1 franc = $1.50 and the bottle of wine sells for 200 francs then the dollar price for this bottle of wine
is $(200x$1.50) or $300.

------------------------------

Most interbank transactions in the U.S. are conducted through ________ who:

1. furnish the rates to various banks who will buy or sell a currency,

2. provide anonymity to the parties until a rate is agreed upon, and

3. assist banks in minimizing their contacts with other traders, thereby making the process more efficient.

* central banks
* commercial customers
* counterparties
* foreign exchange brokers
* major commercial banks

That answer is incorrect.


Correct answer:
foreign exchange brokers

Most small banks and local offices of major banks do not deal directly in the interbank market, but rather
have a credit line with a large bank or with their home office. This is unlike the commercial and central
bank customers that trade in foreign exchange through their banks.

------------------------------

The political popularity of a tariff on imported goods that compete with products of a well-established
domestic industry is

* surprising since one would expect the economic harm resulting from tariffs to be well understood by
voters.
* surprising since one would expect the political power of consumers to override the interests of even a
well-established domestic industry.
* not surprising since such a tariff would generally benefit an easily recognized interest group at the
expense of uninformed, uninterested consumers.

That answer is incorrect.


Correct answer:
not surprising since such a tariff would generally benefit an easily recognized interest group at the
expense of uninformed, uninterested consumers.

Usually well organized special interests favoring trade restrictions have more political clout than those
harmed by the restrictions. As a result, politicians earn more votes by supporting trade restrictions that
benefit organized interest groups than they could gain from the support of consumers and exporters who
tend to be less organized and thus wield less political power.

------------------------------

"Trade restrictions limiting the sale of cheap foreign goods in the U.S. are necessary to protect the
prosperity of Americans." This statement is

* false; American prosperity depends on jobs, not the availability of goods and services.
* true for a reduction in trade restrictions but false regarding an increase in trade restrictions.
* essentially correct.
* false; if a good can be bought cheaper abroad than it can be produced at home, the purchase from
abroad will promote prosperity.

That answer is incorrect.


Correct answer:
false; if a good can be bought cheaper abroad than it can be produced at home, the purchase from
abroad will promote prosperity.

Free trade directs the resources of each nation towards its areas of comparative advantage. American
firms with a comparative advantage in certain goods will export those goods while other goods will be
imported. This scenario will lead to greater prosperity in the U.S. because real income will increase as a
result. Thus, greater trade restrictions will harm the prosperity of the domestic economy.

------------------------------

The argument that import restrictions save jobs and promote prosperity fails to recognize that
* import restrictions cannot create jobs in any industries.
* there are no secondary effects of import restrictions.
* import restrictions will lower prices in the protected industries.
* imports provide foreign countries with the purchasing power to buy exports.

That answer is incorrect.


Correct answer:
imports provide foreign countries with the purchasing power to buy exports.

In order to generate the foreign currency necessary to purchase U.S. exports, foreign countries must be
able to sell goods to the U.S. Without this buying power, U.S. exports will decrease since no one will be
able to purchase them.

------------------------------

A depreciation in the U.S. dollar relative to the English pound

* will make imports from England cheaper for the U.S.


* is the same thing as an increase in the consumer price index.
* is the same thing as an appreciation of the English pound relative to the dollar.
* will make U.S. exports more expensive for the English.

That answer is incorrect.


Correct answer:
is the same thing as an appreciation of the English pound relative to the dollar.

If the U.S. dollar depreciates relative to the pound the purchasing power of the dollar has decreased
relative to the pound; similarly, if the English pound appreciates relative to the U.S. dollar the purchasing
power of the pound relative to the dollar has increased. These statements are equivalent because a
decline in purchasing power of the dollar relative to the pound simply implies that the pound increased in
purchasing power relative to the dollar.

------------------------------

If labor-intensive textile products could be produced more cheaply in low-wage countries than in the U.S.,
the U.S. would gain if it

* levied a tax on the domestic textile products to penalize the industry for inefficiency.
* subsidized the domestic textile industry so it could compete in international markets.
* levied a tariff on the goods produced by the cheap foreign labor and subsidized the domestic textile
industry so it could compete in international markets.
* used its resources to produce other items, while importing textiles from foreigners.
* levied a tariff on the goods produced by the cheap foreign labor.

That answer is incorrect.


Correct answer:
used its resources to produce other items, while importing textiles from foreigners.

Low-wage and high-wage countries are better off if they are allowed to freely trade with each other. The
comparative advantage of low wage countries will be in the production of labor intensive goods while the
advantage for high wage countries will be in the production of capital intensive goods. High wage
countries such as the U.S. should import labor intensive goods and export capital intensive goods. By
specializing in the area of their comparative advantage, total production for each country will increase as
will real income.

------------------------------

NAFTA will allow:

* greater economies of scale in production


* all of these answers
* the greater movement of goods and investment among the countries of North America
* domestic producers to have access to larger markets
* many products to be produced at a lower per unit cost

That answer is incorrect.


Correct answer:
all of these answers

NAFTA will expand the free movement of goods and investment among the countries of North America
and will allow domestic producers access to a larger market. As a result, products will be produced at a
lower per unit cost because producers will be able to plan for and produce a larger volume of output.
Economies of scale are also then more likely.

------------------------------

The world demand Q for a good Z at a price P is given by Q = 1,200 - 4P while the world supply function is
given by 300 + 6P. The demand function for the same good in the U.S. is given by Q = 400 - 2P and the
supply function is given by Q = 200 + 3P. If there are no import or export restrictions, the U.S. net exports
of good Z equal:

* there will be no export but import of 120 units.


* 190 units
* 250 units
* 390 units

That answer is incorrect.


Correct answer:
250 units

The world price of good Z is given by the market equilibrium in the world markets, determined by the
condition, supply = demand. The equilibrium price then satisfies 1,200 - 4P = 300 + 6P, giving the world
price to be 90. At this price, the domestic producers will produce a quantity equal to 200 + 3*90 = 470
units. On the other hand, the domestic consumers will consume 400 - 2*90 = 220 units. Thus, the U.S.
producers will have a net excess domestic supply of 470 - 220 = 250 units, which they will export to the
rest of the world.

------------------------------
When a country imposes restrictions on imports, the price paid by the domestic consumers of the imported
goods:

* decreases.
* could remain unaffected.
* decreases or could remain unaffected.
* increases.

That answer is incorrect.


Correct answer:
increases.

When imports are restricted, the supply of the good in the domestic market is lower than it would be in the
absence of the restrictions. Hence, the price prevailing in the domestic market is higher in the presence of
restrictions.

------------------------------

If the price of one French franc fell from 40 to 20 U.S. cents, then

* computers manufactured in the U.S. would be cheaper for French businesses.


* U.S. currency would have appreciated against the French franc.
* French wines would be more expensive for U.S. consumers.
* U.S. currency would have depreciated against the French franc.

That answer is incorrect.


Correct answer:
U.S. currency would have appreciated against the French franc.

Prior to the depreciation of the French franc Americans had to spend 40 cents to purchase one French
franc. Afterwards, Americans had to only spend 20 cents to purchase on French franc. Therefore the
value of the U.S. dollar has appreciated relative to the French franc because it now costs Americans less
to purchase one franc. The buying power of the dollar has increased.

------------------------------

The exchange rate is:

* all of these answers.


* the rate at which large volumes of money are exchanging on the foreign exchange market.
* none of these answers.
* the fee charged for exchanging a domestic export with an import from another country.
* the price of one national currency in terms of another.

That answer is incorrect.


Correct answer:
the price of one national currency in terms of another.

The exchange rate is the domestic price of one unit of foreign currency.
------------------------------

If the exchange-rate of the German mark goes from 60 cents to 75 cents, then the mark has

* appreciated and Germans will find U.S. goods cheaper.


* depreciated and Germans will find U.S. goods more expensive.
* appreciated and Germans will find U.S. goods more expensive.
* depreciated and Germans will find U.S. goods cheaper.

That answer is correct!

According to the above scenario, it used to cost Americans 60 cents to purchase one German mark (or it
costs 1.666 marks to purchase one dollar); under the depreciation of the U.S. dollar, it now costs 75 cents
to purchase one mark (or it costs 1.333 marks to purchase one dollar). Dollars are now relatively less
expensive for Germans as are U.S. products. As a result, the German currency has appreciated and
Germans will find U.S. goods cheaper.

------------------------------

A nation experiencing an excess of imports over exports can remedy this situation under a fixed rate
system by:

* reducing its trade barriers.


* upwardly revaluing its currency.
* none of these answers.
* following expansionary macroeconomic policy.
* all of these answers.

That answer is incorrect.


Correct answer:
none of these answers.

A country has three choices to remedy the situation of excess imports over exports: a devaluation can
restore equilibrium between the demand and supply of the currency in the exchange market. Enacting
trade barriers in the form of tariffs and quotas reduces imports but is in conflict with economic efficient.
Restrictive macroeconomic policy retards inflation and increases interest rates.

------------------------------

Under a 10 percent depreciation of the U.S. dollar, if Americans reduce their consumption of imports by 5
percent:

* foreigners will earn less than they did before the depreciation.
* total expenditure of U.S. dollars on imports will decline.
* none of these answers.
* total expenditure of U.S. dollars on imports will rise.
* American firms will earn less than they did before the depreciation.
That answer is incorrect.
Correct answer:
total expenditure of U.S. dollars on imports will rise.

Under a 10 percent depreciation, unless Americans reduce the quantity of their imports by more than 10
percent, their expenditures on imports will increase.

------------------------------

Which of the following is correct?

* The wages of U.S. workers would decline if we traded freely with low-wage countries such as Mexico.
* An increase in the tariff on foreign-produced automobiles will benefit U.S. consumers of domestic cars.
* A probable result of the North American Free Trade Agreement is that producers in both countries will
benefit at the expense of consumers.
* Exports provide a nation with its primary source of purchasing power used to import goods.

That answer is incorrect.


Correct answer:
Exports provide a nation with its primary source of purchasing power used to import goods.

Exports such as goods, services and assets provide the buying power that makes it possible for a nation
to import other goods. Without exports, a nation would not have the foreign currency that is required for
the purchase of imports.

------------------------------

________ are contracted today but with delivery and settlement in the future.

* Forward exchange rates in the interbank market


* None of these answers
* Forward premiums
* Spot exchange rates in the interbank market

That answer is correct!

Forward exchange rates in the interbank market are contracted today but with delivery and settlement in
the future, usually 30 to 90 days hence.

------------------------------

________ states that high interest rates on a currency are offset by forward discounts and that low interest
rates are offset by forward premiums.

* Bid-ask spread
* Triangular currency arbitrage
* Covered interest differential
* Interest rate arbitrage
* Interest rate parity

That answer is incorrect.


Correct answer:
Interest rate parity

The formula for approximating interest rate parity is rh - rf = f1 - e0/e0.

------------------------------

________ is the world's largest currency trading market.

* New York
* Hong Kong
* Tokyo
* London
* Washington, DC

That answer is incorrect.


Correct answer:
London

London's volume is followed by New York and Tokyo.

------------------------------

If the French franc moves from 8.00 to 9.10 francs to the dollar:

* the U.S. dollar has depreciated.


* the French franc has appreciated.
* the franc now trades for 9.10 dollars.
* the dollar now trades for 0.11 francs.
* the French franc has depreciated.

That answer is incorrect.


Correct answer:
the French franc has depreciated.

Since the U.S. dollar now purchases more French francs the U.S. dollar has appreciated; symmetrically,
the French franc has depreciated.

------------------------------

The Chinese Yen has a bid price of $.1207 and an ask price of $.1208. What is the bid-ask spread?

* none of these answers


* 0.83%
* 0.1%
* 0.01%
* 0.083%

That answer is incorrect.


Correct answer:
0.083%

The bid-ask spread is computed as follows: Percent spread = (Ask price - Bid price)/Ask price X 100, or
.1208 - .1207/1.4428 X 100 = 0.083%

------------------------------

The primary source of purchasing power used to import goods is

* the monetary sector.


* the domestic currency of a nation.
* taxation and other revenue-generating activities.
* the revenue received from exports.
* the balance of payments deficit.

That answer is incorrect.


Correct answer:
the revenue received from exports.

Exports such as goods, services and assets provide the buying power that makes it possible for a nation
to import other goods. Without exports, a nation would not have the foreign currency that is required for
the purchase of imports.

------------------------------

Suppose the equilibrium price of the U.S. dollar is 5.5 French Franc. If the French government were to fix
the exchange rate at 5.2 French Franc per U.S. dollar, which of the following effects would occur?

I. The French Franc reserves held by the U.S. will deplete.

II. The U.S. will export more goods to France than it would under a free-floating equilibrium.

III. For the exchange rate to be maintained at this level for a long time, the French government will have to
synchronize its fiscal and monetary policies with that of the U.S.

* I & III
* II & III
* I, II & III
* I only

That answer is incorrect.


Correct answer:
II & III

Since the fixed rate has been set below the equilibrium rate, the French Franc is overvalued and the U.S.
dollar undervalued. To defend this higher value for French Franc, the French Central Bank will have to
continuously dip into its reserves of U.S. dollars to satisfy the high demand caused by the low rate. Thus,
in the short term, the U.S. dollar reserves of the French central bank will deplete. Further, since the
French Franc is more expensive under the fixed rate regime, the French goods are costlier and hence, the
U.S. will import less from France. On the other hand, since the U.S. dollar is cheaper for the French
consumers, they will increase their imports of U.S. goods. Clearly, this situation cannot be maintained for
long without something giving way. If France wants to maintain the lower rate for a long time, it must
adjust its fiscal and monetary policy to be consistent with an equilibrium rate of 5.2 FF per USD.

------------------------------

Suppose the dollar rises from 125 to 150 yen. As a result,

* exports to Japan will likely increase.


* Japanese tourists will more likely visit the U.S.
* U.S. businesses will be less likely to use Japanese shipping lines to transport their products.
* U.S. consumers will more likely buy Japanese-made automobiles.

That answer is incorrect.


Correct answer:
U.S. consumers will more likely buy Japanese-made automobiles.

The purchasing power of the U.S. dollar has increased relative to the Japanese Yen. Thus, a single dollar
now buys more Japanese goods than it did before the currency appreciation. As a result, Americans will
demand more Japanese goods since they are now relatively cheaper.

------------------------------

Which of the following would most likely cause a nation's currency to depreciate?

* a balance of trade surplus


* an increase in domestic real interest rates
* an increase in the nation's inflation rate
* an increase in exports coupled with a decline in imports

That answer is incorrect.


Correct answer:
an increase in the nation's inflation rate

Domestic inflation will cause the currency to depreciate because the demand for the nation's output will
fall as prices increases. According to the law of demand as price rises the demand for a good decreases.
Thus, rising domestic prices implies that foreign demand for the nation's output will fall. Falling demand for
goods implies falling demand for domestic currency; this decrease in demand causes the "price" or
exchange rate of the nation to fall or depreciate.

------------------------------

The domestic demand Q for a good A at a price P is given by Q = 800 - 2P while the supply function is
given by 200 + 4P. If the government imposes an import quota of 200 units, the revenues of the domestic
producers of good A equal ________.

* 26,936
* 19,485
* 22,496
* 31,113

That answer is incorrect.


Correct answer:
31,113

With the import quota, the domestic price of good A equals M. At this price, the producers produce 200 +
4M units and consumers demand a larger amount, 800 - 2M units. The difference between these two
equals the total imports allowed. Therefore, (800 - 2M) - (200 + 4M) = 200. Solving for M gives M = 66.67.
The total domestic production at this price is then 200 + 4*66.67 = 467 units and the total revenues equal
467*66.67 = 31,113.

------------------------------

The price of one country's currency in terms of another is called

* inflation.
* the exchange-rate.
* the J-curve.
* special drawing rights.
* the interest rate.

That answer is incorrect.


Correct answer:
the exchange-rate.

The exchange rate refers to the domestic price of one unit of foreign currency.

------------------------------

Which of the following would be a credit in the U.S. balance of payments?

* a trip to Japan by an American student


* the purchase of insurance from Lloyd's of London by a U.S. resident
* a short-term loan extended to an American company by a Swiss bank
* the purchase of a German car by an American
* a short-term loan extended to a European business by a U.S. bank

That answer is incorrect.


Correct answer:
a short-term loan extended to an American company by a Swiss bank

A credit appears on the U.S. balance of payments whenever there is a decrease in U.S. currency on the
foreign exchange market. Since the loan would be extended in U.S. dollars the Swiss bank exchanges
Swiss francs for U.S. dollars and thus diminishes the supply of U.S. currency on the market. Therefore,
this transaction appears as a credit.

------------------------------

A nation experiencing an excess of imports over exports can remedy this situation under a fixed rate
system by:

* upwardly revaluing its currency.


* none of these answers.
* reducing its trade barriers.
* following expansionary macroeconomic policy.
* devaluing it currency.

That answer is incorrect.


Correct answer:
devaluing it currency.

A country has three choices to remedy the situation of excess imports over exports: a devaluation can
restore equilibrium between the demand and supply of the currency in the exchange market. Enacting
trade barriers in the form of tariffs and quotas reduces imports but is in conflict with economic efficient.
Restrictive macroeconomic policy retards inflation and increases interest rates.

------------------------------

International evidence suggests that

* countries that impose low trade restrictions have low rates of economic growth.
* countries that have the lowest trade restrictions have the highest rates of economic growth.
* there is no link between the degree of a country's trade restrictions and its economic growth.
* countries that impose high trade restrictions have high rates of economic growth.

That answer is incorrect.


Correct answer:
countries that have the lowest trade restrictions have the highest rates of economic growth.

It has been shown that LDCs that have low tariff rates, a freely convertible currency and large trade
sectors grew rapidly during the 1980-91 period while the per capita GDP of countries following the most
restrictive trade policies generally declined.

------------------------------

What is not true concerning forward contracts?

* The Deutsche mark is the most widely traded currency.


* Forward markets for the currencies of less-developed countries have become increasingly popular.
* A forward contract is between a bank and a customer.
* A forward contract calls for delivery at a fixed future date.
* A forward contract specifies the amount of one currency against dollar payment.
That answer is incorrect.
Correct answer:
Forward markets for the currencies of less-developed countries have become increasingly popular.

In general, forward markets for the currencies of less-developed countries are either limited or
nonexistent.

------------------------------

Opportunity costs differ among nations primarily because

* nations have different political, religious and economic institutions.


* work-leisure preferences vary considerably from one nation to another.
* the inflation rates in nations differ, causing opportunities to differ.
* nations have different endowments of land, labor skills, capital and technology.
* nations employ different currencies.

That answer is incorrect.


Correct answer:
nations have different endowments of land, labor skills, capital and technology.

Labor force and resource endowments differ greatly across countries. These difference influence costs.
Therefore, a good that is quite costly to produce in one country may be economically produced in another
country.

------------------------------

When a country fixes the exchange rate value of their currency above the market rate which of the
following will result?

* Residents of that country can buy as much foreign currency as they want.
* There will be a surplus of foreign exchange.
* There will be a shortage of foreign exchange.
* A depreciation of the currency will restore equilibrium in the foreign exchange market.

That answer is incorrect.


Correct answer:
There will be a shortage of foreign exchange.

An artificially high foreign exchange rate will cause a country's exports to be extremely expensive to
foreigners. As a result, foreigners will purchase goods elsewhere and the country's exports will be small. A
low level of exports implies a low level of foreign currency available to domestic consumers. There will be
a shortage of foreign exchange.

------------------------------

Which foreign exchange participant engages in forward contracts to protect the home currency value of
various foreign currency-denominated assets and liabilities on their balance sheets?
* speculators
* traders
* arbitrageurs
* brokers
* hedgers

That answer is incorrect.


Correct answer:
hedgers

Hedgers engage in forward contracts to protect the home currency value of various foreign currency-
denominated assets and liabilities on their balance sheets that are not to be realized over the life of the
contracts.

------------------------------

I. The international trade sector contributes a larger share to GDP in the U.S. than in Japan.

II. The international trade sector contributes a larger share to GDP in the United Kingdom than in the U.S.

* Both statements are true.


* I is true, II is false.
* II is true, I is false.
* Both statements are false.

That answer is correct!

As of 1991, the Japanese exports as a percent of total output was equal to 10 percent; the equivalent
figure for the U.S. was 11 percent. The equivalent figure for the United Kingdom was 24 percent.

------------------------------

Propelled by an older population and a high saving rate, Japanese citizens make substantial real and
financial investments abroad. If Japanese investment abroad exceeds foreign investment in Japan (a
capital account deficit), then under a flexible exchange rate system the Japanese

* yen must appreciate on the exchange rate market.


* yen must depreciate on the exchange rate market.
* must run an equal deficit on their current account transactions.
* must run an offsetting surplus on their current account transactions.

That answer is incorrect.


Correct answer:
must run an offsetting surplus on their current account transactions.

Since the balance of payments must balance it must be true that if the capital account is in surplus that the
current account is in deficit or vice versa. This implies that the sum of the capital account and the current
account must equal zero and thus cancel each other out.
------------------------------

________ aligns exchange rate quotations throughout the world.

* None of these answers


* Arbitrage
* The balance of payments
* The offer price
* The bid price

That answer is incorrect.


Correct answer:
Arbitrage

Arbitrage assures that the exchange rate quotations in Frankfurt are the same as they are in NY. Without
arbitrage there would be the potential for huge profits in a very short period of time.

------------------------------

A foreign currency is ________ if the forward rate expressed in dollars is above the spot rate.

* in-the-money
* at a forward discount
* at a forward premium
* out-of-the-money
* positively correlated

That answer is incorrect.


Correct answer:
at a forward premium

There is a forward premium if the forward rate is above the spot rate.

------------------------------

Under a system of fixed exchange rates, which of the following will most likely reduce a balance of
payments deficit?

* an increase in foreign-aid grants


* an expansionary monetary policy, which will drive prices up and interest rates down
* a reduction in income from investments abroad
* a restrictive monetary policy, which will keep prices down and interest rates up

That answer is incorrect.


Correct answer:
a restrictive monetary policy, which will keep prices down and interest rates up

A restrictive monetary policy will prevent prices from increasing in the economy (this is one of the ways
that the Fed can control inflation). Price deflation will attract foreign consumers to purchase U.S. goods
which will cause the balance of payments deficit to decline. Since the exchange rate is fixed it is not
allowed to appreciate and therefore contribute to the balance of payments deficit.

------------------------------

If the French franc is quoted as $0.2051-82 and the German Deutsche mark is quoted as $0.6024-45,
what is the bid cross rate for the French franc?

* FF 2.9473/DM
* FF 2.8821/DM
* FF 2.9035/DM
* FF 2.9371/DM
* FF2.8934/DM

That answer is incorrect.


Correct answer:
FF2.8934/DM

The bid cross rate equals the bid rate for mark divided by the ask rate for the franc, in this case, 0.6024
divided by 0.2082.

------------------------------

Under a depreciation of the U.S. dollar:

* foreigners will buy fewer U.S. exports and therefore total expenditure will decline.
* total expenditure by foreigners on U.S. exports will decrease if demand increases by a larger
percentage than the dollar depreciated.
* total expenditure by foreigners on U.S. exports will decrease if demand increases by a smaller
percentage than the dollar depreciated.
* none of these answers.
* total expenditure by foreigners on U.S. exports will increase if demand increases by a smaller
percentage than the dollar depreciated.

That answer is incorrect.


Correct answer:
total expenditure by foreigners on U.S. exports will decrease if demand increases by a smaller percentage
than the dollar depreciated.

The depreciation of the U.S. dollar will make U.S. exports cheaper to foreigners. Unless foreigners
increase the quantity they purchase by the same percentage as the currency depreciated, their
expenditure in terms of dollars will decline.

------------------------------

If a nation wants to maintain a fixed exchange rate at a time when supply and demand are causing its
currency to depreciate in the exchange market, the nation might
* tax exports and subsidize imports.
* reduce its trade barriers (tariffs and quotas).
* shift to a more restrictive monetary policy.
* expand the money supply to lower interest rates.

That answer is incorrect.


Correct answer:
shift to a more restrictive monetary policy.

By following a restrictive monetary policy deflation will occur and interest rates will rise. As a result exports
are encouraged and imports are encouraged (as is investment in domestic assets). The increase in
demand for the nation's currency will cause the currency to appreciate.

------------------------------

The ________ between the bid and ask prices is _______.

* spread; a form of remuneration for the dealer


* points; dependent on the currency traded
* points; a form of remuneration for the dealer
* none of these answers
* difference; fixed by the other the country market for currency exchange

That answer is correct!

The spread, the difference between the bid and ask price for currencies, is a form of remuneration for the
dealer who makes a firm quotation without knowing whether the customer wants to buy or sell.

------------------------------

Devaluation differs from depreciation primarily in that

* depreciation can be a devaluation, but a devaluation can never be a depreciation.


* depreciation applies to imports, whereas devaluation pertains only to exports.
* depreciation applies to exports, whereas devaluation pertains only to imports.
* devaluation is an official government act under a system of fixed exchange rates, whereas depreciation
may occur under either fixed or flexible rates.
* depreciation involves a change in the price of gold in terms of a nation's currency; devaluation does not.

That answer is incorrect.


Correct answer:
devaluation is an official government act under a system of fixed exchange rates, whereas depreciation
may occur under either fixed or flexible rates.

Devaluation refers to a one-step reduction in the value of a nation's currency under a fixed rate system.
Depreciation refers to a reduction in the value of a domestic currency relative to foreign currencies.
Depreciation occurs due to the forces of supply and demand in the foreign exchange market.

------------------------------
A(n) ________ quota, like a tariff, is designed to restrict foreign goods and protect domestic industries.

* dumping
* none of these answers
* import
* transference
* piling

That answer is incorrect.


Correct answer:
import

A import quota specifies the quantity of a good that is permitted to be imported into a country during a
given year.

------------------------------

A tariff differs from a quota in that a tariff is

* levied on exports, whereas a quota is imposed on imports.


* levied on imports, whereas a quota is imposed on exports.
* a percentage or absolute tax imposed on imports, whereas a quota is an absolute limit to the number of
units of a good that can be imported.
* a tax levied by a foreign country, whereas a quota is a limit on the trade allowed.

That answer is incorrect.


Correct answer:
a percentage or absolute tax imposed on imports, whereas a quota is an absolute limit to the number of
units of a good that can be imported.

A tariff is levied on goods imported into a country while an import quota specifies a maximum amount of
goods to be imported into a country during a given year.

------------------------------

A Japanese automobile manufacturer building an auto plant in the U.S. creates a

* supply of both dollars and yen in the foreign exchange market.


* demand for both dollars and yen in the foreign exchange market.
* supply of dollars and demand for yen in the foreign exchange market.
* demand for dollars and a supply of yen in the foreign exchange market.

That answer is incorrect.


Correct answer:
demand for dollars and a supply of yen in the foreign exchange market.

In order to finance the construction of an auto plant in the U.S. the Japanese firm must exchange
Japanese yen for U.S. dollars. As a result the foreign exchange market experiences an increase in the
demand for U.S. dollars and an increase in the supply of Japanese yen.

------------------------------

A major difference between a tariff and a quota is that a tariff

* typically generates tax revenues while a quota does not.


* will reduce the ability of foreigners to obtain the purchasing power to buy a nation's export goods, but a
quota will not affect the demand of foreigners for the nation's exports.
* will reduce imports but a quota generally will not.
* can easily be rescinded but a quota cannot.

That answer is correct!

With a quota, foreign producers are prohibited from selling additional units regardless of how much lower
their costs are relative to domestic producers. A quota brings in no revenue for the government. A tariff
transfers revenue from domestic consumers to the Treasury while a quota transfers these revenues to
foreign producers.

------------------------------

If restrictive monetary policy results in a deceleration in domestic inflation and higher real interest rates,
other things constant, the

* nation will run a balance of trade surplus.


* nation's currency will appreciate.
* nation will run a capital account deficit.
* nation's currency will depreciate.

That answer is incorrect.


Correct answer:
nation's currency will appreciate.

A decline in domestic inflation will make U.S. goods less expensive to foreigners; therefore, the demand
for U.S. goods will increase as will the demand for U.S. currency (which must be used to purchase those
goods from abroad). A rise in the real interest in the U.S. will also increase the demand for U.S. currency
since foreign investors will want to buy high yield U.S. assets. The consequence of an increased demand
for U.S. currency is an appreciation on the exchange rate.

------------------------------

The Eurobond market is:

* an interbank market closely linked to the foreign exchange market.


* none of these answers.
* a market for short-term borrowing and lending and is an off-shore market.
* a joint float system of currency exchange between major European countries.
* a traditional bond market whereby a syndicate of underwriters is assembled and in a few weeks floats a
bond for a specific company.
That answer is incorrect.
Correct answer:
a traditional bond market whereby a syndicate of underwriters is assembled and in a few weeks floats a
bond for a specific company.

The Eurobond market should be carefully distinguished from the Eurocurrency market which is a market
for short-term borrowing and lending and is an off-shore market. In contrast, the Eurobond market is a
traditional bond market whereby a syndicate of underwriters is assembled and in a few weeks floats a
bond for a specific company.

------------------------------

Under a flexible exchange rate system, a nation that offers more attractive investment opportunities than
its trading partners can expect to run a

* deficit on current account transactions and capital account surplus.


* deficit on current account transactions.
* surplus on current account transactions.
* balance of trade surplus.
* capital account surplus.

That answer is correct!

Since the capital account records transactions that involve the exchange of ownership rights to real of
financial assets and the extension of loans a capital account surplus would develop if foreign investors
start to invest heavily in the nation. If a nation is experiencing a surplus on its capital account balance, it
must experience an offsetting deficit on its current account and vice versa. By definition, the balance of
payments must be in balance so that the capital account offsets the current account.

------------------------------

Suppose transactions costs are .2% on currency transactions. The quote for the French franc is FF 6.35/$.
What is the bid-ask spread that results from the existence of transaction costs? Hint: convert a dollar into
francs and then back into dollars and see how much is lost due to transaction costs.

* 0.1534 to 0.1559
* 0.1569 to 0.1581
* 0.1498 to 0.1546
* 0.1735 to 0.1789
* 0.1645 to 0.1699

That answer is incorrect.


Correct answer:
0.1569 to 0.1581

To calculate the transaction costs, convert $1 into francs minus transaction costs: 6.35 x .998 = 6.3373.
Converting francs into dollars provides 6.35 x .998 x e x .998, solving for exchange rate e. The dollar
amount must be less than or equal to $1, or else there will be arbitrage opportunities, so e
<=1[6.35/(0.998)^2)] = 0.1581. In an alternative transaction, the arbitrageur could convert $1 to francs at
rate e, exchange the francs for dollars at FF 6.35 and pay transaction costs for both exchanges. For there
to be no arbitrage, (0.998)^2 x (1/6.35e) <=1, or e>=$0.1569. So, the existence of 0.2% transactions costs
results in a bid-ask spread of 0.1569 to 0.1581.

------------------------------

If the value of the Japanese yen is quoted as 90.00 yen per U.S. dollar:

* then the dollar has appreciated.


* one dollar can be exchanged in the foreign exchange market for 90.00 yen.
* one dollar purchases one-ninetieth of a yen.
* one yen can be exchanged in the foreign exchange market for 90.00 dollars.
* then the yen has depreciated.

That answer is incorrect.


Correct answer:
one dollar can be exchanged in the foreign exchange market for 90.00 yen.

This quote implies two things: one dollar can be exchanged in the foreign exchange market for 90.00
francs or one yen can be exchanged for 1/90 dollars.

------------------------------

The major commercial banks continuously deal in foreign exchange for their own accounts and, therefore,
are ________.

* CHIPS
* market makers
* foreign markets
* foreign exchange brokers
* marketeers

That answer is incorrect.


Correct answer:
market makers

Most interbank transactions in the U.S. are conducted through foreign exchange brokers.

------------------------------

Propelled by an older population and a high saving rate, Japanese citizens make substantial real and
financial investments abroad. If Japanese investment abroad exceeds foreign investment in Japan (a
capital account deficit), then under a flexible exchange rate system the Japanese

* must run an offsetting surplus on their current account transactions.


* yen must appreciate on the exchange rate market.
* must run an equal deficit on their current account transactions.
* yen must depreciate on the exchange rate market.
That answer is correct!

According to the balance of payments which requires that the capital account exactly offset the current
account, the Japanese must have a current account surplus.

------------------------------

The short-run impact of a foreign exchange depreciation often differs from the long-run impact because

* it takes time for other countries to notice the change.


* it takes time for consumers to adjust their purchasing patterns to the new prices.
* people react to the new prices quite favorably at first but then fall back into old buying patterns.
* none of these answers, there is little difference between the long-run and short-run impacts of a change
in exchange rates.

That answer is incorrect.


Correct answer:
it takes time for consumers to adjust their purchasing patterns to the new prices.

The J-curve effect (the tendency for a nation's current account deficit to widen initially before it shrinks in
response to an exchange rate depreciation, occurs because the short-run elasticity of domestic demand
for imports and foreign demand for exports is inelastic. In the long run, however, the demand for both
imports and exports is elastic.

------------------------------

I. Individuals harmed by trade restrictions will likely be uninformed and unconcerned about our trade
policy.

II. Often there will be a conflict between sound economics and good politics on trade restriction issues.

* Both statements are true.


* I is true, II is false.
* Both statements are false.
* I is false, II is true.

That answer is correct!

In general, the groups that benefit from trade restrictions such as producers and resource suppliers, are
well organized and provide visible evidence in the form of increased jobs etc of the positive effects of trade
restrictions. The groups that lose under trade restrictions such as consumers are unorganized or unaware
of the potential harm imposed by the restrictions. Politicians are often swayed by the political clout of the
gainers under restricted trade and thus sound economics often conflicts with a winning political strategy. In
these cases, the winning political strategy often prevails.

------------------------------

The Balance of Trade account belongs to which of the following?


I. Current account
II. Capital account
III. Balance of Payments account.
IV. Reserve account.

* II & III
* II only
* I only
* III only
* I & III
* IV only

That answer is incorrect.


Correct answer:
I & III

The Balance of Trade account is a sub-account of the Current account, which is part of the Balance of
Payments account.

------------------------------

I. Since imports and exports are usually consumed and produced in different sectors of an economy,
imports exert little impact on the quantity of goods a nation exports.

II. Even in a situation where one country has an absolute advantage over another in producing all goods,
both nations will still gain from trade if the opportunity costs of producing goods differ between the two
countries.

* Both statements are false.


* Both statements are true.
* I is false, II is true.
* I is true, II is false.

That answer is incorrect.


Correct answer:
I is false, II is true.

I is false because exports such as goods, services and assets provide the buying power that makes it
possible for a nation to import other goods. Without exports, a nation would not have the foreign currency
that is required for the purchase of imports. Nations can gain from trade even if there is one nation with an
absolute advantage in both goods. This is because as long as there are differences in the relative costs of
producing the goods across two countries, both can gain from trade. This is because each country will find
it cheaper to trade for goods that can be produced only at a high opportunity cost.

------------------------------

Currency A is selling for $0.61 and the buying rate for Currency B is $0.19. The currency A : currency B
cross rate is ______.

* A 1 = B .31
* None of these answers
* A 1 = B .81
* A 1 = B .16
* A 1 = B 1.33

That answer is incorrect.


Correct answer:
None of these answers

.61/.19 = A 1 = B 3.21

------------------------------

What is the annualized forward premium if Deutsche marks are selling for $0.5869 and 180 day forward
market rate is $0.5975?

* 2.35%
* 3.45%
* 3.61%
* 1.17%
* 1.48%

That answer is incorrect.


Correct answer:
3.61%

The forward premium equals the forward rate minus the spot rate all divided by the spot rate times 360
divided by the number of the days of the forward contract, in this case, (.5875-.5869)/0.5869 x (360/180) =
3.61%

------------------------------

Those who advocate fixed, rather than flexible, exchange rates base their arguments primarily on the view
that

* fixed rates automatically create an equilibrium price for each nation's currency in the foreign exchange
market.
* under fixed exchange rates a nation pays for its imports with exports.
* flexible rates generally lead to a balance of payments surplus.
* flexible rates generally lead to a balance of payments surplus and fixed rates automatically create an
equilibrium price for each nation's currency in the foreign exchange market.
* flexible rates generally lead to a balance of payments deficit.

That answer is incorrect.


Correct answer:
under fixed exchange rates a nation pays for its imports with exports.

The premise of a fixed rate system is that a nation ordinarily pays for its imports with exports. Countries
may temporarily experience periods where imports exceed exports; during such periods, nations can draw
down their reserve balance.
------------------------------

In response to an exchange rate depreciation, a nation's current account deficit will often initially widen
before it shrinks in the long run. Economists refer to this time path of adjustment as

* devaluation.
* the J-curve effect.
* the Laffer curve.
* the substitution effect.

That answer is incorrect.


Correct answer:
the J-curve effect.

The tendency of a nation's current account deficit to widen initially before it shrinks in response to an
exchange rate depreciation.

------------------------------

When the government imposes tariffs on imported goods, which of the following groups benefit(s)?

I. The government.
II. The domestic consumers.
III. The domestic producers.
IV. The foreign producers.

* I, II & IV
* I & III
* III only
* II only
* III & IV
* IV only
* II & IV
* I only

That answer is incorrect.


Correct answer:
I & III

When the government implements imports tariffs, the imports fall and domestic consumers pay higher
prices to domestic producers than they would otherwise pay. In the process, the government collects part
of the higher prices as revenue.

------------------------------

Under a system of flexible exchange-rates, which of the following will most likely cause a nation's currency
to appreciate on the foreign exchange market?
* a decrease in domestic interest rates
* an increase in foreign interest rates
* domestic inflation of 10 percent while the nation's trading partners are experiencing stable prices
* stable domestic prices while the nation's trading partners are experiencing 10 percent inflation

That answer is incorrect.


Correct answer:
stable domestic prices while the nation's trading partners are experiencing 10 percent inflation

Inflation abroad coupled with stable prices at home will attract foreign consumers to purchase U.S. goods.
U.S. goods will now seem relatively less expensive. As a result, the demand for U.S. currency will
increase (consumers require U.S. dollars in order to purchase U.S. goods). Whenever the demand for a
currency increases the "price" or exchange rate of the currency increases. Thus, the currency appreciates.
The exchange rate is essentially the price of one currency in terms of another currency.

------------------------------

Macroeconomic policy:

* none of these answers.


* has no effect on the capital or current account.
* exerts an impact on the capital account.
* exerts an impact on both current account and capital account balances.
* exerts an impact on the current account.

That answer is incorrect.


Correct answer:
exerts an impact on both current account and capital account balances.

Since unanticipated shifts in macroeconomic policy influence both the demand for imports and real
interest rates, they will exert an impact on both current account and capital account balances.

------------------------------

An depreciation in a currency implies:

* fewer units of foreign currency are now required to purchase on unit of the currency.
* that foreign goods are now more expensive.
* all of these answers.
* less units of a foreign currency are now required to purchase the currency.
* the purchasing power of the currency has decreased.

That answer is incorrect.


Correct answer:
all of these answers.

All of these answers reflect the same idea: a depreciation in a currency implies an decrease in the value of
a domestic currency relative to foreign currencies. A depreciation decreases the purchasing power of the
domestic currency over foreign goods.
------------------------------

What impact will an increase in the supply of ball-point pens imported to the U.S. have on the domestic
price of ball-point pens? Assume the pens are virtually identical regardless of country of origin.

* The domestic price will fall.


* The domestic price will remain constant.
* It is impossible to predict the impact.
* The domestic price will rise.

That answer is correct!

An increase in supply of any good will cause the price of the good to fall.

------------------------------

The domestic demand Q for a good X at a price P is given by Q = 400 - 2P while the supply function is
given by 100 + 4P. If the world price for good X is 45, the amount of good X imported, in the absence of
any restrictions, equals ________.

* 30 units
* 13 units
* 40 units
* there will be no import but export of 10 units

That answer is correct!

Without restrictions, the domestic consumers will be able to consume at the world market price of 45. At
this price, the domestic producers will produce a quantity equal to 100 + 4 * 45 = 280 units. On the other
hand, the domestic consumers will consume 400 - 2 * 45 = 310 units. Thus, the U.S. producers will not be
able to meet the entire domestic demand and the U.S. will end up importing 310 - 280 = 30 units.

------------------------------

Other things constant, which of the following will most likely cause the dollar to appreciate on the
exchange-rate market?

* expansionary domestic monetary policy


* reduced inflation abroad
* higher interest rates abroad
* higher domestic interest rates

That answer is incorrect.


Correct answer:
higher domestic interest rates

High domestic interest rates attract foreign investors wishing to buy high-yield assets. Thus, the demand
for U.S. currency increases. As the demand for the currency increases, the "price" of the currency
increases too. This implies that the currency will appreciate.

------------------------------

Under fixed exchange rates:

* the exchange rate value of a currency rises and falls further than the major currency to which it is tied.
* the exchange rate value of a currency rises and falls less than the major currency to which it is tied.
* none of these answers.
* the exchange rate is determined by the forces of supply and demand.
* the exchange rate value of a currency rises and falls with the major currency to which it is tied.

That answer is incorrect.


Correct answer:
the exchange rate value of a currency rises and falls with the major currency to which it is tied.

Fixed exchange rates imply that the exchange rate for a country is set at a fixed rate relative to all other
currencies; government policies are used to maintain the fixed rate.

------------------------------

Which of the following would be recorded as a credit in the U.S. balance of payments accounts?

* the purchase of a German business by a U.S. investor


* the import of Honda trucks by a U.S. automobile distributor
* European travel expenditures of an American college student
* the purchase of a U.S. Treasury bond by a French investment company

That answer is incorrect.


Correct answer:
the purchase of a U.S. Treasury bond by a French investment company

Since the purchase of a bond by a French company decreases the supply of U.S. dollars on the foreign
exchange market, this transaction is accounted for as a credit on the U.S. balance of payments account.

------------------------------

Imposing a restrictive quota on the import of spiked track shoes will likely

* leave the price of the shoes unchanged but decrease the quantity consumed.
* increase both the price of the shoes and the quantity consumed.
* leave the price of the shoes unchanged and also leave the quantity consumed unchanged since
domestic producers will expand output to make up for the reduction in the supply of the imported shoes.
* increase the price of the shoes but decrease the quantity consumed.

That answer is incorrect.


Correct answer:
increase the price of the shoes but decrease the quantity consumed.
Restricted foreign supply implies that the supply of the good will fall. Domestic supply will rise but not to
the level high enough to maintain the same level of supply in the market. Domestic producers enjoy a
higher price as a result of the restricted supply. The higher price in the market implies that domestic
consumption will fall.

------------------------------

A ________ exists when the ________ is higher than the spot rate.

* discount; forward rate


* discount; futures rate
* premium; forward rate
* premium; annualized spot rate
* none of these answers

That answer is incorrect.


Correct answer:
premium; forward rate

A premium exists when the forward exchange rate is higher than the spot rate and a discount exists
otherwise.

If the one month forward exchange rate is FF/$ = 8.0200 and the spot rate is FF/$=8.0000 then the dollar
quotes with a premium of .0200 francs.

------------------------------

Dumping

* is the sale of a good abroad at a cheaper price than what the good is sold for in the producer's domestic
market.
* all of these answers are correct.
* is generally encouraged by domestic producers of the product being dumped since they are the primary
beneficiaries of the dumping.
* generally benefits consumers of the nation receiving the "dumped" goods.
* is the sale of a good abroad at a cheaper price than what the good is sold for in the producer's domestic
market and generally benefits consumers of the nation receiving the "dumped" goods.

That answer is incorrect.


Correct answer:
is the sale of a good abroad at a cheaper price than what the good is sold for in the producer's domestic
market and generally benefits consumers of the nation receiving the "dumped" goods.

Dumping, which refers to the sale of a good by a foreign supplier in another country at a price lower than
the supplier sells it in its home country, generally benefits domestic consumers and imposes costs on
domestic producers of goods for which the imports are good substitutes. Lower prices permit consumers
to obtain the goods more economically than they are available from domestic producers. Lower prices in
turn make it more difficult for domestic producers of the goods to compete.
------------------------------

________ is the difference between the domestic interest rate and the hedged or "covered" foreign rate.

* Bid-ask spread
* The swap rate
* Interest rate parity
* Covered interest differential
* Interest rate parity

That answer is incorrect.


Correct answer:
Covered interest differential

When interest parity ensures that the return on a hedged foreign investment will equal the domestic
interest rate on investments of identical risk, the covered interest differential equals zero.

------------------------------

Under a flexible exchange system, which of the following will most likely cause a nation's currency to
appreciate on the foreign exchange market?

* a current account deficit


* a balance of trade deficit
* an acceleration in the nation's inflation rate
* a decline in the domestic inflation rate
* a decrease in domestic real interest rates

That answer is incorrect.


Correct answer:
a decline in the domestic inflation rate

A decline in the domestic inflation rate causes domestic goods to be relatively less expensive to foreign
consumers. Therefore, the demand for exports will increase as will the demand for the nation's currency.
This increase in demand will cause the "price" of the currency (or the exchange rate) to increase. This is a
currency appreciation.

------------------------------

An expansionary monetary policy implies that:

* the Fed decreases the growth rate of the money supply.


* none of these answers.
* the government decreases its spending.
* the government reduces the budget surplus.
* the Fed increases the growth rate of the money supply.

That answer is incorrect.


Correct answer:
the Fed increases the growth rate of the money supply.

An expansionary monetary policy implies that the Fed increases the growth rate of the money supply.

------------------------------

The law of comparative advantage explains why a nation will benefit from trade when

* it exports goods for which it is a high-opportunity cost producer while importing those for which it is a
low-opportunity cost producer.
* it imports more than it exports.
* it exports more than it imports.
* it exports goods for which it is a low-opportunity cost producer while importing those for which it is a
high-opportunity cost producer.
* its trading partners have higher wages.

That answer is incorrect.


Correct answer:
it exports goods for which it is a low-opportunity cost producer while importing those for which it is a high-
opportunity cost producer.

International trade leads to mutual gain because it allows each country to specialize more fully in the
production of those things that it does best. As long as trade is conducted according to the rules of
comparative advantage, total output will increase. Comparative advantage suggests that nations import
goods which are high opportunity cost goods and export low opportunity cost goods.

------------------------------

The primary benefits derived from tariffs usually accrue to the

* domestic consumers of goods protected by the tariffs.


* foreign producers of goods protected by the tariffs.
* domestic producers of export goods.
* domestic suppliers of goods protected by the tariffs.

That answer is incorrect.


Correct answer:
domestic suppliers of goods protected by the tariffs.

Domestic producers benefit under a tariff because the price of imported goods rise. This will create
upward pressure on the price of the good which the producer will enjoy. The domestic producer will
increase its supply. The domestic consumer will reduce consumption because of the higher price. Thus,
the producer wins and the consumer loses under a tariff.

------------------------------

If high-yield investment opportunities attract capital from abroad and lead to a capital account surplus,
then the
* nation must run a current account surplus under a flexible exchange rate system.
* nation's currency must depreciate.
* nation's currency must appreciate.
* nation must run a current account deficit under a flexible exchange rate system.

That answer is incorrect.


Correct answer:
nation must run a current account deficit under a flexible exchange rate system.

Since the balance of payments must balance it must be true that if the current account is in deficit that the
capital account is in surplus or vice versa. This implies that the sum of the capital account and the current
account must equal zero and thus cancel each other out.

------------------------------

Forward contracts are typically available for:

* 60-days
* 90-days
* 30-days
* All of these answers
* 180-360 days

That answer is incorrect.


Correct answer:
All of these answers

Banks may also customize forward contracts for odd maturities to meet their customers' needs.

------------------------------

When the government imposes import tariffs,

I. domestic producers are hurt.

II. there is a deadweight loss, representing inefficient allocation of resources.

III. domestic consumers gain since they now pay a lower price despite the tariffs.

IV. comparative advantage is not fully exploited.

* IV only
* II only
* III & IV
* II & IV
* II & III
* III only
* I only
* I & II
That answer is incorrect.
Correct answer:
II & IV

When the government implements imports tariffs, the imports fall and domestic consumers pay higher
prices to domestic producers than they would otherwise pay. In the process, the government collects part
of the higher prices as revenue but there is a net dead-weight loss.

------------------------------

In a purely floating exchange rate economy, a shift toward a more expansionary monetary policy will move
the capital account toward a ________. The current account will move toward a ________.

* surplus; deficit
* deficit; deficit also
* deficit; surplus
* surplus; surplus also

That answer is incorrect.


Correct answer:
deficit; surplus

An expansionary monetary policy will speed up the economic growth, increase inflation and decrease real
interest rates by increasing the money supply. This decrease in real rates will cause an outflow of funds
into economies offering higher rates, causing the domestic currency to depreciate. The outflow of funds
will move the capital account toward a deficit (or a smaller surplus). The current account will move toward
a surplus since the changes in current and capital accounts in a purely floating economy must equal zero.
Over a slightly longer run, the depreciation in the domestic currency will increase exports and decrease
imports, moving the current account further toward a surplus. Therefore, always look at the effects on the
capital account first.

------------------------------

Compared to the no-trade situation, when a country imports a good

* domestic consumers gain, domestic producers lose and the gains outweigh the losses.
* domestic consumers lose, domestic producers gain and the gains outweigh the losses.
* domestic consumers gain, domestic producers lose and the losses outweigh the gains.
* domestic consumers gain, but domestic producers lose an equal amount.

That answer is correct!

Domestic consumers gain under trade because the market price of the good will fall. Domestic producers
lose because the market price will fall and their market share will decline. The gains here outweigh the
losses to the total economy because the resources that are not being used due to a decline in domestic
market share can be applied to other goods which are low opportunity cost goods. A country will only
import those goods for which it is a high opportunity cost producer.

------------------------------
According to the law of comparative advantage, joint output will be greatest when each country

* lowers tariffs in a manner that will maximize employment in its domestic industries.
* raises tariff barriers to protect its domestic industries from cheap foreign products.
* specializes in producing those products for which it is the low-opportunity cost producer.
* specializes in producing those products for which it is the high-opportunity cost producer.

That answer is incorrect.


Correct answer:
specializes in producing those products for which it is the low-opportunity cost producer.

With international trade, each country can gain by specializing in the production of goods that it can
produce economically and using the proceeds to import goods that would be expensive to produce
domestically. This is essentially the law of comparative advantage.

------------------------------

Which of the following is a partially valid economic argument for restricting free trade?

* Infant industries may need temporary protection to develop and gain productive efficiency.
* A nation needs to protect its national defense; hence, it should restrict some products that threaten an
industry considered vital to its defense.
* Infant industries may need temporary protection to develop and gain productive efficiency and a nation
needs to protect its national defense; hence, it should restrict some products that threaten an industry
considered vital to its defense.
* Removal of restrictions that have existed for years will initially cause inflation.
* Restrictions on foreign trade will increase employment and permanently reduce unemployment.

That answer is incorrect.


Correct answer:
Infant industries may need temporary protection to develop and gain productive efficiency and a nation
needs to protect its national defense; hence, it should restrict some products that threaten an industry
considered vital to its defense.

The infant industry argument suggests that new domestic industries should be protected from older,
established foreign competitors. As the new industry matures, it will be able to stand on its own feet and
compete effectively with foreign producers, at which time protection can be removed. The national
defense argument suggests that certain industries are vital to national defense and should be protected
from foreign competitors so that a domestic supply of necessary materials would be available in case of
an international conflict.

------------------------------

All of the following are major participants in the foreign exchange market except:

* Commercial banks
* Foreign exchange brokers
* Country central banks
* Multinational corporations
* Individual travelers exchanging currencies
That answer is incorrect.
Correct answer:
Individual travelers exchanging currencies

While foreign travelers participate in the foreign exchange markets, they participate at relatively small
levels, compared to the other players.

------------------------------

The General Agreement of Tariffs and Trade (GATT) is

* an organization established in the early 1960s to protect U.S. industries from low-cost foreign
producers.
* an organization established after World War II to set the rules for international trade and reduce trade
barriers among member nations.
* legislation that substantially lowered tariffs in the early 1930s.
* legislation that substantially increased tariffs in the early 1930s.

That answer is incorrect.


Correct answer:
an organization established after World War II to set the rules for international trade and reduce trade
barriers among member nations.

GATT is an organization composed of 115 countries designed to set the rules for the conduct of
international trade and reduce barriers to trade among nations. GATT has successfully reduced tariffs and
relaxed quotas.

------------------------------

If FF/$ = 4.00 and DM/$ = 3.50 then

* FF/DM = 1.14
* FF/DM = 1
* None of these answers
* DM/FF = 1.14
* FF/DM = .875

That answer is correct!

To find FF/DM when given FF/$ and DM/$ implies creating the fraction FF/DM. When this fraction is
created, we get: (FF/$) / (DM/$) or (4.00) / (3.5) = 1.14 = FF/DM.

------------------------------

You might also like